OB FINAL EXAM

Réussis tes devoirs et examens dès maintenant avec Quizwiz!

CSE (combined spinal epidural)

"walking epidural" if low dose immediate relief of a spinal block and continuous relief of an epidural

Traditionally, a ______ ml blood loss after a vaginal birth and a _____ ml blood loss after a cesarean birth constitute PPH. However, medical personnel tend to underestimate blood loss by as much as 50%.

500 and 1,000

A nurse is monitoring a healthy newborn's blood glucose level 90 minutes after birth. Which result should the nurse anticipate in terms of mg/dL? 80 to 100 60 to 70 Less than 40 55 to 60

55 to 60

HbA1c goal

< 6.5%

A group of nurses are discussing the concept of pain experience during labor. Which statement should the nurses identify as correct? A: Sensory pain for nulliparous women often is greater than for multiparous women during early labor. A: Affective pain for nulliparous women usually is less than for multiparous women throughout the first stage of labor. A: Women with a history of substance abuse experience more pain during labor. A: Multiparous women have more fatigue from labor and therefore experience more pain.

A

A group of nurses are reviewing Category Characteristics of Fetal Monitoring. Which finding should the nurses identify as being representative of Category I ? A: Early decelerations, either present or absent B: Bradycardia not accompanied by baseline variability C: Tachycardia D: Sinusoidal pattern

A

A group of nursing students are discussing the condition and reconditioning of the urinary system after childbirth. Which statement should the nursing students identify as correct? A: Fluid loss through perspiration and increased urinary output account for a weight loss of more than 2 kg during the puerperium. B: Kidney function returns to normal a few days after birth. C: With adequate emptying of the bladder, bladder tone is usually restored 2 to 3 weeks after childbirth. D: Diastasis recti abdominis is a common condition that alters the voiding reflex

A

A mother expresses fear about changing her infant's diaper after he is circumcised. What should the nurse teach the mother about providing caring for the infant upon discharge? A. Cleanse the penis gently with water and put petroleum jelly around the glans after each diaper change B. Apply constant, firm pressure by squeezing the penis with the fingers for at least 5 minutes if bleeding occurs C. Cleanse the penis with prepackaged diaper wipes every 3 to 4 hours D. Wash off the yellow exudate that forms on the glans at least once every day to prevent infection

A

A nurse is reviewing concepts relative to fetal circulation. Which factor should the nurse identify as not affecting fetal circulation during labor? A: fetal position B: uterine contractions C: blood pressure D: umbilical cord blood flow

A

A nurse is reviewing the onset of labor. Which sign should the nurse identify as not preceding the onset of labor? A: A decline in energy, as the body stores up for labor B: Stronger and more frequent uterine (Braxton Hicks) contractions C: A return of urinary frequency as a result of increased bladder pressure D: Persistent low backache from relaxed pelvic joints

A

A nurse is working with a pregnant client who is inquring about childbirth preparation. Based on current practice, which method should the nurse indicate as being most effective? A: Encouraging expectant parents to attend childbirth preparation in any or no specific method B: The Dick-Read (natural) childbirth method C: The Bradley (husband-coached) method D: The Lamaze (psychoprophylactic) method

A

A nurse must administer erythromycin ophthalmic ointment to a newborn after birth. Which action should the nurse include when administering the medication? A. Cleanse eyes from inner to outer canthus before administration if necessary B. Flush eyes 10 minutes after instillation to reduce irritation C. Apply directly over the cornea D. Instill within 15 minutes of birth for maximum effectiveness

A

Following birth, the nurse assigns an Apgar score of 10 at 1 minute to a newborn. How would the nurse explain this score? A. An infant having no difficulty adjusting to extrauterine life but who should be assessed again at 5 minutes after birth B. An infant having no difficulty adjusting to extrauterine life and needing no further testing C. A prediction of a future free of neurologic problems D. An infant in severe distress that needs resuscitation

A

The maternity nurse must be cognizant that cultural practices have significant influence on infant feeding methods. Which statement should the maternity identify as correct? A. A common practice among Mexican women is known as las dos cosas B. Muslim cultures do not encourage breastfeeding because of modesty concerns C. Latino women born in the United States are more likely to breastfeed D. East Indian and Arab women believe that cold foods are best for a new mother

A

Which opiate causes euphoria, relaxation, drowsiness, and detachment from reality and has possible effects on the pregnancy, including preeclampsia, intrauterine growth restriction, and premature rupture of membranes? A. Heroin B. Alcohol C. Phencyclidine palmitate (PCP) D. Cocaine

A

While caring for the newborn, the nurse must be alert for any signs of cold stress. Which finding should the nurse anticpate? A. Increased respiratory rate B. Decreased activity level C. Hyperglycemia D. Shivering

A

The nurse examines a woman 1 hour after birth. The woman's fundus is boggy, midline, and 1 cm below the umbilicus. Her lochial flow is profuse, with two plum-sized clots. What is the nurse's intial action? A: Massage her fundus B: Place her on a bedpan to empty her bladder C: Administer methylergonovine (Methergine), 0.2 mg IM, which has been ordered prn D: Call the physician

A A boggy or soft fundus indicates that uterine atony is present. This is confirmed by the profuse lochia and passage of clots. The first action is to massage the fundus until firm. The physician can be called or methylergonovine administered after the fundus massage, especially if the fundus does not become or remain firm with massage. There is no indication of a distended bladder, so having the woman urinate will not alleviate the problem

A thrombosis results from the formation of a blood clot or clots inside a blood vessel and is caused by inflammation or partial obstruction of the vessel. Three thromboembolic conditions are of concern during the postpartum period; which of the following is not? A. Amniotic fluid embolism (AFE) B. Superficial venous thrombosis C. Deep vein thrombosis D. Pulmonary embolism

A An AFE occurs during the intrapartum period, when amniotic fluid containing particles of debris enters the maternal circulation. Although AFE is rare, the mortality rate is as high as 80%. A superficial venous thrombosis includes involvement of the superficial saphenous venous system. With deep vein thrombosis, the involvement varies but can extend from the foot to the iliofemoral region. A pulmonary embolism is a complication of deep vein thrombosis, occurring when part of a blood clot dislodges and is carried to the pulmonary artery, where it occludes the vessel and obstructs blood flow to the lungs.

A nurse is working on the labor and delivery unit. Under which circumstance would a nurse not perform a vaginal examination on a client in labor? A: When accelerations of the fetal heart rate (FHR) are noted B: An admission to the hospital at the start of labor C: When membranes rupture D: On maternal perception of perineal pressure or the urge to bear down

A An accelerated FHR is a positive sign not requiring vaginal examination; variable decelerations, however, merit a vaginal examination.

A nurse is caring for a pregnant client in labor using tocolytic therapy. Which statement should the nurse identify as correct? A. Its most important function is to afford the opportunity to administer antenatal glucocorticoids. B. There are no important maternal (as opposed to fetal) contraindications. C. The drugs can be given efficaciously up to the designated beginning of term at 37 weeks. D. If pulmonary edema develops while the client is receiving tocolytics, IV fluids should be given.

A Buying time for antenatal glucocorticoids to accelerate fetal lung development might be the best reason to use tocolytics. Once the pregnancy has reached 34 weeks, the risks of tocolytic therapy outweigh the benefits. There are important maternal contraindications to tocolytic therapy. Tocolytic-induced edema can be caused by IV fluids.

Which TORCH infection could be contracted by the infant because the mother owned a cat? A. Toxoplasmosis B. Varicella-zoster C. Parvovirus B19 D. Rubella

A Cats that eat birds infected with the Toxoplasma gondii protozoan excrete infective oocysts. Humans (including pregnant women) can become infected if they fail to wash their hands after cleaning a cat's litter box. The infection is passed through the placenta. The varicella-zoster virus is responsible for chickenpox and shingles. Approximately 90% of childbearing women are immune. During pregnancy, infection with parvovirus can result in abortion, fetal anemia, hydrops, intrauterine growth restriction (IUGR), and stillbirth; this virus is spread by vertical transmission, not by felines. Since vaccination for rubella was begun in 1969, cases of congenital rubella infection have been reduced significantly. Vaccination failures, lack of compliance, and the migration of nonimmunized persons result in periodic outbreaks of rubella (German measles).

With regard to injuries to the infant's plexus during labor and birth, nurses should be aware that: A. If the nerves are stretched with no avulsion, they should recover completely in 3 to 6 months B. Erb palsy is damage to the lower plexus C. Parents of children with brachial palsy are taught to pick up the child from under the axillae D. Breastfeeding is not recommended for infants with facial nerve paralysis until the condition resolves

A If the ganglia are disconnected completely from the spinal cord, the damage is permanent. Erb palsy is damage to the upper plexus and is less serious than brachial palsy. Parents of children with brachial palsy are taught to avoid picking up the child under the axillae or by pulling on the arms. Breastfeeding is not contraindicated in facial nerve paralysis, but both mother and infant will need help from the nurse at the start.

During the assessment of a preterm infant, the nurse notices continued respiratory distress even though oxygen and ventilation have been provided. In this situation, which condition should the nurse suspect? a. Hypovolemia and/or shock b. Excessively cool environment c. Central nervous system (CNS) injury d. Pending renal failure

A Other symptoms might include hypotension, prolonged capillary refill, and tachycardia, followed by bradycardia. Intervention is necessary. Preterm infants are susceptible to temperature instability. The goal of thermoregulation is to provide a neutral thermal environment. Hypoglycemia is likely to occur if the infant is attempting to conserve heat. CNS injury is manifested by hyperirritability, seizures, and abnormal movements of the extremities. Urine output and testing of specific gravity are appropriate interventions for the infant with suspected renal failure. This neonate is unlikely to be delivered with respiratory distress.

Screening questions for alcohol and drug abuse should be included in the overall assessment during the first prenatal visit for all women. The 4 Ps-Plus is a screening tool designed specifically to identify when there is a need for a more in-depth assessment. Which of the following is not included in the 4 Ps-Plus screening tool? A. Present B. Partner C. Past D. Pregnancy

A Parents: The woman should be asked, "Did either of your parents have a problem with alcohol or drugs?" Partner: "Does your partner have a problem with alcohol or drugs?" Past: "Have you ever had any beer, wine, or liquor?" Pregnancy: "In the month before you knew you were pregnant, how many cigarettes did you smoke? How much beer, wine, or liquor did you drink?"

Which of the following is the most common kind of placental adherence seen in pregnant women? A. Accreta B. Placenta Previa C. Percreta D. Increta

A Placenta accreta is the most common kind of placental adherence seen in pregnant women and is characterized by slight penetration of myometrium. In placenta previa, the placenta does not embed correctly and results in what is known as a low-lying placenta. It can be marginal, partial, or complete in how it covers the cervical os, and it increases the patient's risk for painless vaginal bleeding during the pregnancy and/or delivery process. Placenta percreta leads to perforation of the uterus and is the most serious and invasive of all types of accrete. Placenta increta leads to deep penetration of the myometrium

The nurse is assessing the respiratory system of a newborn. Which statement should the nurse be aware of with regard to the respiratory development of the newborn? A. Crying increases the distribution of air in the lungs B. Seesaw respirations are no cause for concern in the first hour after birth C. Newborns must expel the fluid at uterine life from the respiratory system within a few minutes of birth D. Newborns are instinctive mouth breathers

A Respirations in the newborn can be stimulated by mechanical factors such as changes in intrathoracic pressure resulting from the compression of the chest during vaginal birth. With birth, the pressure on the chest is released, helping draw air into the lungs. The positive pressure created by crying helps keep the alveoli open and increases distribution of air throughout the lungs. Newborns continue to expel fluid for the first hour of life. They are natural nose breathers and may not have the mouth-breathing response to nasal blockage for 3 weeks. Seesaw respirations instead of normal abdominal respirations are not normal and should be reported.

A male infant at 26 weeks of gestation arrives from the delivery room intubated. The nurse weighs the infant, places him under the radiant warmer, and attaches him to the ventilator at the prescribed settings. A pulse oximeter and cardiorespiratory monitor are placed. The pulse oximeter is recording oxygen saturation values of 80%. The prescribed saturation value is 92%. The nurse's most appropriate action is to: A. Listen to breath sounds and ensure the patency of the endotracheal tube, increase oxygen, and notify a physician. B. Continue to observe and make no changes until the saturations are 75%. C. Continue with the admission process to ensure that a thorough assessment is completed. D. Notify the parents that their infant is not doing well.

A The actions described in A are appropriate nursing interventions to assist in optimal O2 saturation of the infant. Oxygenation of the infant is crucial. O2 saturation should be maintained at more than 92%, and the nurse should delay other tasks to stabilize the infant. The action described in D is not appropriate. Further assessment and intervention are warranted prior to determination of fetal status.

Which nursing intervention should be immediately performed after the forceps-assisted birth of an infant? a. Assessing the infant for signs of trauma b. Administering prophylactic antibiotic agents to the infant c. Applying a cold pack to the infant's scalp d. Measuring the circumference of the infant's head

A The infant should be assessed for bruising or abrasions at the site of application, facial palsy, and subdural hematoma. Prophylactic antibiotics are not necessary with a forceps delivery. A cold pack would place the infant at risk for cold stress and is contraindicated. Measuring the circumference of the head is part of the initial nursing assessment.

A nurse providing care to a woman in labor should be aware that cesarean birth: A. Is performed primarily for the benefit of the fetus. B. Can be either elected or refused by women as their absolute legal right. C. Is declining in frequency in the United States. D. Is more likely to be performed in the poor in public hospitals who do not receive the nurse counseling that wealthier clients do.

A The most common indications for cesarean birth are danger to the fetus related to labor and birth complications. Cesarean births are increasing in the United States. Wealthier women who have health insurance and who give birth in a private hospital are more likely to experience cesarean birth. A woman's right to elect cesarean surgery is in dispute, as is her right to refuse it if in doing so she endangers the fetus. Legal issues are not absolutely clear.

A pregnant woman's amniotic membranes have ruptured. A prolapsed umbilical cord is suspected. What intervention would be the nurse's highest priority? a. Placing the woman in the knee-chest position b. Covering the cord in sterile gauze soaked in saline c. Preparing the woman for a cesarean birth d. Starting oxygen by face mask

A The woman is assisted into a modified Sims position, Trendelenburg position, or the knee-chest position in which gravity keeps the pressure of the presenting part off the cord.Although covering the cord in sterile gauze soaked saline, preparing the woman for a cesarean, and starting oxygen by face mark are appropriate nursing interventions in the event of a prolapsed cord, the intervention of top priority would be positioning the mother to relieve cord compression.

A nurse is working with a client who is grieving over the loss of a stillborn. Which statement would the nurse identify as correct with regard to the emotional state of grief? A. Time limit for grief experiences is variable among individuals B. Aspects of grief occur simultaneously across family units C. It represents a linear process D. It is a static concept applied to loss

A There is no prescribed time limit for the expression of grief. Grief is a dynamic concept involving complex emotions. The expression of grief is individualized and may not occur simultaneously across family units. The process of grief represents an iterative process.

Which actions are examples of appropriate techniques to wake a sleepy infant for breastfeeding? (Select all that apply.) a. Unwrapping the infant b. Changing the diaper c. Talking to the infant d. Slapping the infant's hands and feet e. Applying a cold towel to the infant's abdomen

A, B, C

Which statements regarding physiologic jaundice are accurate? (Select all that apply) a. Neonatal jaundice is common; however, kernicterus is rare b. Appearance of jaundice during the first 24 hours or beyond day 7 indicates a pathologic process c. Because jaundice may not appear before discharge, parents need instruction on how to assess for jaundice and when to call for medical help d. Jaundice iscaused by reduced levels of serum bilirubin e. Breastfed babies have a lower incidence of jaundice

A, B, C

Pain should be regularly assessed in all newborns. If the infant is displaying physiologic or behavioral cues that indicate pain, then measures should be taken to manage the pain. Which interventions are examples of nonpharmacologic pain management techniques? (Select all that apply) a. Swaddling b. Nonnutritive sucking c. Skin-to-skin contact with the mother d. Sucrose e. Acetaminophen

A, B, C, D Swaddling, nonnutritive sucking, skin-to-skin contact with the mother, and sucrose are all appropriate nonpharmacologic techniques used to manage pain in neonates. Acetaminophen is a pharmacologic method of treating pain.

A nurse is discussing the signs and symptoms of mastitis with a mother who is breastfeeding. Which findings should the nurse include in the discussion? (select all that apply) a. Breast tenderness b. Warmth in the breast c. Area of redness on the breast often resembling the shape of a pie wedge d. Small white blister on the tip of the nipple e. Fever and flulike symptoms

A, B, C, E

Which medications are used to manage postpartum hemorrhage (PPH)? (Select all that apply.) a. Oxytocin b. Methergine c. Terbutaline d. Hemabate e. Magnesium sulfate

A, B, D Terbutaline and magnesium sulfate are tocolytic medications that are used to relax the uterus, which would cause or worsen PPH. *****Methergine is contraindicated in moms with high BP*****

Screening questions for alcohol and drug abuse should be included in the overall assessment during the first prenatal visit for all women. The 4 Ps Plusis a screening tool specifically designed to identify the need for a more in-depth assessment. Which are the correct components of the 4 Ps Plus? (Select all that apply) a. Parents b. Partner c. Present d. Past e. Pregnancy

A, B, D, E

Which risk factors are associated with necrotizing enterocolitis (NEC)? (Select all that apply.) a. Polycythemia b. Anemia c. Congenital heart disease d. Bronchopulmonary dysphasia e. Retinopathy

ABC Risk factors for NEC include asphyxia, RDS, umbilical artery catheterization, exchange transfusion, early enteral feedings, patent ductus arteriosus (PDA), congenital heart disease, polycythemia, anemia, shock, and gastrointestinal infection. Bronchopulmonary dysphasia and retinopathy are not associated with NEC.

What are the complications and risks associated with cesarean births? Select all that apply a. Pulmonary edema b. Wound dehiscence c. Hemorrhage d. Urinary tract infections e. Fetal injuries

ABCDE Pulmonary edema, wound dehiscence, hemorrhage, urinary tract infections, and fetal injuries are possible complications and risks associated with cesarean births.

Women who are obese are at risk for several complications during pregnancy and birth. Which of these would the nurse anticipate with an obese client? Select all that apply a. Thromboembolism b. Cesarean birth c. Wound infection d. Breech presentation e. Hypertension

ABCE A breech presentation is not a complication of pregnancy or birth for the client who is obese. Venous thromboembolism is a known risk for obese women. Therefore, the use of thromboembolism-deterrent (TED) hose and sequential compression devices may help decrease the chance for clot formation. Women should also be encouraged to ambulate as soon as possible. In addition to having an increased risk for complications with a cesarean birth, in general, obese women are also more likely to require an emergency cesarean birth. Many obese women have a pannus(i.e., large roll of abdominal fat) that overlies a lower transverse incision made just above the pubic area. The pannus causes the areato remain moist, which encourages the development of infection. Obese women are more likely to begin pregnancy with comorbidities such as hypertension and type 2 diabetes.

Sedatives

AKA: H1 receptor antagonists a systemic analgesic helpful early on for anxiety and apprehension can cause fetal respiratory depression Ex: Benadryl, Phenergan

A woman is diagnosed with having a stillborn. At first she appears stunned by the news, cries a little, and then asks the nurse to call her mother. What phase of bereavement does the nurse identify that the woman is experiencing? Grief Reorganization Intense grief Acute distress

Acute Distress The immediate reaction to news of a perinatal loss or infant death encompasses a period of acute distress. Disbelief and denial can occur. However, parents also feel very sad and depressed. Intense outbursts of emotion and crying are normal, but lack of affect, euphoria, and calmness may occur and may reflect numbness, denial, or personal ways of coping with stress. Grief applies to the grief related to a potential loss of an infant. The parent grieves in preparation of the infant's possible death, although the parent clings to the hope that the child will survive. Intense grief occurs in the first few months after the death of the infant. This phase encompasses many different emotions, including loneliness, emptiness, yearning, guilt, anger, and fear. Reorganization occurs after a long and intense search for meaning. Parents are better able to function at work and home, experience a return of self-esteem and confidence, can cope with new challenges, and have placed the loss in perspective.

A nurse is monitoring an obstetric client who is in early labor. Which of the following findings would be a cause for concern if observed by the nurse? Android pelvis Biparietal diameter of less than 9.25cm Vertex presenting part General flexion attitude Transverse lie

Android pelvis Biparietal diameter of less than 9.25cm Transverse lie

A nursing student is reviewing concepts related to infant feeding. Which statement should the nurse identify as being correct concerning tandem feeding? A. Supplementing breastfeeding with bottle feeding to maintain adequate weight gain. B. Breastfeeding an infant and an older sibling during the same period. C. Using both breasts to nurse the baby. D. Adequate nutritional stores for the mother and infant

B

Which statement regarding Postpartum Depression (PPD) is essential for the nurse to be aware of when attempting to formulate a nursing diagnosis? A: PPD symptoms are consistently severe B: PPD can easily go undetected C: Only mental health professionals should teach new parents about this condition D: This syndrome affects only new mothers

B

A nurse is reviewing the concept of uterine rupture. Which factors would the nurse identify as leading to an increased likelihood of this occurence? (Select all that apply.) A. G3P3 with all vaginal deliveries B. Client who had a primary caesarean section with a classic incision C. Short interval between pregnancies D. Client receiving a trial of labor (TOL) following a VBAC delivery E. Preterm singleton pregnancy

B, C, D The shorter the interval between pregnancies/deliveries, the higher the risk of uterine rupture. A client who is having a TOL following a VBAC and a client who has had a C section with a classic incision into the uterus are at increased risk for uterine rupture. A pregnant woman with a singleton pregnancy (one fetus), even if preterm, is not considered to be at increased risk for uterine rupture; nor is a multipara who has delivered all her infants vaginally.

A nurse is reviewing possible etiologies for hyperbilirubinemia in the newborn. Which findings would the nurse expect to lead to increased bilirubin levels in the newborn? (Select all that apply) A. Cord clamped immediately following delivery of newborn B. Initiation of newborn feedings delayed following birth C. Twin-to-twin transfusion syndrome D. Hyperglycemia E. Meconium passed after 24 hours

B, C, E

A group of nurses are discussing care options for lesbian partners in childbearing experiences. Which opportunity should the nurses identified as not being able to be provided to male partners? Cutting the cord Breastfeeding the infant Rooming-in during hospitalization Labor support

Breastfeeding the infant

A nurse is caring for a client in labor. Which observation should the nurse indicate as being an accurate assessment? A: Use of the Valsalva maneuver is encouraged during the second stage of labor to relieve fetal hypoxia. B: Having the woman point her toes reduces leg cramps. C: The endogenous endorphins released during labor raise the woman's pain threshold and produce sedation. D: The woman's blood pressure increases during contractions and falls back to prelabor normal between contractions

C

A nurse is caring for a client who is bottlefeeding but has engorged breasts. What action should the nurse implement? A: Allow warm water to soothe the breasts during a shower B: Express milk from breasts occasionally to relieve discomfort C: Wear a snug, supportive bra D: Place absorbent pads with plastic liners into her bra to absorb leakage

C

A nurse is making a home visit to a postpartum woman 1 week after childbirth. Which client observation should the nurse expect? A: Exhibit a reduced attention span, limiting readiness to learn B: Have reestablished her role as a spouse or partner C: Vacillate between the desire to have her own nurturing needs met and the need to take charge of her own care and that of her newborn D: Express a strong need to review the events and her behavior during the process of labor and birth

C

A nurse is placing a newborn under a radiant heat warmer for temperature stabilization. Which action should the nurse include during this procedure? A. Cover the probe with a non-reflective material B. Recheck temperature by periodically taking a rectal temperature C. Perform all examinations and activities under the warmer D. Place the thermistor probe on the left side of the chest

C

What is the primary purpose for the use of tocolytic therapy to suppress uterine activity? a. Drugs can be efficaciously administered up to the designated beginning of term at 37 weeks gestation b. Tocolytic therapy has no important maternal (as opposed to fetal) contraindications c. The most important function of tocolytic therapy is to provide the opportunity to administer antenatal glucocorticoids d. If the client develops pulmonary edema while receiving tocolytic therapy, then intravenous (IV) fluids should be given.

C Buying time for antenatal glucocorticoids to accelerate fetal lung development may be the best reason to use tocolytic therapy. Once the pregnancy has reached 34 weeks, however, the risks of tocolytic therapy outweigh the benefits. Important maternal contraindications to tocolytic therapy exist. Tocolytic-induced edema can be caused by IV fluids.

A nurse is taking care of a client in labor who is exhibiting signs and symptoms of maternal hypotension syndrome. Which action should the nurse implement? Regional anesthesia Oxytocin administration Change in position Intravenous analgesic

Change in position

A group of nursing students are reviewing the process of bathing for a newborn. Which statement should the nursing students identify as being incorrect? A. Tub baths may be given before the infant's umbilical cord falls off and the umbilicus is healed B. Only plain warm water should be used to preserve the skin's acid mantle C. Powders are not recommended because the infant can inhale powder D. Newborns should be bathed every day, for the bonding as well as the cleaning

D

A nurse is caring for a patient in labor. Which observation by the nurse would indicate that the second stage of labor, the descent phase, has begun? A: The presenting part is below the ischial spines. B: The amniotic membranes rupture. C: The woman experiences a strong urge to bear down. D: The cervix cannot be felt during a vaginal examination.

D

A nurse is assessing a client at 42 weeks of gestation. Which finding, if noted by the nurse requires more assessment? A. Cervix dilated 2 cm and 50% effaced B. Score of 8 on the biophysical profile C. Fetal heart rate of 116 beats/min D. One fetal movement noted in 1 hour of assessment by the mother

D Self-care in a postterm pregnancy should include performing daily fetal kick counts three times per day. The mother should feel four fetal movements per hour. If she feels fewer than four movements, she should count for 1 more hour. Fewer than four movements in that hour warrants evaluation. The findings described in the other choices are normal at 42 weeks of gestation.

A woman gave birth to a 7-lb, 3-oz boy 2 hours ago. The nurse determines that the woman's bladder is distended because her fundus is now 3 cm above the umbilicus and to the right of the midline. What does the nurse identify as being the most serious complication based on this finding? Urinary tract infection A ruptured bladder Excessive uterine bleeding Bladder wall atony

Excessive uterine bleeding

A nurse observes a postpartum client to have excessive blood loss. Which cause should the nurse identify as being the most common cause for this finding? Unrepaired lacerations of the vagina or cervix Vaginal or vulvar hematomas Failure of the uterine muscle to contract firmly Retained placental fragments

Failure of the uterine muscle to contract firmly Although vaginal or vulvar hematomas, unrepaired lacerations, and retained placental fragments are possible causes of excessive blood loss, uterine muscle failure (uterine atony) is the most common cause. Uterine atony can best be thwarted by maintaining good uterine tone and preventing bladder distention.

A nurse is reviewing factors leading to decreased variabilty. Which cause should the nurse determine as being the most common for decreased variability in the fetal heart rate that lasts 30 minutes or less? ltered cerebral blood flow Umbilical cord compression Fetal sleep cycles Fetal hypoxemia

Fetal sleep cycles

Two hours after giving birth, a primiparous woman becomes anxious and complains of intense perineal pain with a strong urge to have a bowel movement. Her fundus is firm, at the umbilicus, and midline. Her lochia is moderate rubra with no clots. What clinical finding should the nurse expect? Bladder atony Constipation Hematoma formation Bladder distenstion

Hematoma formation Increasing perineal pressure along with a firm fundus and moderate lochial flow are characteristic of hematoma formation. Bladder distention results in an elevation of the fundus above the umbilicus and deviation to the right or left of midline. Uterine atony results in a boggy fundus. Constipation is unlikely at this time.

A nurse administers an opioid agonist analgesic to a woman in active labor. Which medication should the nurse identify as being used cautiously due to potential impact on the fetus? Meperidine (Demerol) Fentanyl (Sublimaze) Promethazine (Phenergan) Nalbuphine (Nubain)

Meperidine (Demerol)

A nurse caring for a woman in labor understands that increased variability of the fetal heart rate might be caused by: Tranquilizers Methamphetamines Barbiturates Narcotics

Methamphetamines

A client tells the nurse about the funeral arrangements for her newborn son. The client is thereby providing the nurse with information about: Expression of loss Mourning process Family reaction Grief process

Mourning process The mourning process is reflected by traditions and rituals such as the funeral arrangements. The grief process represents the emotional expression of loss. The expression of loss is related to the meaning of perception. Providing information related to funeral arrangements is not an indicator of family reaction.

The nurse is reviewing concepts related to healthy-parent infants bonding. The nurse recognizes that the process in which the infant's behaviors and characteristics call forth a corresponding set of maternal behaviors and characteristics is called: Claiming Mutuality Bonding Acquaintance

Mutuality

After Nubain is administered, labor progresses rapidly and the baby is born less than one hour later. The baby shows signs of respiratory depression. Which med should the nurse be prepared to administer to the newborn?

Narcan (naloxone)

Antidepressant medication is the mainstay treatment for maternal depression, with selective serotonin reuptake inhibitors (SSRIs) being the first line of pharmacotherapy. Reports of cardiac defects have been associated with the use of which SSRI? Citalopram Fluoxetine Sertraline Paroxetine

Paroxetine The American College of Obstetricians and Gynecologists (ACOG) has issued a recommendation that paroxetine be avoided both during pregnancy and in women considering pregnancy. There have also been reports linking paroxetine to other abnormalities, such as omphalocele, craniosynostosis, and anencephaly. The absolute risk of any congenital abnormality associated with use of citalopram, fluoxetine, or sertraline is small.

A nurse is monitoring an obstetrical client. Which test result would provide evidence to the nurse that there is fetal blood in maternal circulation? Negative Coombs test result Positive Fern test result Positive Coombs test result Positive Kleihauer-Betke test result

Positive Kleihauer-Betke test result A Kleihauer-Betke test determines the presence of fetal blood in maternal circulation. A positive fern test result would indicate the presence of amniotic fluid, noting ROM. A positive Coombs test result would indicate that the mother has Rh antibodies, and a negative result would indicate no presence of Rh antibodies.

The maternity nurse is reviewing the concept of primary and secondary powers. What are primary powers?

Primary powers are responsible for dilation and effacement of the cervix

After change of shift report, the nurse assumes care of a multiparous client in labor. The woman is complaining of pain that radiates to her abdominal wall, lower back, and buttocks, and down her thighs. Before implementing a plan of care, the nurse should understand that this type of pain is: Somatic Afterpain Referred Visceral

Referred

The nurse observes several interactions between a postpartum woman and her new son. What behavior, if exhibited by this woman, does the nurse identify as a possible maladaptive behavior regarding parent-infant attachment? Tells visitors how well her son is feeding Talks and coos to her son Cuddles her son close to her Seldom makes eye contact with her son

Seldom makes eye contact with her son

PP Maternal Vitals

Temp less than 100.4 but could be slightly increased due to fluid loss HR possible bradycardia, but mostly normal All other vitals should be normal

Phenergan (promethazine)

-ataractic (tranquilizer) -may be used to augment desirable effects of the opioid analgesics -few of the undesirable effects of opioid analgesics

Meperidine (Demerol)

-most commonly used around the world -opioid agonist analgesic -does cross the placenta causing neonatal sedation and behavioral change

Nalbuphine (Nubain)

-opioid agonist-antagonist -provides sufficient analgesia -does not impact fetal respiratory depression

Fentanyl (Sublimaze)

-used in epidural administration -quickly metabolized by the fetus with little side affects

The client has asked the nurse why her cervix has only changed from 1 to 2 cm in 3 hours of contractions occurring every 5 minutes. What is the nurse's best response to the client? 1. "Your cervix has also effaced, or thinned out, and that change in the cervix is also labor progress." 2. "When your perineal body thins out, your cervix will begin to dilate much faster than it is now." 3. "What did you expect? You've only had contractions for a few hours. Labor takes time." 4. "The hormones that cause labor to begin are just getting to be at levels that will change your cervix."

1

Which ratio would be used to restore effective circulating volume in a postpartum patient who is experiencing hypovolemic shock?

3:1 A 3:1 ratio, of 3 ml fluid infused for every 1 ml of estimated blood loss, is recommended to restore circulating volume

The client has stated that she wants to avoid an epidural and would like an unmedicated birth. Which nursing action is most important for this client? 1. Encourage the client to vocalize during contractions 2. Perform vaginal exams only between 3. Provide a CD of soft music with sounds of nature 4. Offer to teach the partner how to massage muscles

4

A nurse is reviewing the clinical manifestation of fetal bradycardia. What indication should the nurse identify as being the most common cause? Maternal hypothyroidism Fetal anemia Tocolytic treatment using ritodrine Viral infection

Viral infection

The nurse is teaching about pain relief in labor. If the client says "local anesthetics provide good labor pain relief" does the nurse need to do additional teaching?

Yes

A perinatal nurse is caring for a woman in the immediate postbirth period. Assessment reveals that the client is experiencing profuse bleeding. What is the most likely cause for this bleeding? a. Uterine atony b. Uterine inversion c. Vaginal hematoma d. Vaginal laceration

a

A pregnant client tells her nurse that she is worried about the blotchy, brownish coloring over her cheeks, nose, and forehead. The nurse can reassure her that this is a normal condition related to hormonal changes. What is the correct term for this integumentary finding? a. Melasma b. Linea nigra c. Striae gravidarum d. Palmar erythema

a

With one exception, the safest pregnancy is one during which the woman is drug and alcohol free. What is the optimaltreatment for women addicted to opioids? a. Methadone b. Detoxification c. Smoking cessation d. 4 Ps Plus

a

A nurse is examining a client who has been admitted for possible ectopic pregnancy who is approximately 8 weeks pregnant. Which finding if observed by the nurse would be a priority concern? a. Ecchymosis noted around umbilicus b. No FHT heard via Doppler c. Blood pressure 100/80 d. Scant vaginal bleeding noted on peri pad

a Because this client is most likely in the early stages of pregnancy, FHT would not be able to be auscultated at this time. Scant vaginal bleeding would not be a priority concern but should still be monitored by the nurse. Ecchymosis around the umbilicus indicates Cullen sign, which indicates hematoperitoneum, and may also develop in an undiagnosed, ruptured intraabdominal ectopic pregnancy.

A client is warm and asks for a fan in her room for her comfort. The nurse enters the room to assess the mother and her infant and finds the infant unwrapped in his crib with the fan blowing over him on high. The nurse instructs the mother that the fan should not be directed toward the newborn and that the newborn should be wrapped ina blanket. The mother asks why. How would the nurse respond? a. "Your baby may lose heat by convection, which means that he will lose heat from his body to the cooler ambient air. You should keep him wrapped and should prevent cool air from blowing on him." b. "Your baby may lose heat by conduction, which means that he will lose heat from his body to the cooler ambient air. You should keep him wrapped and should prevent cool air from blowing on him." c. "Your baby may lose heat by evaporation, which means that he will lose heat from his body to the cooler ambient air. You should keep him wrapped and should prevent cool air from blowing on him." d. "Your baby will easily get cold stressed and needs to be bundled up at all times."

a Convection is the flow of heat from the body surface to cooler ambient air. Because of heat loss by convection, all newborns in open bassinets should be wrapped to protect them from the cold.

Which order should the nurse expect for a client admitted with a threatened abortion? a. Bed rest b. Administration of ritodrine IV c. Nothing by mouth (nil per os[NPO]) d. Narcotic analgesia every 3 hours, as needed

a Decreasing the woman's activity level may alleviate the bleeding and allow the pregnancy to continue. Ritodrine is not the first drug of choice for tocolytic medications. Having the woman placed on NPO is unnecessary. At times, dehydration may produce contractions; therefore, hydration is important. Narcotic analgesia will not decrease the contractions and may mask the severity of the contractions.

With regard to protein in the diet of pregnant women, nurses should be aware that: a. Many protein-rich foods are also good sources of calcium, iron, and B vitamins. b. Many women need to increase their protein intake during pregnancy. c. As with carbohydrates and fat, no specific recommendations exist for the amount of protein in the diet. d. High-protein supplements can be used without risk by women on macrobiotic diets.

a Good protein sources such as meat, milk, eggs, and cheese have a lot of calcium and iron. Most women already eat a high-protein diet and do not need to increase their intake. Protein is sufficiently important that specific servings of meat and dairy are recommended. High-protein supplements are not recommended because they have been associated with an increased incidence of preterm births.

What is the most common reason for late postpartum hemorrhage (PPH)? a. Subinvolution of the uterus b. Defective vascularity of the decidua c. Cervical lacerations d. Coagulation disorders

a Although defective vascularity, cervical lacerations, and coagulation disorders of the decidua may also cause PPH, late PPH typically results from subinvolution of the uterus, pelvic infection, or retained placental fragments.

Mutuality

a shared set of behaviors that is a part of the bonding process

analgesics vs anesthetics

analgesics alleviate sensation while anesthetics abolish pain

How would the nurse best analyze the results from a client's sonogram that show's the fetal shoulder as the presenting part?

breech, transverse

While working with the pregnant client in her first trimester, what information does the nurse provide regarding when chorionic villus sampling (CVS) can be performed (in weeks of gestation)? a. 4 b. 8 c. 10 d. 14

c CVS can be performed in the first or second trimester, ideally between 10 and 13weeks of gestation. During this procedure, a small piece of tissue is removed from the fetal portion of the placenta. If performed after 9 completed weeks of gestation, then the risk of limb reduction is no greater than in the general population.

How many kilocalories per kilogram (kcal/kg) of body weight does a breastfed term infant require each day? a. 50 to 65 b. 75 to 90 c. 95 to 110 d. 150 to 200

c For the first 3 months, the infant needs 110 kcal/kg/day. At ages 3 to 6 months, the requirement is 100 kcal/kg/day. This level decreases slightly to 95 kcal/kg/day from 6 to 9 months and increases again to 100 kcal/kg/day until the baby reaches 12 months.

The most effective and least expensive treatment of puerperal infection is prevention. What is the most important strategy for the nurse to adopt? a. Large doses of Vitamin C during pregnancy b. Prophylactic antibiotics c. Strict aseptic technique by all health care personnel d. Limited protein and fat intake

c Strict adherence by all health care personnel to aseptic techniques during childbirth and the postpartum period is extremely important and the least expensive measure to prevent infection.

Which conditions are infants of diabetic mothers (IDMs) at a higher risk for developing? a. Iron deficiency anemia b. Hyponatremia c. Respiratory distress syndrome d. Sepsis

c. Respiratory distress syndrome. IDMs are at risk for macrosomia, birth trauma, perinatal asphyxia, respiratory distress syndrome, hypoglycemia, hypocalcemia, hypomagnesemia, cardiomyopathy, hyperbilirubinemia, and polycythemia. IDMs are not at risk for anemia, hyponatremia, or sepsis.

Neonatal signs of maternal SSRI use include: _____________________________________________. The onset of signs and symptoms ranges from several hours to several days after birth, but the signs generally resolve within 2 weeks.

continuous crying, irritability, jitteriness, shivering, fever, hypertonia, respiratory distress, feeding difficulty, hypoglycemia, and seizures

A nurse is responsible for teaching new parents regarding the hygienic care of their newborn. Which instruction should the nurse provide regarding bathing? a. Avoid washing the head for at least 1 week to prevent heat loss b. Sponge bathe the newborn for the first month of life c. Cleanse the ears and nose with cotton-tipped swabs, such as Q-tips d. Create a draft-free environment of at least 26° C (75° F)when bathing the infant

d

As a powerful central nervous system (CNS) stimulant, which of these substances can lead to miscarriage, preterm labor, placental separation (abruption), and stillbirth? a. Heroin b. Alcohol c. Phencyclidine (1-phenylcyclohexylpiperidine; PCP) d. Cocaine

d

Early this morning, an infant boy was circumcised using the PlastiBell method. Based on the nurse's evaluation, when will the infant be ready for discharge? a. When the bleeding completely stops b. When yellow exudate forms over the glans c. When the PlastiBell plastic rim (bell) falls off d. When the infant voids

d

How should the nurse interpret an Apgar score of 10 at 1 minute after birth? a. The infant is having no difficulty adjusting to extrauterine life and needs no further testing b. The infant is in severe distress and needs resuscitation c. The nurse predicts a future free of neurologic problems d. The infant is having no difficulty adjusting to extrauterine life but should be assessed again 5 mins after birth

d

If nonsurgical treatment for late postpartum hemorrhage (PPH) is ineffective, which surgical procedure would be appropriate to correct the cause of this condition? a. Hysterectomy b. Laparoscopy c. Laparotomy d. Dilation and curettage (D&C)

d

Pregnancy hormones prepare the vagina for stretching during labor and birth. Which change related to the pelvic viscera should the nurse share with the client? a. Because of several changes in the cervix, abnormal Papanicolaou (Pap) tests are easier to evaluate b. Quickening is a technique of palpating the fetus to engage it in passive movement c. The deepening color of the vaginal mucosa and cervix (Chadwick sign) usually appears in the second trimester or later as the vagina prepares to stretch during labor d. Increased vascularity of the vagina increases sensitivity and may lead to a high degree of arousal, especially in the second trimester

d

What is the most critical physiologic change required of the newborn after birth? a. Closure of fetal shunts in the circulatory system b. Full function of the immune defense system c. Maintenance of a stable temperature d. Initiation and maintenance of respirations

d

Which structure is responsible for oxygen and carbon dioxide transport to and from the maternal bloodstream? a .Decidua basalis b. Blastocyst c. Germ layer d. Chorionic villi

d

While evaluating the reflexes of a newborn, the nurse notes that with a loud noise the newborn symmetrically abducts and extends his arms, his fingers fan out and form a C with the thumb and forefinger, and he has a slight tremor. The nurse would document this finding as a which positive reflex. a. tonic neck b. glabellar (Myerson) c. Babinski d. Moro

d

hCG is an important biochemical marker for pregnancy and therefore the basis for many tests. Which statement regarding hCG is true? a. hCG can be detected as early as weeks after conception b. hCG levels gradually and uniformly increase throughout pregnancy c. Significantly lower-than-normal increases in the levels of hCG may indicate a postdate pregnancy d. Higher-than-normal levels of hCG may indicate an ectopic pregnancy or Down syndrome

d

A woman diagnosed with gestational diabetes has had little or no experience reading and interpreting glucose levels. The client shows the nurse her readings for the past few days. Which reading signals the nurse that the client may require an adjustment of insulin or carbohydrates? a. 75 mg/dl before lunch. This is low; better eat now b. 115 mg/dl 1 hour after lunch. This is a little high; maybe eat a little less next time c. 115 mg/dl 2 hours after lunch. This is too high; it is time for insulin d. 50 mg/dl just after waking up from a nap. This is too low; maybe eat a snack before going to sleep

d 50 mg/dl after waking from a nap is too low. During hours of sleep, glucose levels should not be less than 60 mg/dl. Snacks before sleeping can be helpful. The premeal acceptable range is 60 to 99 mg/dl. The readings 1 hour after a meal should be less than 129 mg/dl. Two hours after eating, the readings should be less than 120 mg/dl.

An African-American woman noticed some bruises on her newborn daughter's buttocks. The client asks the nurse what causes these. How would the nurse best explain this integumentary finding to the client? a. Lanugo b. Vascular nevus c. Nevus flammeus d. Mongolian spot

d A Mongolian spot is a bluish-black area of pigmentation that may appear over anypart of the exterior surface of the infant's body and is more commonly noted on the back and buttocks and most frequently observed on infants whose ethnic origins are Mediterranean, Latin American, Asian, or African. Lanugo is the fine, downy hair observed on a term newborn. A vascular nevus, commonly called a strawberry mark, is a type of capillary hemangioma. A nevus flammeus, commonly called a port wine stain, is most frequently found on the face.

A woman with severe preeclampsia is receiving a magnesium sulfate infusion. Which assessment finding if observed by the nurse would indicate a concern? a. Deep tendon reflexes of 2+ b. A sleepy, sedated affect c. Absence of ankle clonus d. A respiratory rate of 10 breaths/min

d A respiratory rate of 10 breaths/min indicates that the client is experiencing respiratory depression (bradypnea) from magnesium toxicity. Because magnesium sulfate is a central nervous system (CNS) depressant, the client will most likely become sedated when the infusion is initiated. Deep tendon reflexes of 2+ are a normal finding, as is absence of ankle clonus.

A nurse is monitoring a pregnant woman who is being treated with antihypertensive medications. Which medication should the nurse suspect could cause her infant to have a positive Coombs test result following delivery? a. Hydralazine (Apresoline) b. Labetalol hydrochloride (Trandate) c. Nifedipine (Procardia) d. Methyldopa (Aldomet)

d After 4 months of maternal therapy, a postive Coombs test result can occur in the infant. None of the other drugs listed would have this effect.

A woman who is 14 weeks pregnant tells the nurse that she always had a glass of wine with dinner before she became pregnant. She has abstained during her first trimester and would like to know whether it is safe for her to have a drink with dinner now. The nurse tells her: a. "Because you're in your second trimester, there's no problem with having one drink with dinner." b. "One drink every night is too much. One drink three times a week should be fine." c. "Because you're in your second trimester, you can drink as much as you like." d. "Because no one knows how much or how little alcohol it takes to cause fetal problems, the best course is to abstain throughout your pregnancy."

d Although the consumption of occasional alcoholic beverages may not be harmful to the mother or her developing fetus, complete abstinence is strongly advised. A safe level of alcohol consumption during pregnancy has not yet been established. The first trimester is a crucial period of fetal development, but pregnant women at all gestations are counseled to eliminate all alcohol from their diet.

A 40-year-old woman with a high body mass index (BMI) is 10 weeks pregnant. Which diagnostic tool is appropriate to suggest to her at this time? a. Biophysical profile b. Amniocentesis c. Maternal serum alpha-fetoprotein (MSAFP) d. Transvaginal ultrasound

d An ultrasound is the method of biophysical assessment of the infant that is performed at this gestational age. Transvaginal ultrasound is especially useful for obese women, whose thick abdominal layers cannot be penetrated adequately with the abdominal approach. A biophysical profile is a method of biophysical assessment of fetal well-being in the third trimester. An amniocentesis is performed after the fourteenth week of pregnancy. A MSAFP test is performed from week 15 to week 22 of the gestation (weeks 16 to 18 are ideal).

Which classification of placental separation is notrecognized as an abnormal adherence pattern? a. Placenta accreta b. Placenta increta c. Placenta percreta d. Placenta abruptio

d Placenta abruptiois premature separation of the placentaas opposed to partial or complete adherence

Which physiologic alteration of pregnancy most significantly affects glucose metabolism? a. Pancreatic function in the islets of Langerhans is affected by pregnancy b. Pregnant women use glucose at a more rapid rate than nonpregnant women c. Pregnant women significantly increase their dietary intake d. Placental hormones are antagonistic to insulin, thus resulting in insulin resistance

d Placental hormones, estrogen, progesterone, and human placental lactogen (HPL) create insulin resistance. Insulin is also broken down more quickly by the enzyme placental insulinase. Pancreatic functioning is not affected by pregnancy. The glucose requirements differ because of the growing fetus. The pregnant woman should increase herintake by 200 calories a day.

How does the nurse document a non-stress test (NST) during which two or more fetal heart rate (FHR) accelerations of 15 beats per minute or more occur with fetal movement in a 20-minute period? a. Nonreactive b. Positive c. Negative d. Reactive

d The NST is reactive (normal) when two or more FHR accelerations of at least 15 beats per minute (each with a duration of at least 15 seconds) occur in a 20-minute period. A nonreactive result means that the heart rate did not accelerate during fetal movement. A positive resultis not used with NST. CST uses positive as a result term. A negative resultis not used with NST. CST uses negative as a result term.

Which developmental finding is accurate with regard to fetal growth? a. Heart starts beating at 12 weeks b. Lungs take shape by 8 weeks c. Brain configuration is complete by 8 weeks d. Main blood vessels form by 8 weeks

d The heart starts beating by 4 weeks, the lungs take shape by 12 weeks, and brain configuration is complete by 12 weeks.

A primiparous woman is delighted with her newborn son and wants to begin breastfeeding as soon as possible. How should the client be instructed to position the infant to facilitate correct latch-on? a. The infant should be positioned with his or her arms folded together over the chest b. The infant should be curled up in a fetal position c. The woman should cup the infant's head in her hand d. The infant's head and body should be in alignment with the mother

d The infant's head and body should be in correct alignment with the mother and the breast during latch-on and feeding. The infant should be facing the mother with his arms hugging the breast.The baby's body should be held in correct alignment (i.e., ears, shoulder, and hips in a straight line) during feedings. The mother should support the baby's neck and shoulders with her hand and not push on the occiput.

Which infant response to cool environmental conditions is either not effective or not available to them? a. Constriction of peripheral blood vessels b. Metabolism of brown fat c. Increased respiratory rates d. Unflexing from the normal position

d The newborn's flexed position guards against heat loss, because it reduces the amount of body surface exposed to the environment. The newborn's body is able to constrict the peripheral blood vessels to reduce heat loss. Burning brown fat generates heat. The respiratory rate may rise to stimulate muscular activity, which generates heat.

Which suggestion about weight gain is not an accurate recommendation? a. Underweight women should gain 12.5 to 18 kg b. Obese women should gain at least 7 kg c. Adolescents are encouraged to strive for weight gains at the upper end of the recommended scale d. In twin gestations, the weight gain recommended for a single fetus pregnancy should simply be doubled

d Women bearing twins need to gain more weight (usually 16 to 20 kg) but not necessarily twice as much. Underweight women need to gain the most. Obese women need to gain weight during pregnancy to equal the weight of the products of conception. Adolescents are still growing; therefore, their bodies naturally compete for nutrients with the fetus.

The nurse has evaluated a client with preeclampsia by assessing deep tendon reflexes (DTRs). The result is a grade of 3+. Which DTR response most accurately describes this score? a. Sluggish or diminished b. Brisk, hyperactive, with intermittent or transient clonus c. Active or expected response d. More brisk than expected, slightly hyperactive

d DTRs reflect the balance between the cerebral cortex and the spinal cord. They are evaluated at baseline and to detect changes. A slightly hyperactive and brisk response indicates a grade 3+ response.

Human immunodeficiency virus (HIV) may be transmitted perinatally or during the postpartum period. Which statement regarding the method of transmission is most accurate? a. Only in the third trimester from the maternal circulation b. From the use of unsterile instruments c. Only through the ingestion of amniotic fluid d. Through the ingestion of breast milk from an infected mother

d. Through the ingestion of breast milk from an infected mother. Postnatal transmission of the HIV through breastfeeding and breast milk may occur. Transmission of the HIV from the mother to the fetus may occur through the placenta at various gestational ages. Transmission of the HIV from the use of unsterile instruments is highly unlikely; most health care facilities must meet sterility standards for all instrumentation.

Placenta accreta/increta/percreta

endometrium, myometrium, entire

platypelloid pelvis

flat pelvis

variability in FHR

fluctuation of the baseline in amplitude and frequency of 2 cycles per min absent: non-reassuring low: <5 bpm moderate: 6-25 bpm (reassuring) marked: >25 bpm (can be associated with fetal hypoxia)

afterpains

from contraction of uterus post partum (involution) induced by breastfeeding can use ibuprofen to help

most favorable pelvis for successful labor?

gynecoid

android pelvis

heart shaped pelvis

normal CV changes in pregnancy

increased CO increased blood volume increased HR decreased BP

spinal block

no sensation from nipples to feet for cesarean SE: maternal hypotension, fetal bradycardia, spinal headache, maternal bladder, uterine atony

anthropoid pelvis

oval shaped, with a wider anteroposterior diameter

acquaintance

part of attachment describes how parents get to know their baby during the immediate postpartum period through eye contact, touching, and talking

involution

return of the uterus to a nonpregnant state after birth

lochia

rubra - days 2-3, red serosa - day 4 until about 2 weeks, pink alba - 2 to 6 weeks, brown/yellow concern if: saturated pad in an hour or less, foul smell, blood squirting or spraying out clots normal, massage them out

Opioids

systemic analgesic give IV with contraction Ex: morphine, fentanyl, ultiva (PCA), Nubain morphine has serious respiratory depression Nubain least amount of side effects antidote: Narcan (naloxone), can be given to mom or baby

Claiming

the process by which parents identify their new baby in terms of likeness to other family members, the differences, and the baby's uniqueness

Bonding

the process through which over time parents form an emotional attachment to their infant

What is the primarynursing responsibility when caring for a client who is experiencing an obstetric hemorrhage associated with uterine atony? a. Establishing venous access b. Performing fundal massage c. Preparing the woman for surgical intervention d. Catheterizing the bladder

b

What would a steady trickle of bright red blood from the vagina in the presence of a firm fundus suggest to the nurse? a. Uterine atony b. Lacerations of the genital tract c. Perineal hematoma d. Infection of the uterus

b

Which finding in the urinalysis of a pregnant woman is considered a variation of normal? a. Proteinuria b. Glycosuria c. Bacteria in the urine d. Ketones in the urine

b

The nurse is observing a postpartum client who has been bleeding excessively during the first hour, saturating multiple pads. Which interventions would the nurse anticipate that the physician would order? (Select all that apply) 1. Administer oxygen via nonrebreather mask @ 10 L/minute 2. Insert a secondary intravenous line access 3. Document findings in the health care record 4. Type & screen for 2 units of blood 5. Decrease flow rate for intravenous fluid administration

1 and 2 Administration of oxygen @ 10L/minute via nonrebreather mask would be an anticipated order, as would insertion of a secondary line access for administration of fluids, blood, and/or medications. Although documentation of findings in a health care record is required, this is part of the nursing role and does not require an order by the physician. With regard to the presence of hypovolemic shock, intravenous fluids would be increased and maintained. The flow rate would not typically be decreased unless there was another comorbidity leading to potential fluid overload. Type & Screen would not be an anticipated order because no blood would be held for use; rather a Type & Cross order would be anticipated

A nurse is reviewing best practice for placing an infant to breast following birth. What timeframe should the nurse identify as representing a Baby-friendly hospital mandate? 1 hour 30 mins 4 hours 2 hours

1 hour

Which of the following are reasons for late PPH? 1. Subinvolution of the uterus 2. Pelvic infection 3. Coagulation disorders 4. Retained placental fragments 5. Cervical lacerations

1, 2, 4 The others are also causes of PPH, but not causes of LATE PPH

A nurse is assisting a breastfeeding mother with positioning of the baby. Which finding should the nurse be aware of? A. While supporting the head, the mother should push gently on the occiput B. Whatever the position used, the infant is held in direct skin with the mother C. Women with perineal pain and swelling prefer the modified cradle position D. The cradle position is usually preferred by mothers who had a cesarean birth

B

Which instruction should the nurse provide to reduce the risk of nipple trauma? a. Limit the feeding time to less than 5 minutes b. Position the infant so the nipple is far back in the mouth c. Assess the nipples before each feeding d. Wash the nipples daily with mild soap and water

b

Which statement regarding the structure and function of the placenta is correct? a. Produces nutrients for fetal nutrition b. Secretes both estrogen and progesterone c .Forms a protective, impenetrable barrier to microorganisms such as bacteria and viruses d. Excretes prolactin and insulin

b

The 5 As screening intervention tool is used to implement smoking cessation strategies on the basis of patient response. What do the 5 As stand for? A. Ask, advise, administer, approve, and admit B. Ask, assess, advise, assist, and arrange follow-up C. Assess, assist, advise, apply, and arrange D. Assess, apply, advise, ask, and arrange follow-up

B

If the newborn has excess secretions, the mouth and nasal passages can be easily cleared with a bulb syringe. How should the nurse instruct the parents on the use of this instrument? a. Avoid suctioning the nares b. Insert the compressed bulb into the center of the mouth c. Suction the mouth first d. Remove the bulb syringe from the crib when finished

c

Nurses must be cognizant of the growing problem of methamphetamine use during pregnancy. When caring for a woman who uses methamphetamines, it is important for the nurse to be aware of which factor related to the abuse of this substance? A. Methamphetamine is a depressant B. All methamphetamines are vasodilators C. Methamphetamine users are extremely psychologically addicted D. Rehabilitation is usually successful

C

The first and most important nursing intervention when a nurse observes profuse postpartum bleeding is to: A. Call the woman's primary health care provider B. Administer the standing order for an oxytocin C. Palpate the uterus and massage it if it is boggy D. Assess maternal blood pressure and pulse for signs of hypovolemic shock

C

Which intervention by the nurse would reduce the risk of abduction of the newborn from the hospital? a. Instructing the mother not to give her infant to anyone except the one nurse assigned to her that day b. Applying an electronic and identification bracelet to the mother and the infant c. Carrying the infant when transporting him or her in the halls d. Restricting the amount of time infants are out of the nursery

b

Which statements concerning the benefits or limitations of breastfeeding are accurate? (Select all that apply.) a. Breast milk changes over time to meet the changing needs as infants grow b. Breastfeeding increases the risk of childhood obesity c. Breast milk and breastfeeding may enhance cognitive development d. Long-term studies have shown that the benefits of breast milk continue after the infant is weaned e. Benefits to the infant include a reduced incidence of SIDS

A, C, D, E

A woman in preterm labor at 30 weeks of gestation receives two 12-mg intramuscular (IM) doses of betamethasone. What is the purpose of this pharmacologic intervention? a. To stimulate fetal surfactant production b. To reduce maternal and fetal tachycardia associated with ritodrine administration c. To suppress uterine contractions d. To maintain adequate maternal respiratory effort and ventilation during magnesium sulfate therapy

A Antenatal glucocorticoids administered as IM injections to the mother accelerate fetal lung maturity. Propranolol (Inderal) is given to reduce the effects of ritodrine administration. Betamethasone has no effect on uterine contractions. Calcium gluconate is given to reverse the respiratory depressive effects of magnesium sulfate therapy.

A nurse is working with a client in labor. Which of the following should the nurse not include in the plan of care? A: Administer antibiotics B: Observe for bloody, or pink, show C: To monitor the onset of progressive, regular contractions D: Assess for spontaneous rupture of membranes

A Antibiotic adminsitration is typically not included in the plan of care for a laboring client unless there is evidence of a Beta strept infection.

The nurse is providing discharge instructions related to the baby's respiratory system. Which statement should the nurse not include as part of discharge teaching? A. Don't let the infant sleep on his or her back B. Avoid loose bedding, waterbeds, and beanbag chairs C. Prevent exposure to people with upper respiratory tract infections D. Keep the infant away from secondhand smoke

A

Which of the following findings would raise concern for the nurse who is monitoring a postpartum client who had a spontaneous vaginal delivery (SVD) of a 10-lb baby boy? A: Fundus midline and firm with spurts of bright red blood upon fundal massage B: Lochia rubra with minimal clots expressed on fundal massage C: Fundus midline and firm with nonpalpable bladder D: Client report of mild to moderate cramping and request for pain medication

A

A nurse is preparing to perform a vaginal exam on a client in labor. Which principle should guide the nurse's action? A: Cleanse the vulva and perineum before and after the examination as needed B: Perform the examination every hour during the active phase of the first stage of labor C: Perform an examination immediately if active bleeding is present D: Wear a clean glove lubricated with tap water to reduce discomfort

A

A nurse is preparing to weight a newborn. Which action should the nurse include as part of the procedure? A. Weigh the newborn at the same time each day for accuracy B. Place a sterile scale paper on the scale for infection control C. Keep a hand on the newborn's abdomen for safety D. Leave its diaper on for comfort

A

A nurse is working with parents who have a sensory impairment. Which statement should the nurse identify as being inaccurate? A: Visually impaired mothers cannot overcome the infant's need for eye-to-eye contact B: The best approach for the nurse is to assess the parents' capabilities rather than focusing on their disabilities C: Technologic advances, including the Internet, can provide deaf parents with a full range of parenting activities and information D: One of the major difficulties visually impaired parents experience is the skepticism of health care professionals

A

Which description most accurately describes the augmentation of labor? a. Is part of the active management of labor that is instituted when the labor process is unsatisfactory b. Relies on more invasive methods when oxytocin and amniotomy have failed c. Is a modern management term to cover up the negative connotations of forceps-assisted birth d. Uses vacuum cups

A Augmentation is part of the active management of labor that stimulates uterine contractions after labor has started but is not progressing satisfactorily. Augmentation uses amniotomy and oxytocin infusion, as well as some more gentle, noninvasive methods. Forceps-assisted births are less common than in the past and not considered a method of augmentation. A vacuum-assisted delivery occurs during childbirth if the mother is too exhausted to push. Vacuum extraction is not considered an augmentation methodology.

A woman at 26 weeks of gestation is being assessed to determine whether she is experiencing preterm labor. What finding would indicate to the nurse that preterm labor is occurring? A. The cervix is effacing and dilated to 2 cm. B. Fetal fibronectin is present in vaginal secretions. C. Irregular, mild uterine contractions are occurring every 12 to 15 minutes. D. Estriol is not found in maternal saliva.

A Cervical changes such as shortened endocervical length, effacement, and dilation are predictors of imminent preterm labor. Changes in the cervix accompanied by regular contractions indicate labor at any gestation. Irregular, mild contractions that do not cause cervical change are not considered a threat. Estriol is a form of estrogen produced by the fetus that is present in plasma at 9 weeks of gestation. Levels of salivary estriol have been shown to increase before preterm birth. The presence of fetal fibronectin in vaginal secretions between 24 and 36 weeks of gestation could predict preterm labor, but it has only a 20% to 40% positive predictive value. Of more importance are other physiologic clues of preterm labor, such as cervical changes.

An infant at 26 weeks of gestation arrives intubated from the delivery room. The nurse weighs the infant, places him under the radiant warmer, and attaches him to the ventilator at the prescribed settings. A pulse oximeter and cardiorespiratory monitor are placed. The pulse oximeter is recording oxygen saturations of 80%. The prescribed saturations are 92%. What is the nurse's most appropriate action at this time? a. Listening to breath sounds, and ensuring the patency of the endotracheal tube, increasing oxygen, and notifying a physician b. Continuing to observe and making no changes until the saturations are 75% c. Continuing with the admission process to ensure that a thorough assessment is completed d. Notifying the parents that their infant is not doing well

A Listening to breath sounds and ensuring the patency of the endotracheal tube, increasing oxygen, and notifying a physician are appropriate nursing interventions to assist in optimal oxygen saturation of the infant. Oxygen saturation should be maintained above 92%, and oxygenation status of the infant is crucial. The nurse should delayother tasks to stabilize the infant. Notifying the parents that the infant is not doing well is not an appropriate action. Further assessment and intervention are warranted before determining fetal status.

A pregnant woman was admitted for induction of labor at 43 weeks of gestation with sure dates. A nonstress test (NST) in the obstetrician's office revealed a nonreactive tracing. On artificial rupture of membranes, thick meconium-stained fluid was noted. What should the nurse caring for the infant after birth anticipate? a. Meconium aspiration, hypoglycemia, and dry, cracked skin b. Excessive vernix caseosa covering the skin, lethargy, and RDS c. Golden yellow to green-stained skin and nails, absence of scalp hair, and an increased amount of subcutaneous fat d. Hyperglycemia, hyperthermia, and an alert, wide-eyed appearance

A Meconium aspiration, hypoglycemia, and dry, cracked skin are consistent with a postmature infant. Excessive vernix caseosa, lethargy, and RDS are consistent with a very premature infant. The skin may be meconium stained, but the infant will most likely have long hair and decreased amounts of subcutaneous fat. Postmaturity with a nonreactive NST is indicative of hypoxia. Signs and symptoms associated with fetal hypoxia are hypoglycemia, temperature instability, and lethargy.

A premature infant with respiratory distress syndrome (RDS) receives artificial surfactant. How does the nurse explain surfactant therapy to the parents? a. "Surfactant improves the ability of your baby's lungs to exchange oxygen and carbon dioxide." b. "The drug keeps your baby from requiring too much sedation." c. "Surfactant is used to reduce episodes of periodic apnea." d. "Your baby needs this medication to fight a possible respiratory tract infection."

A Surfactant can be administered as an adjunct to oxygen and ventilation therapy. With the administration of an artificial surfactant, respiratory compliance is improved until the infant can generate enough surfactant on his or her own. Surfactant has no bearing on the sedation needs of the infant. Surfactant is used to improve respiratory compliance, including the exchange of oxygen and carbon dioxide. The goal of surfactant therapy in an infant with RDS is to stimulate the production of surfactant in the type 2 cells of the alveoli. The clinical presentation of RDS and neonatal pneumonia may be similar. The infant may be started on broad-spectrum antibiotics to treat infection.

Which practices are ideal for role modeling when attempting to prevent sudden infant death syndrome (SIDS)? (Select all that apply) a. Fully supine position for all sleep b. Side-sleeping position as an acceptable alternative c. "Tummy time" for play d. Infant sleep sacks or buntings e. Soft mattress

A, C, D The "back to sleep" position is now recommended as the only position for every sleep period. To prevent positional plagiocephaly (flattening of the head) the infant should spend time on his or her abdomen while awake and for play. Loose sheets and blankets may be dangerous because they could easily cover the baby's head. The parents should be instructed to tuck any bedding securely around the mattress or use sleep sacks or bunting bags instead. The side-sleeping position is no longer an acceptable alternative position, according to the AAP. Infants should always sleep on a firm surface, ideally a firm crib mattress covered by a sheet only. Quilts and sheepskins, among other bedding, should not be placed under the infant.

The AAP recommends pasteurized donor milk for preterm infants if the mother's own milk in not available. Which statements regarding donor milk and milk banking are important for the nurse to understand and communicate to her client? (Select all that apply.) a. All milk bank donors are screened for communicable diseases b. Internet milk sharing is an acceptable source for donor milk c. Donor milk may be given to transplant clients d. Donor milk is used in neonatal intensive care units (NICUs) for severely low-birth-weight infants only e. Donor milk may be used for children with immunoglobulin A (IgA) deficiencies

A, C, E

Infants born between 34 0/7 and 36 6/7 weeks of gestation are called late-preterm infantsbecause they have many needs similar to those of preterm infants. Because they are more stable than early-preterm infants, they may receive care that is similar to that of a full-term baby. These infants are at increased risk for which conditions? Select all that apply a. Problems with thermoregulation b. Cardiac distress c. Hyperbilirubinemia d. Sepsis e. Hyperglycemia

ACD Thermoregulation problems, hyperbilirubinemia, and sepsis are all conditions related to immaturity and warrant close observation. After discharge, the infant is at risk for rehospitalization related to these problems. Association of Women's Health, Obstetric and Neonatal Nurses (AWHONN) launched the Near-Term Infant Initiative to study the problem and ways to ensure that these infants receive adequate care. The nurse should ensure that this infant is adequately feeding before discharge and that parents are taught the signs and symptoms of these complications. Late-preterm infants are also at increased risk for respiratory distress and hypoglycemia.

The induction of labor is considered an acceptable obstetric procedure if it is in the best interest to deliver the fetus. The charge nurse on the labor and delivery unit is often asked to schedule clients for this procedure and therefore must be cognizant of the specific conditions appropriate for labor induction. What are appropriate indications for induction? Select all that apply a. Rupture of membranes at or near term b. Convenience of the woman or her physician c. Chorioamnionitis (inflammation of the amniotic sac) d. Postterm pregnancy e. Fetal death

ACDE The conditions listed are all acceptable indications for induction. Other conditions include intrauterine growth restriction (IUGR), maternal-fetal blood incompatibility, hypertension, and placental abruption. Elective inductions for the convenience of thewoman or her provider are not recommended; however, they have become commonplace. Factors such as rapid labors and living a long distance from a health care facility may be valid reasons in such a circumstance. Elective delivery should not occur before 39weeks of completed gestation.

A 3.8-kg infant was delivered vaginally at 39 weeks after a 30-minute second stage. There was a nuchal cord. After birth the infant is noted to have petechiae over the face and upper back. What information should the nurse provide to the parents regarding the presence of petechiae? A. Should always be further investigated B. Are benign if they disappear within 48 hours of birth C. Usually occur with forceps delivery D. Result from increased blood volume.

B

A nurse administers Vitamin K to the newborn post delivery. The nurse understands that the reason for this medication to be given is? A. Reduce bilirubin levels B. Enhance the ability of blood to clot C. Stimulate the formation of surfactant D. Increase the production of red blood cells

B

A nurse examining a newborn infant notes that the infant is jaundiced. Which observation would lead the nurse to continue to monitor but not to intervene and contact the physician? A. Infant is being bottle fed and within the first 24 hours of life. B. Jaundice appeared on the third day of life. C. Jaundice appeared within the first 24 hours of life. D. Preterm infant who is 12 hours old

B

A nurse is reviewing the characteristics of uterine contractions. Which option should the nurse indicate as not being a characteristic? Appearance (shape and height) Frequency (how often contractions occur) Intensity (the strength of the contraction at its peak) Resting tone (the tension in the uterine muscle)

Appearance

The nurse is monitoring a client in labor who is experiencing back labor and complains of intense pain in her lower back. Which relief measure should the nurse implement? Apply counterpressure against the sacrum Effleurage Biofeedback Encourage pant-blow (breaths and puffs) breathing techniques

Apply counterpressure against the sacrum

A nurse has provided client teaching to a breastfeeding mother. Which action if observed by the nurse would indicate the need for further instruction? A. Puts her finger into newborn's mouth before removing breast B. Leans forward to bring breast toward the baby C. Holds breast with four fingers along bottom and thumb at top D. Stimulates the rooting reflex and then inserts nipple and areola into newborn's open mouth

B

A nurse is providing information to a client in labor with regard to tactile approaches to comfort management. Which option should the nurse include in the plan of care? A: Acupuncture can be performed by a skilled nurse with just a little training B: Hand and foot massage may be especially relaxing in advanced labor, when a woman's tolerance for touch is limited C: Therapeutic touch (TT) uses handheld electronic stimulators that produce sympathetic vibrations D: Either hot or cold applications may provide relief, but they should never be used together in the same treatment

B

A nurse is reviewing the concept of lochia. Which statement should the nurse identify as correct? A: Will usually decrease with ambulation and breastfeeding B: Should smell like normal menstrual flow unless an infection is present C: Is similar to a light menstrual period for the first 6 to 12 hours D: Is usually greater after cesarean births

B

A nurse is reviewing the concept of normal labor. Which statement should the nurse indicate as being incorrect? A: A regular progression of contractions, effacement, dilation, and descent occurs B: It is completed within 8 hours C: A single fetus presents by vertex D: No complications are involved

B

A nurse is reviewing the concept of weaning with regard to infant care. Which statement should the nurse identify as correct? A. Abrupt weaning is easier than gradual weaning B. Weaning can be mother or infant initiated C. Weaning should proceed from breast to bottle to cup D. The feeding of most interest should be eliminated first

B

A nurse is reviewing the use of systemic analgesics administered during labor. Which statement should the nurse indicate as correct? (select all that apply) A: Systemic analgesics cross the maternal blood-brain barrier as easily as they do the fetal blood-brain barrier B: Effects on the fetus and newborn can include decreased alertness and delayed sucking C: IV client-controlled analgesia (PCA) results in increased use of an analgesic D: IM administration is preferred over IV administration

B

A nurse is taking care of a client in the third stage of labor. Which statement should the nurse identify as correct? A: The major risk for women during the third stage is a rapid heart rate B: The duration of the third stage may be as short as 3 to 5 minutes C: The placenta eventually detaches itself from a flaccid uterus D: It is important that the dark, roughened maternal surface of the placenta appear before the shiny fetal surface

B

During rounds, a nurse suspects that a patient who has recently delivered via vaginal route is having excessive postpartum bleeding. Which intervention would be the priority action taken by the nurse at this time? A. Call the physician B. Massage the uterine fundus C. Increase rate of IV fluids D. Monitor pad count and perform catheterization

B

The birth weight of a breastfed newborn was 8 lb, 4 oz. On the third day the newborn's weight is 7 lb, 12 oz. Which action should the nurse take based on this finding? A. Notify the physician because the newborn is being poorly nourished B. Encourage the mother to continue breastfeeding because it is effective in meeting the newborn's nutrient and fluid needs C. Suggest that the mother switch to bottle feeding because breastfeeding is ineffective in meeting newborn needs for fluid and nutrients D. Refer the mother to a lactation consultant to improve her breastfeeding technique

B

When managing the care of a woman in the second stage of labor, the nurse uses various measures to enhance the progress of fetal descent. Which measure should the nurse include? A: Coaching the woman to use sustained, 10- to 15-second, closed-glottis bearing-down efforts with each contraction B: Encouraging the woman to try various upright positions, including squatting and standing C: Telling the woman to start pushing as soon as her cervix is fully dilated D: Continuing an epidural anesthetic so that pain is reduced and the woman can relax

B

A nurse is caring for a client whose labor is being augmented with oxytocin. The nurse recognizes that the oxytocin should be discontinued immediately if there is evidence of: A. Rupture of the client's amniotic membranes. B. A fetal heart rate (FHR) of 180 with absence of variability. C. Uterine contractions occurring every 8 to 10 minutes. D. The client needing to void.

B A fetal heart rate (FHR) of 180 with absence of variability is nonreassuring; the oxytocin should be immediately discontinued and the physician should be notified. The oxytocin should also be discontinued if uterine hyperstimulation occurs. Uterine contractions that occur every 8 to 10 minutes do not qualify as hyperstimulation. The client needing to void is not an indication to discontinue the oxytocin induction immediately or to call the physician. The oxytocin does not need to be discontinued when the membranes rupture, but the physician should be notified.

What is a maternal indication for the use of forceps-assisted birth? a. Wide pelvic outlet b. Maternal exhaustion c. History of rapid deliveries d. Failure to progress past station 0

B A mother who is exhausted may be unable to assist with the expulsion of the fetus. The client witha wide pelvic outlet will likely not require forceps extraction. With a rapid delivery, forceps extraction is not necessary. A station of 0 is too high for a forceps-assisted birth.

A Gravida III, Para 0 is concerned about the potential outcome for this pregnancy because all of her prior pregnancies have resulted in stillborn deliveries. Which diagnostic test would the nurse identify to assess for fetal well-being now that her pregnancy is at 32 weeks gestation? A. Chorionic villi sampling (CVS) B. Ultrasound C. Kleihauer-Betke test D. Contraction stress test (CST)

B An ultrasound could be used to determine fetal well-being. The Kleihauer-Betke test is a blood test to evaluate for the presence of fetal blood in maternal circulation; there is no evidence to support the use of this test at this time. CVS testing is typically done earlier in the pregnancy, between 10 and 12 weeks. There is no evidence to support the use of a CST at this time; determination of fetal well-being would first be evaluated with a nonstress test.

A number of methods can be used for inducing labor. Which cervical ripeningmethod falls under the category of mechanical or physical? a. Prostaglandins are used to soften and thin the cervix. b. Labor can sometimes be induced with balloon catheters or laminaria tents c. Oxytocin is less expensive and more effective than prostaglandins but creates greater health risks d. Amniotomy can be used to make the cervix more favorable for labor.

B Balloon catheters or laminaria tents are mechanical means of ripening the cervix. Ripening the cervix, making it softer and thinner, increases the success rate of induced labor. Prostaglandin E1 is less expensive and more effective than oxytocin but carries a greater risk. Amniotomy is the artificial rupture of membranes, which is used to induce labor only when the cervix is already ripe.

Which of the following would be considered to be an intrapartum risk factor for neonatal sepsis? A. Mechanical ventilation B. Chorioamnionitis C. Galactosemia D. Meconium aspiration

B Chorioamnionitis would be considered to be an intrapartum risk factor. The other conditions described are neonatal risk factors.

In appraising the growth and development potential of a preterm infant, the nurse should be cognizant of the information that is best described in which statement? a. Tell the parents that their child will not catch up until approximately age 10 years (for girls) to age 12 years (for boys) b. Correct for milestones, such as motor competencies and vocalizations, until the child is approximately 2 years of age c. Know that the greatest catch-up period is between 9 and 15 months postconceptual age d. Know that the length and breadth of the trunk is the first part of the infant to experience catch-up growth

B Corrections are made with a formula that adds gestational age and postnatal age. Whether a girl or boy, the infant experiences catch-up body growth during the first 2 to 3 years of life. Maximum catch-up growth occurs between 36 and 40 weeks of postconceptual age. The head is the first to experience catch-up growth.

For diagnostic and treatment purposes, nurses should know the birth weight classifications of high risk infants. For example, extremely low birth weight (ELBW) is the designation for an infant whose weight is: A. Less than 1500 g. B. Less than 1000 g. C. Less than 2000 g. D. Dependent on the gestational age

B ELBW is defined as less than 1000 g. At such weights, problems are so numerous that ethical issues regarding when to treat arise. Less than 1500 g is the designation for very low birth rate (VLBW). Less than 2000 g is less than LBW but too high for VLBW. Gestational age is a factor with weight in the condition of the preterm birth, but it is not part of the birth weight categorization.

A nurse is caring for a client who had a previous cesarian section and now presents with a transverse presentation in labor. Which information should the nurse provide to the client? A. "Because this is a repeat procedure, you are at the lowest risk for complications." B. "Even though this is your second cesarean birth, you may wish to review the preoperative and postoperative procedures." C. "You will not need preoperative teaching because this is your second cesarean birth." D. "Because this is your second cesarean birth, you will recover faster."

B Even though the client has previously had this surgical procedure, the prudent nurse should provide client teaching at this time. Maternal and fetal risks are associated with every cesarean section. Physiologic and psychologic recovery from a cesarean section is multifactorial and individual to each client each time. Preoperative teaching should always be performed regardless of whether the client has already had this procedure.

A nurse is caring for a laboring client who has a breech presentation. Which statement would the nurse identify as being most associated with this type of presentation? A. A rapid descent B. High rate of neuromuscular disorders C. Least common malpresentation D. Diagnosis by ultrasound only

B Fetuses with neuromuscular disorders have a higher rate of breech presentation, perhaps because they are less capable of movement within the uterus. Breech is the most common malpresentation, affecting 3% to 4% of all labors. Descent is often slow because the breech is not as good a dilating wedge as the fetal head. Diagnosis is made by abdominal palpation and vaginal examination, and is confirmed by ultrasound.

With regard to the classification of neonatal bacterial infection, nurses should be aware that: A. Congenital infection progresses slower than health care-associated infection B. Health care-associated infection can be prevented by effective handwashing; early-onset infection cannot C. Infections occur with about the same frequency in boy and girl infants, although female mortality is higher D. The clinical sign of a rapid, high fever makes infection easier to diagnose

B Handwashing is an effective preventive measure for late-onset (health care-associated) infections because these infections come from the environment around the infant. Early-onset (congenital) infections are caused by the normal flora at the maternal vaginal tract. Congenital (early-onset) infections progress more rapidly than health care-associated (late-onset) infections. Infection occurs about twice as often in boys and results in higher mortality. Clinical signs of neonatal infection are nonspecific and similar to noninfectious problems, making diagnosis difficult

With regard to infants who are diagnosed with both small gestational age (SGA) and intrauterine growth restriction (IUGR), the nurse should be aware of which information? a. In the first trimester, diseases or abnormalities result in asymmetric IUGR b. Infants with asymmetric IUGR have the potential for normal growth and development c. In asymmetric IUGR, weight is slightly larger than SGA, whereas length and head circumference are somewhat less than SGA d. Symmetric IUGR occurs in the later stages of pregnancy.

B IUGR is either symmetric or asymmetric. The symmetric form occurs in the first trimester; infants who are SGA have reduced brain capacity. The asymmetric form occurs in the later stages of pregnancy. Weight is less than the 10th percentile; head circumference is greater than the 10th percentile. Infants with asymmetric IUGR have the potential for normal growth and development.

A nurse is caring for a client who has a dysfunctional labor pattern. What should the nurse be aware of? A. Hypertonic uterine dysfunction is more common than hypotonic dysfunction. B. Women experiencing precipitous labor are about the only women experiencing dysfunctional labor who are not exhausted. C. Women who are underweight are more at risk. D. Abnormal labor patterns are most common in older women.

B Precipitous labor lasts less than 3 hours. Short women more than 30 pounds overweight are more at risk for dysfunctional labor. Hypotonic uterine dysfunction, in which the contractions become weaker, is more common. Abnormal labor patterns are more common in women younger than 20 years

In caring for the preterm infant, what complication is thought to be a result of high arterial blood oxygen level? a. Necrotizing enterocolitis (NEC) b. Retinopathy of prematurity (ROP) c. Bronchopulmonary dysplasia (BPD) d. Intraventricular hemorrhage (IVH)

B ROP is thought to occur as a result of high levels of oxygen in the blood. NEC is caused by the interference of blood supply to the intestinal mucosa. Necrotic lesions occur at that site. BPD is caused by the use of positive pressure ventilation against the immature lung tissue. IVH results from the rupture of the fragile blood vessels in the ventricles of the brain and is most often associated with hypoxic injury, increased blood pressure, and fluctuating cerebral blood flow.

Necrotizing enterocolitis (NEC) is an inflammatory disease of the gastrointestinal mucosa. The signs of NEC are nonspecific. Some generalized signs are: A. Hypertonia, tachycardia, and metabolic alkalosis B. Abdominal distention, temperature instability, and grossly bloody stools C. Hypertension, absence of apnea, and ruddy skin color D. Scaphoid abdomen, no residual with feedings, and increased urinary output

B Some generalized signs of NEC include decreased activity, hypotonia, pallor, recurrent apnea and bradycardia, decreased oxygen saturation values, respiratory distress, metabolic acidosis, oliguria, hypotension, decreased perfusion, temperature instability, cyanosis, abdominal distention, residual gastric aspirates, vomiting, grossly bloody stools, abdominal tenderness, and erythema of the abdominal wall.

Necrotizing enterocolitis (NEC) is an inflammatory disease of the gastrointestinal mucosa. The signs of NEC are nonspecific. What are generalized signs and symptoms of this condition? a. Hypertonia, tachycardia, and metabolic alkalosis b. Abdominal distention, temperature instability, and grossly bloody stools c. Hypertension, absence of apnea, and ruddy skin color d. Scaphoid abdomen, no residual with feedings, and increased urinary output

B Some generalized signs of NEC include decreased activity, hypotonia, pallor, recurrent apnea and bradycardia, decreased oxygen saturation values, respiratory distress, metabolic acidosis, oliguria, hypotension, decreased perfusion, temperature instability, cyanosis, abdominal distention, residual gastric aspirates, vomiting, grossly bloody stools, abdominal tenderness, and erythema of the abdominal wall. The infant may display hypotonia, bradycardia, and metabolic acidosis.

Which nursing intervention is paramount when providing care to a client with preterm labor who has received terbutaline? a. Assess deep tendon reflexes (DTRs) b. Assess for dyspnea and crackles c. Assess for bradycardia d. Assess for hypoglycemia.

B Terbutaline is a beta2-adrenergic agonist that affects the mother's cardiopulmonary and metabolic systems. Signs of cardiopulmonary decompensation include adventitious breath sounds and dyspnea. An assessment for dyspnea and crackles is important for the nurse to perform if the woman is taking magnesium sulfate. Assessing DTRs does not address the possible respiratory side effects of using terbutaline. Since terbutaline is a beta2-adrenergic agonist, it can lead to hyperglycemia, not hypoglycemia. Beta2-adrenergic agonist drugs cause tachycardia, not bradycardia.

A nurse is providing umbilical cord care to a newly delivered infant. What information should the nurse be aware of? A. The cord clamp is removed at cord separation B. The stump can easily become infected C. The average cord separation time is 5 to 7 days D. A nurse noting bleeding from the vessels of the cord should immediately call for assistance

B The cord stump is an excellent medium for bacterial growth. If bleeding occurs, the nurse should first check the clamp (or tie) and apply a second one; if the bleeding does not stop, then the nurse calls for assistance. The cord clamp is removed after 24 hours when it is dry. The average cord separation time is 10 to 14 days.

A pregnant client who is at term has been informed that her fetus has died and has been admitted to the obstetric unit. When developing a plan of care, the nurse would focus on which priority measure? A. Providing the client with phone numbers so as to make funeral arrangements B. Incorporating perinatal palliative care into the client's plan of care C. Including case management to participate in the client's care when she is admitted to the hospital D. Referral to a perinatologist

B The incorporation of a perinatal palliative care plan would be the priority intervention at this time to help the client and family members deal with the tragedy of the situation. At this point, a referral to a perinatologist would not be necessary because the determination has already been made that the fetus is dead. Although case management may be included in the plan of care and phone numbers may be provided to the client regarding funeral arrangements, these actions are not the priority measure.

With regard to small-for-gestational age (SGA) infants and intrauterine growth restriction (IUGR), nurses should be aware that: A. In the first trimester, diseases or abnormalities result in asymmetric IUGR B. Infants with asymmetric IUGR have the potential for normal growth and development C. In asymmetric IUGR, weight is slightly more than SGA, whereas length and head circumference are somewhat less than SGA D. Symmetric IUGR occurs in the later stages of pregnancy

B The infant with asymmetric IUGR has the potential for normal growth and development.IUGR is either symmetric or asymmetric. The symmetric form occurs in the first trimester, as a result of disease or abnormalities; SGA infants have reduced brain capacity. The asymmetric form occurs in the later stages of pregnancy. Weight is less than the 10th percentile, but the head circumference is greater than the 10th percentile (within normal limits).

A newborn was admitted to the neonatal intensive care unit (NICU) after being delivered at 29 weeks of gestation to a 28-year-old multiparous, married, Caucasian woman whose pregnancy was uncomplicated until the premature rupture of membranes and preterm birth. The newborn's parents arrive for their first visit after the birth. The parents walk toward the bedside but remain approximately 5 feet away from the bed. What is the nurse's most appropriate action? a. Wait quietly at the newborn's bedside until the parents come closer b. Go to the parents, introduce him or herself, and gently encourage them to meet their infant. Explain the equipment first, and then focus on the newborn. c. Leave the parents at the bedside while they are visiting so that they have some privacy. d. Tell the parents only about the newborn's physical condition and caution them to avoid touching their baby.

B The nurse is instrumental in the initial interactions with the infant. The nurse can help the parents seethe infant rather than focus on the equipment. The importance and purpose of the apparatus that surrounds their infant also should be explained to them. Parents often need encouragement and recognition from the nurse to acknowledge the reality of the infant's condition. Parents need to see and touch their infant as soon as possible to acknowledge the reality of the birth and the infant's appearance and condition. Encouragement from the nurse is instrumental in this process. Telling the parents to avoid touching their baby is inappropriate and unhelpful.

A nurse is providing instruction to a postpartum client regarding perineal care technique. When evaluating the postpartum woman's perineal care technique, the nurse would recognize the need for further instruction if the woman: A: Uses soap and warm water to wash the vulva and perineum B: Uses the peribottle to rinse upward into her vagina C: Washes from symphysis pubis back to the episiotomy D: Changes her perineal pad every 2 to 3 hours

B The peribottle should be used in a backward direction over the perineum. The flow should never be directed upward into the vagina because debris would be forced upward into the uterus through the still-open cervix. Using soap and warm water to wash the vulva and perineum is an appropriate measure. Washing from the symphysis pubis back to the episiotomy is an appropriate infection control measure. The client should be instructed to change her perineal pad every 2 to 3 hours

A pregnant woman at 29 weeks of gestation has been diagnosed with preterm labor. Her labor is being controlled with tocolytic medications. She asks when she might be able to go home. Which response by the nurse is most accurate? a. "After the baby is born." b. "When we can stabilize your preterm labor and arrange home health visits." c. "Whenever your physician says that it is okay." d. "It depends on what kind of insurance coverage you have."

B This client's preterm labor is being controlled with tocolytics. Once she is stable, home care may be a viable option for this type of client. Care of a client with preterm labor is multidisciplinary and multifactorial; the goal is to prevent delivery. In many cases, this goal may be achieved at home. Managed care may dictate an earlier hospital discharge or a shift from hospital to home care. Insurance coverage may be one factor in client care, but ultimately, client safety remains the most important factor.

During pregnancy, alcohol withdrawal may be treated using: A. Disulfiram (Antabuse) B. Corticosteroids C. Benzodiazepines D. Aminophylline

C

Following a vaginal delivery, the client tells the nurse that she intends to breastfeed her infant but she is very concerned about returning to her prepregnancy weight. On the basis of this interaction, the nurse would advise the client that: (Select all that apply.) A. She should join Weight Watchers as soon as possible to ensure adequate weight loss. B. Even though more calories are needed for lactation, typically women who breastfeed lose weight more rapidly than women who bottle feed in the postpartum period. C. If breastfeeding, she should regulate her fluid consumption in response to her thirst level. D. If she decreases her calorie intake by 100-200 calories a day she will lose weight more quickly. E. Weight loss diets are not recommended for women who breastfeed.

B, C, E

In which situations would the use of Methergine or prostaglandin be contraindicated even if the patient was experiencing a postpartum significant bleed? A. Patient has delivered twin pregnancies B. Patient's blood pressure postpartum is 180/90 C. Patient has a history of asthma D. Patient has a mitral valve prolapse E. Patient is a grand multip

B, C, D

What are the various modes of heat loss in the newborn? (Select all that apply) a. Perspiration b. Convection c. Radiation d. Conduction e. Urination

B, C, D

A nurse is working with clients who have experienced a perinatal loss. Which statements would not be advisable for the nurse to use as a basis for therapeutic discussion? (Select all that apply.) A. "You wanted a boy anyway, so now you have another chance" B. "I'm sorry" C. "I am sad for you" D. "This must be hard for you" E. "You're young, you can have other children"

B, C, D "This must be hard for you," "I'm sorry," and "I am sad for you" are acceptable statements following perinatal loss. "You're young, you can have other children" and "You wanted a boy anyway, so now you have another chance" would not be considered therapeutic.

Which of the following statements about the prevalence of perinatal mental health problems is true? SATA A. The World Health Organization (WHO) recognizes postpartum depression as the leading cause of disability in women. B. Up to a ¼ of pregnant women will experience some aspect of depression during their pregnancies. C. Income status plays a significant role in the presentation of perinatal mental health problems. D. A higher incidence of postpartum depression (PPD) is found in about 50% of the population. E. Between 30 and 50 billion dollars accounts for productivity and direct medical costs related to depression in women

B, C, E

A nurse is reviewing metabolic functions occuring during the postpartum period. Which of the following changes would the nurse identify as being consistent with that timeframe? (select all that apply) A: Mildly increased T3 and T4 levels for the first several weeks postpartum B: Increased BMR in the immediate postpartum period C: Secretion of insulinase D: Decrease in estrogen and cortisol levels E: Moderate hyperglycemia

BCD BMR remains elevated for the first 2 weeks after birth and then returns to prepregnancy levels. Insulinase enzyme reverses the diabetogenic effects of pregnancy, leading to decreased glucose levels in the postpartum period. Decreases in hormones such as estrogen and cortisol are seen during the postpartum period.Blood sugar levels typically decrease in the postpartum period as a result of the reversal of diabetogenic effects of pregnancy. Thyroid hormones gradually decrease to prepregnant levels in the 4 weeks following delivery.

A client has just delivered a healthy newborn. Which action should the nurse peform based on evidence-based care practice in the immediate newborn period? A: After drying, the infant should be given to the mother wrapped in a receiving blanket B: The healthy newborn should be taken to the nursery for a complete assessment C: Skin-to-skin contact of mother and baby should be encouraged D: The father or support person should be encouraged to hold the infant while awaiting delivery of the placenta

C

A nurse is advising a pregnant patient who has a substance abuse problem about a contingency management program. Which statement identifies an aspect of this type of program? A. Pregnant woman are confined to an inpatient treatment method during their pregnancy B. Pregnant woman are given biofeedback modalities as stimulus responses to control their addiction C. Pregnant woman are given motivational incentives as a primary approach to stop their drug abuse problem D. Pregnant woman must follow a strict medication nutritional program during the course of pregnancy

C

A nurse is discussing with an obese client potential long-term consequences of infant feeding practices. Which method should the nurse identify to the client as having a decreased risk for the development of childhood obesity for the infant? A. Lower-calorie infant formula B. An on-demand feeding schedule C. Breastfeeding D. Smaller, more frequent feedings

C

A nurse is examining a newborn male, who is estimated to be 39 weeks of gestation. Which physical finding should the nurse anticipate to be present? A. Abundant lanugo over his entire body B. Ability to move his elbow past his sternum C. Testes descended into the scrotum D. Extended posture when at rest

C

A nurse is reviewing the concept of breastfeeding. Which statement should the nurse identify as being inaccurate as it relates to the effect of breastfeeding on the family or society at large? A. Breastfeeding benefits the environment B. Breastfeeding requires fewer supplies and less cumbersome equipment C. Breastfeeding costs employers in terms of time lost from work D. Breastfeeding saves families money

C

A nurse is talking to parents about the adjustment of a new baby to the family unit. Which parent action should the nurse identify as facilitating the adjustement of other children to the new baby? A: Emphasizing activities that keep the new baby and other children together B: Having the mother carry the new baby into the home so she can show the other children the baby C: Having children at home choose or make a gift to give the new baby on his or her arrival home D: Reducing stress on the other children by limiting their involvement and care of the new baby

C

A nurse is working with a postpartum client about resumption of menstrual activity following childbirth. Which of the following statements indicate that the client has a correct understanding? A: "My first menstrual cycle will be heavier than normal and then will be light for several months after." B: "My first menstrual cycle will be lighter than normal and then will get heavier every month thereafter." C: "My first menstrual cycle will be heavier than normal and will return to my pre-pregnant volume within three or four cycles." D: "I will not have a menstrual cycle for 6 months after childbirth."

C

A nurse teaches a pregnant woman about the characteristics of true labor contractions. Which of the following statements indicates that the client correctly undersands the nurse's instruction? A: "True labor contractions will cause discomfort over the top of my uterus." B: "True labor contractions will subside when I walk around." C: "True labor contractions will continue and get stronger even if I relax and take a shower." D: "True labor contractions will remain irregular but become stronger."

C

A woman who is 39 weeks pregnant expresses fear about her impending labor and how she will manage. What is the best nursing response? A: "You may have an epidural. You won't feel anything." B: "Don't worry about it. You'll do fine." C: "It's normal to be anxious about labor. Let's discuss what makes you afraid." D: "Labor is scary to think about, but the actual experience isn't"

C

The nurse is assessing a newborn and discovers unequal movement or uneven gluteal skinfolds during the Ortolani maneuver. What should be the priority action taken by the nurse? A. Informs the parents and physician that molding has not taken place B. Tells the parents that one leg may be longer than the other, but they will equal out by the time the infant is walking C. Alerts the physician that the infant has a dislocated hip D. Suggests that if the condition does not change, surgery to correct vision problems might be needed

C

The nurse is using intermittent auscultation (IA) to assess uterine activity. Which statement should the nurse identify as correct? A: The resting tone between contractions is described as either placid or turbulent B: Contraction intensity is given a judgment number of 1 to 7 by the nurse and client together C: The examiner's hand should be placed over the fundus before, during, and after contractions D: The frequency and duration of contractions are measured in seconds for consistency

C

Where do most deliveries for pregnant women who have mental health issues take place? A. Mental health hospital setting B. Midwife assisted births C. Community hospital settings D. Psychiatric hospitals on locked units

C

Which priority action implemented by the nurse would be most beneficial in helping a couple deal with fetal loss following the delivery of a stillborn? A. Provide a quiet environment for the couple for several hours restricting any visitors or family members. B. Allow all family members to come in immediately after the delivery to console the couple. C. Allow the parents to hold and view the baby following delivery if they so request. D. Take a photograph of the stillborn prior to the client's discharge to use as a keepsake.

C Bonding with the stillborn by holding and viewing after delivery is well documented by research to provide a source of comfort and closure. Although it will be important for family members to comfort the couple, it is more important for the family unit to be alone to adapt to the delivery. Providing a quiet environment is important but it not the priority action to be taken at this time. Taking a photograph is important as a keepsake but it is typically taken before the stillborn leaves the hospital.

Which priority intervention would be needed if the nurse suspected that an infant was septic? A. Administration of oxygen B. Electronic monitoring of vital signs C. Intravenous access D. Recorded intake and output

C Establishing intravenous access for the administration of antibiotics would be a priority intervention. The other actions described might be required but are not the priority intervention.

An infant is to receive gastrostomy feedings. Which intervention should the nurse institute to prevent bloating, gastrointestinal reflux into the esophagus, vomiting, and respiratory compromise? a. Rapid bolusing of the entire amount in 15 minutes b. Warmcloths to the abdomen for the first 10 minutes c. Slow, small, warm bolus feedings over 30 minutes d. Cold, medium bolus feedings over 20 minutes

C Feedings by gravity are slowly accomplished over 20-to 30-minute periods to prevent adverse reactions. Rapid bolusing would most likely lead to the adverse reactions listed. Temperature stability in the newborn is critical. Applying warm cloths to the abdomen would not be appropriate because the environment is not thermoregulated. In addition, abdominal warming is not indicated with feedings of any kind. Small feedings at room temperature are recommended to prevent adverse reactions.

With regard to an eventual discharge of the high-risk newborn or the transfer of the newborn to a different facility, which information is essential to provide to the parents? a. Infants stay in the NICU until they are ready to go home b. Once discharged to go home, the high-risk infant should be treated like any healthy term newborn c. Parents of high-risk infants need special support and detailed contact information d. If a high-risk infant and mother need to be transferred to a specialized regional center, then waiting until after the birth and until the infant is stabilized is best.

C High-risk infants can cause profound parental stress and emotional turmoil. Parents need support, special teaching, and quick access to variousresources available to help them care for their baby. Parents and their high-risk infant should get to spend a night or two in a predischarge room, where care for the infant is provided away from the NICU. Simply because high-risk infants are eventually discharged does not mean they are normal, healthy babies. Follow-up by specialized practitioners is essential. Ideally, the mother and baby are transported with the fetus in utero; this reduces neonatal morbidity and mortality.

Preterm infants are more likely to become septic because: A. IgG and IgA levels are adequate at birth B. Immune function is suppressed because of increased IgG levels C. IgG level is directly proportional to gestational age D. Serum complement levels are adequate

C IgG levels are directly proportional to gestational age, being decreased in preterm infants, and reflect immune function. Levels of IgG and IgA are not adequate at birth and require time to become optimal. Serum complement levels are decreased at birth in preterm infants

A nurse practicing in the perinatal setting should promote kangaroo care regardless of an infant's gestational age. Which statement regarding this intervention is most appropriate? a. Kangaroo care was adopted from classical British nursing traditions b. This intervention helps infants with motor and CNS impairments c. Kangaroo care helps infants interact directly with their parents and enhances their temperature regulation d. This intervention gets infants ready for breastfeeding.

C Kangaroo care is skin-to-skin holding in which the infant, dressed only in a diaper, is placed directly on the parent's bare chest and then covered. The procedure helps infants interact with their parents and regulates their temperature, among other developmental benefits. Kangaroo care was established in Bogota, Colombia, assists the infant in maintaining an organized state, and decreases pain perception during heelsticks. Even premature infants who are unable to suckle benefit from kangaroo care. This practice fosters increased vigor and an enhanced breastfeeding experience as the infant matures.

Which postpartum infection is most often contracted by first-time mothers who are breastfeeding? A. Endometritis B. Wound infections C. Mastitis D. UTIs

C Mastitis is infection in a breast, usually confined to a milk duct. Most women who get it are first-timers who are breastfeeding. Endometritis is the most common postpartum infection. Its incidence is higher after a cesarean birth, not in first-time mothers. Wound infections are also a common postpartum complication. Sites of infection include both a cesarean incision and the episiotomy or repaired laceration. The gravidity of the mother and her feeding choice are not factors in the development of a wound infection. UTIs occur in 2% to 4% of all postpartum women. Risk factors include catheterizations, frequent vaginal exams, and epidural anesthesia.

A nurse is working with bereaved parents. What is the most appropriate statement that the nurse can make? A. "You're young and can have other children." B. "I understand how you must feel." C. "I'm sorry." D. "You have an angel in heaven."

C One of the most important goals of the nurse is to validate the experience and feelings of the parents by encouraging them to tell their stories and listening with care. At the very least, the nurse should acknowledge the loss with a simple but sincere comment, such as, "I'm sorry." The initial impulse may be to reduce one's sense of helplessness and to say or do something that you think will reduce their pain. Although such a response may seem supportive at the time, it can stifle the further expression of emotion. The nurse should resist the temptation to give advice or to use clichés when offering support to the bereaved. The statement in C is not a therapeutic response for the nurse to make.

A nurse is reviewing the concept of complicated bereavement. Which statement would the nurse identify as being accurate? A. Occurs when, in multiple births, one child dies and the other or others live B. Is felt by the family of adolescent mothers who lose their babies C. Is an extremely intense grief reaction that persists for a long time D. Is a state in which the parents are ambivalent, as with an abortion

C Parents showing signs of complicated grief should be referred for counseling. Multiple births in which not all the babies survive creates a complicated parenting situation, abortion can generate complicated emotional responses, and families of lost adolescent pregnancies may have to deal with complicated issues, but these situations are not complicated bereavement.

Some pregnant clients may report changes in their voice and impaired hearing. What should the nurse explain to the client concerning these findings? a. Voice changes are caused by decreased estrogen levels b. Displacement of the diaphragm results in thoracic breathing c. These changes are the results of congestion and swelling of the upper respiratory tract d. Increased blood volume causes changes in the voice

c

Which indicator would lead the nurse to suspect that a postpartum patient experiencing hemorrhagic shock is getting worse? A. Restoration of blood pressure levels to normal range B. Capillary refill brisk C. Patient complaint of headache and increased reaction time to questioning D. Patient statement that she sees "stars"

C Patient complaint of a headache accompanied by an increased reaction (response) time indicates that cerebral hypoxia is getting worse. Return of blood pressure to normal range would indicate resolving symptoms. Brisk capillary refill is a normal finding. The patient may see "stars" early on in decreased blood flow states.

Prostaglandin gel has been ordered for a pregnant woman at 43 weeks of gestation. What is the primary purpose of prostaglandin administration? a. To enhance uteroplacental perfusion in an aging placenta b. To increase amniotic fluid volume c. To ripen the cervix in preparation for labor induction d. To stimulate the amniotic membranes to rupture

C Preparations of prostaglandin E1and E2are effective when used before labor induction to ripen (i.e., soften and thin) the cervix. Uteroplacental perfusion is not altered by the use of prostaglandins. The insertion of prostaglandin gel has no effect on the level of amniotic fluid. In some cases, women will spontaneously begin laboring after the administration of prostaglandins, thereby eliminating the need for oxytocin. It is not common for a woman's membranes to rupture as a result of prostaglandin use.

For clinical purposes, the most accurate definition of preterm and postterm infants is defined as what? a. Preterm: Before 34 weeks of gestation if the infant is appropriate for gestational age (AGA); before 37 weeks if the infant is small for gestational age (SGA) b. Postterm: After 40 weeks of gestation if the infant is large for gestational age (LGA); beyond 42 weeks if the infant is AGA c. Preterm: Before 37 weeks of gestation and postterm beyond 42 weeks of gestation; no matter the size for gestational age at birth d. Preterm: Before 38 to 40 weeks of gestation if the infant is SGA; postterm, beyond 40 to 42 weeks gestation if the infant is LGA

C Preterm and postterm are strictly measures of time—before 37 weeks and beyond 42 weeks, respectively—regardless of the size for gestational age.

In planning for home care of a woman with preterm labor, which concern should the nurse need to address? a. Nursing assessments are different from those performed in the hospital setting b. Restricted activity and medications are necessary to prevent a recurrence of preterm labor c. Prolonged bed rest may cause negative physiologic effects d. Home health care providers are necessary

C Prolonged bed rest may cause adverse effects such as weight loss, loss of appetite, muscle wasting, weakness, bone demineralization, decreased cardiac output, risk for thrombophlebitis, alteration in bowel functions, sleep disturbance, and prolonged postpartum recovery. Nursing assessments differ somewhat from those performed in the acute care setting, but this concern does not need to be addressed. Restricted activity and medications may prevent preterm labor but not in all women. In addition, the plan of care is individualized to meet the needs of each client. Many women receive home health nurse visits, but care is individualized for each woman.

A nurse is preparing to educate a group of postpartum clients. Which description of postpartum restoration or healing times should the nurse identify as being accurate? A: Most episiotomies heal within a week B: Hemorrhoids usually decrease in size within 2 weeks of childbirth C: Rugae reappear within 3 to 4 weeks D: The cervix shortens, becomes firm, and returns to form within a month postpartum

C Rugae are never again as prominent as in a nulliparous woman but begin to return after 3-4 weeks

In caring for a mother who has abused (or is abusing) alcohol and for her infant, nurses should be aware that: A. The pattern of growth restriction of the fetus begun in prenatal life is halted after birth, and normal growth takes over B. Two thirds of newborns with fetal alcohol syndrome (FAS) are boys C. Alcohol-related neurodevelopmental disorders (ARNDs) not sufficient to meet FAS criteria (learning disabilities, speech and language problems) are often not detected until the child goes to school D. Both the distinctive facial features of the FAS infant and the diminished mental capacities tend toward normal over time

C Some learning problems do not become evident until the child is in school. The pattern of growth restriction persists after birth. Two thirds of newborns with FAS are girls. Although the distinctive facial features of the FAS infant tend to become less evident with growth, the mental capacities never become normal

After giving birth to a stillborn infant, the woman turns to the nurse and says, "I just finished painting the baby's room. Do you think that caused my baby to die?" What is the nurse's best response? A. Silence. B. "That's not likely. Paint is associated with elevated pediatric lead levels." C. "I can understand your need to find an answer to what caused this. What else are you thinking about?" D. "That's an old wives' tale; lots of women are around paint during pregnancy, and this doesn't happen to them."

C The statement in D is very appropriate for the nurse. It demonstrates caring and compassion and allows the mother to vent her thoughts and feelings, which is therapeutic in the process of grieving. The nurse should resist the temptation to give advice or to use clichés in offering support to the bereaved. Trying to give bereaved parents answers when no clear answers exist or trying to squelch their guilt feelings does not help the process of grief. Additionally the response in B probably would increase the mother's feelings of guilt. One of the most important goals of the nurse is to validate the experience and feelings of the parents by encouraging them to tell their stories, and listening with care, which silence would not do.

An infant is being discharged from the NICU after 70 days of hospitalization. The infant was born at 30 weeks of gestation with several conditions associated with prematurity, including RDS, mild bronchopulmonary dysplasia (BPD), and retinopathy of prematurity (ROP), requiring surgical treatment. During discharge teaching, the infant's mother asks the nurse if her baby will meet developmental milestones on time, as did her son who was born at term. What is the nurse's most appropriate response? a. "Your baby will develop exactly like your first child." b. "Your baby does not appear to have any problems at this time." c. "Your baby will need to be corrected for prematurity." d. "Your baby will need to be followed very closely."

C The age of a preterm newborn is corrected by adding the gestational age and the postnatal age. The infant's responses are accordingly evaluated against the norm expected for the corrected age of the infant. The baby is currently 40 weeks of postconceptional age and can be expected to be doing what a 40-week-old infant would be doing. Although predicting with complete accuracy the growth and development potential of each preterm infant is impossible, certain measurable factors predict normal growth and development. The preterm infant experiences catch-up body growth during the first 2 to 3 years of life. Development needs to be evaluated over time. The growth and developmental milestones are corrected for gestational age until the child is approximately 2.5 years old.

A primigravida at 40 weeks of gestation is having uterine contractions every to 2 minutes and states that they are very painful. Her cervix is dilated 2 cm and has not changed in 3 hours. The woman is crying and wants an epidural. What is the likelystatus of this woman's labor? a. She is exhibiting hypotonic uterine dysfunction b. She is experiencing a normal latent stage c. She is exhibiting hypertonic uterine dysfunction d. She is experiencing precipitous labor

C The contraction pattern observed in this woman signifies hypertonic uterine activity. Typically, uterine activity in this phase occurs at 4-to 5-minute intervals lasting 30 to 45 seconds. Women who experience hypertonic uterine dysfunction, or primary dysfunctional labor, are often anxious first-time mothers who are having painful and frequent contractions that are ineffective at causing cervical dilation or effacement to progress. With hypotonic uterine dysfunction, the woman initially makes normal progress into the active stage of labor; then the contractions become weak and inefficient or stop altogether. Precipitous labor is one that lasts less than 3 hours from the onset of contractions until time of birth.

An infant weighing 4.1 kg was born 2 hours ago at 37 weeks of gestation. The infant appears chubby with a flushed complexion and is very tremulous. The tremors are most likely the result of: A. Birth injury B. Hypocalcemia C. Hypoglycemia D. Seizures

C The description is indicative of a macrocosmic infant. Hypoglycemia is common in the infant with macrosomia. The tremors are jitteriness that is associated with hypoglycemia. Other signs of hypoglycemia are apnea, tachypnea, and cyanosis

Which factor increases the risk of complications for infants of diabetic mothers? A. Glycemic control B. Hemoglobin A1c level of 7 C. Duration of maternal disease D. Hemoglobin A1c level of 7 prior to pregnancy

C The duration and severity of maternal disease are significant factors in increasing the risk for complications in infants of diabetic mothers. Glycemic control would be a positive factor indicating that blood glucose levels were maintained within normal range. A hemoglobin A1c level of 7 is within normal range.

By understanding the four mechanisms of heat transfer (convection, conduction, radiation, and evaporation), the nurse can create an environment for the infant that prevents temperature instability. Which significant symptoms will the infant display when experiencing cold stress? a. Decreased respiratory rate b. Bradycardia, followed by an increased heart rate c.I rritability with central cyanosis d. Increased physical activity

C The infant has minimal-to-no fat stores. During times of cold stress, the skin becomes mottled and acrocyanosis develops, progressing to cyanosis. The infant will likely demonstrate irritability. Even if the infant is being cared for on a radiant warmer or in an isolette, the nurse's role is to observe the infant frequently to prevent heat loss and to respond quickly if signs and symptoms of cold stress occur. The respiratory rate increases, followed by periods of apnea. The infant initially tries to conserve heat and burns more calories, after which the metabolic system goes into overdrive. In the preterm infant who is experiencing heat loss, the heart rate initially increases, followed by periods of bradycardia. In theterm infant, increased physical activity is the natural response to heat loss. However, in a term infant who is experiencing respiratory distress or in a preterm infant, physical activity is decreased.

An infant at 36 weeks of gestation has increasing respirations (80 to 100 breaths per minute with significant substernal retractions). The infant is given oxygen by continuous nasal positive airway pressure (CPAP). What level of partial pressure of arterial oxygen (PaO2) indicates hypoxia? a. 67 mm Hg b. 89 mm Hg c. 45 mm Hg d. 73 mm Hg

C The laboratory value of PaO2 of 45 mm Hg is below the range for a normal neonate and indicates hypoxia in this infant. The normal range for PaO2 is 60 to 80 mm Hg; therefore, PaO2 levels of 67 and 73 mm Hg fall within the normal range, and a PaO2 of 89 mm Hg is higher than the normal range.

Nurses can help their clients by keeping them informed about the distinctive stages of labor. What description should a nurse indicate as being accurate with regard to the phases of the first stage of labor? A: Lull: no contractions; dilation stable; duration of 20 to 60 minutes B: Latent: mild, regular contractions; no dilation; bloody show; duration of 2 to 4 hours C: Active: moderate, regular contractions; 4 to 7 cm dilation; duration of 3 to 6 hours D: Transition: very strong but irregular contractions; 8 to 10 cm dilation; duration of 1 to 2 hours

C The latent phase is characterized by mild to moderate, irregular contractions; dilation up to 3 cm; brownish to pale pink mucus; and a duration of 6 to 8 hours. No official lull phase exists in the first stage. The transition phase is characterized by strong to very strong, regular contractions; 8 to 10 cm dilation; and a duration of 20 to 40 minutes.

A nurse is reviewing the stages of labor. Which statement should the nurse identify as correct? A: Latent phase: feels sleepy, fetal station is 2+ to 4+, duration is 30 to 45 minutes B: Active phase: overwhelmingly strong contractions, Ferguson reflux activated, duration is 5 to 15 minutes C: Descent phase: significant increase in contractions, Ferguson reflux activated, average duration varies D: Transitional phase: woman "laboring down," fetal station is 0, duration is 15 minutes

C The latent phase is the lull, or "laboring down" period, at the beginning of the second stage. It lasts 10 to 30 minutes on average. The second stage of labor has no active phase. The transition phase is the final phase in the second stage of labor; contractions are strong and painful.

Which factor would contribute to depletion of weight and metabolic stores in the high risk newborn? A. Frequent breast feedings B. Core temperature within normal range C. Phototherapy D. Bathing

C The use of phototherapy could lead to insensible heat loss and as a result lead to decreased weight and metabolic stores in the high risk newborn. Frequent breastfeedings and bathing would not have these effects. Maintaining a core temperature would help maintain weight and metabolic stores in the high risk newborn.

Which statement related to cephalopelvic disproportion (CPD) is the least accurate? a. CPD can be related to either fetal size or fetal position b. The fetus cannot be born vaginally c. CPD can be accurately predicted d. Causes of CPD may have maternal or fetal origins

C Unfortunately, accurately predicting CPD is not possible. Although CPD is often related to excessive fetal size (macrosomia), malposition of the fetal presenting part is the problem in many cases,not true CPD. When CPD is present, the fetus cannot fit through the maternal pelvis to be born vaginally. CPD may be related to either fetal origins such as macrosomia or malposition or maternal origins such as a too small or malformed pelvis.

Nurses are getting ready for bedside reporting at change of shift. What benefit do the nurses identify for this type of change of shift report? A: Information is transparent so that the nurses and clients are aware of all pertinent data and delivery of care aspects. B: Clients can ask questions of the nurses during change of shift report so that they can better direct the delivery of their health care. C: Nurses are able to visualize their client's directly at the time of report leading to better client satisfaction. D: There is no need for additional information to be exchanged as the client is right there to answer questions and voice concerns.

C Using a bedside report technique helps the nurse directly visualize the client in question so as to improve his/her understanding of each client's clinical situation.

A woman is having her first child. She has been in labor for 15 hours. A vaginal examination performed 2 hours earlier revealed the cervix to be dilated to 5 cm and 100% effaced, and the presenting part of the fetus was at station 0; however, another vaginal examination performed 5 minutes ago indicated no changes. What abnormal labor pattern is associated with this description? a. Prolonged latent phase b. Protracted active phase c. Secondary arrest d. Protracted descent

C With a secondary arrest of the active phase, the progress of labor has stopped. This client has not had any anticipated cervical change, indicating an arrest of labor. In the nulliparous woman, a prolonged latent phase typically lasts longer than 20 hours. A protracted active phase, the first or second stage of labor, is prolonged (slow dilation). With a protracted descent, the fetus fails to descend at an anticipated rate during the deceleration phase and second stage of labor.

The measurement of lecithin in relation to sphingomyelin (lecithin/sphingomyelin [L/S] ratio) is used to determine fetal lung maturity. Which ratio reflects fetal maturity of the lungs? a. 1.4:1 b. 1.8:1 c. 2:1 d. 1:1

c

The nurse is assessing a full term, quiet, and alert newborn. What is the average expected apical pulse range (in beats per minute)? a. 80 to 100 b. 100 to 120 c. 110 to 160 d. 150 to 180

c

Which factors predispose an infant to birth injuries? select all that apply A. Multip between the ages of 25 and 30 B. Vertex presentation C. Application of an internal fetal scalp electrode D. Vacuum-assisted birth

C and D The use of an internal fetal scalp electrode could result in a scalp injury, which would be evident upon birth. The use of vacuum extraction could lead to a birth injury. Very young age (less than 16) and older age (more than 35) in a primipara are more likely to predispose an infant to birth injuries. Vertex presentation is a normal finding and as such would not typically lead to a birth injury.

Which of the following processes or findings increase the risk of preterm infants in which hematologic problems are developing? Select all that apply. A. Decrease in size of red blood cells B. Decreased capillary fragility C. Prolonged PT time D. Decreased red blood cell survival time E. Decrease in erythropoiesis

C, D, E Prolonged PT reflects an increased tendency to bleed in preterm infants. Decrease in red blood cell survival time is seen in such infants. So is decreased functional ability of erythropoietin, which limits red blood cell synthesis. One sees an increase in the size of red blood cells in preterm infants, which affects their survival time. Increased capillary fragility also occurs in preterm infants.

In follow-up appointments or visits with parents and their new baby, it may be useful if the nurse can identify parental behaviors that can either facilitate or inhibit attachment. Which of the following should the nurse identify as a facilitating behavior? A: The parents have difficulty naming the infant B: The parents make no effort to interpret the actions or needs of the infant C: The parents do not move from fingertip touch to palmar contact and holding D: The parents hover around the infant, directing attention to and pointing at the infant

D

A nurse is reviewing spinal and epidural (block) anesthesia use during labor. Which statement should the nurse identify as being accurate? A: Epidural blocks allow the woman to move freely B: This type of anesthesia is commonly used for cesarean births but is not suitable for vaginal births C: Spinal and epidural blocks are never used together D: A high incidence of postbirth headache is seen with spinal blocks

D

A nurse is taking care of a client in labor who is experiencing back pain. What action should the nurse implement? A: Lie on her back for a while with her knees bent B: Take some deep, cleansing breaths C: Do less walking around D: Lean over a birth ball with her knees on the floor

D

A nurse is working with a Hispanic client. Which statement accurately reflects an after childbirth ritual? A: No restrictions are placed on the mother during this ritual period B: The ritual is limited to preparing the woman to become a good mother C: Breastfeeding is started immediately after birth D: Naming of the child can not be done until the 40 day period has been met

D

A nurse is working with a client in the second-stage of labor. Which position would the nurse suggest if the pelvic outlet needs to be increased? A: Semirecument B: Side-lying C: Sitting D: Squatting

D

A nurse is working with a client who is in labor and providing information relative to breathing techniques. Which option should the nurse include in the plan of care? A: Controlled breathing techniques are most difficult near the end of the second stage of labor B: The patterned-paced breathing technique can help prevent hyperventilation C: By the time labor has begun, it is too late for instruction in breathing and relaxation D: Breathing techniques in the first stage of labor are designed to increase the size of the abdominal cavity to reduce friction

D

A nurse is working with a postpartm client who is experiencing after birth pains. Which statement should the nurse identify as being accurate with regard to afterbirth pains? A: Alleviated somewhat when the mother breastfeeds B: They are caused by mild, continual contractions for the duration of the postpartum period C: More common in first-time mothers D: More noticeable in births in which the uterus was over-distended

D

The nurse helps a breastfeeding mother change the diaper of her 16-hour-old newborn after the first bowel movement. The mother expresses concern because the large amount of thick, sticky stool is very dark green, almost black. She asks the nurse whether something is wrong. Which of the following would be the best response offered by the nurse? A. Telling the mother not to worry because all breastfed babies have this type of stool. B. Asking the mother what she ate for her last meal. C. Suggesting to the mother that she ask her pediatrician to explain normal newborn stooling patterns to her. D. Explaining to the mother that this stool is called meconium and is expected for the first few bowel movements of all newborns

D

The nurse is monitoring a client during labor. Which observation if noted by the nurse would indicate fetal well-being? A: Accelerations in the FHR B: An FHR greater than 110 beats/min C: Maternal pain is being controlled by using breathing techniques D: The response of the fetal heart rate (FHR) to uterine contractions (UCs

D

The nurse observes accelerations on the fetal monitor in a laboring client. How should the nurse interpret this finding? A: as caused by umbilical cord compression B: warrants close observation C: as caused by uteroplacental insufficiency D: as reassuring

D

The nurse taught new parents the guidelines to follow regarding the bottle feeding of their newborn who will be using formula from a can of concentrate. Which action if observed by the nurse would indicate that the parents correctly understand the nurse's instruction? A. Warm formula in a microwave oven for a couple of minutes prior to feeding B. Add some honey to sweeten the formula and make it more appealing to a fussy newborn C. Adjust the amount of water added according to weight gain pattern of the newborn D. Wash the top of can and can opener with soap and water before opening the can

D

To provide adequate postpartum care, the nurse should be aware that postpartum depression (PPD) with psychotic features: A. Is more likely to occur in women with more than two children. B. Is rarely delusional and is usually about someone trying to harm her (the mother). C. Although serious, is not likely to need psychiatric hospitalization. D. Is typified by auditory or visual hallucinations

D

Which of these medications would be classified as a Category X substance that is not to be used during pregnancy? A. Lorazepam (Ativan) B. Alprazolam (Xanax) C. Chlordiazepoxide (Librium) D. Temazepam (Restoril)

D

While evaluating the reflexes of a male newborn, the nurse notes that with a loud noise, the newborn symmetrically abducts and extends his arms, his fingers fan out and form a "C" with the thumb and forefinger, and he has a slight tremor. The nurse documents this finding as a positive: A. Glabellar (Myerson) reflex response B. Babinski reflex response C. Tonic neck reflex response D. Moro reflex response

D

A woman is in her seventh month of pregnancy. She reports episodes of nasal congestion and occasional epistaxis. Which statement bestdescribes why this may be happening to this client? a. This respiratory change is normal in pregnancy and caused by an elevated level of estrogen b. This cardiovascular change is abnormal, and the nosebleeds are an ominous sign c. The woman is a victim of domestic violence and is being hit in the face by her partner d. The woman has been likely intranasally using cocaine for several months

a

Which condition might premature infants who exhibit 5 to 10 seconds of respiratory pauses, followed by 10 to 15 seconds of compensatory rapid respiration, be experiencing? a. Suffering from sleep or wakeful apnea b. Experiencing severe swings in blood pressure c. Trying to maintain a neutral thermal environment d. Breathing in a respiratory pattern common to premature infants

D Breathing in a respiratory pattern is called periodic breathing and is common to premature infants. This pattern may still require nursing intervention of oxygen and/or ventilation. Apnea is the cessation of respirations for 20 seconds or longer and should not be confused with periodic breathing.

A woman at 26 weeks of gestation is being assessed to determine whether she is experiencing preterm labor. Which finding indicates that preterm labor is occurring? a. Estriol is not found in maternal saliva b. Irregular, mild uterine contractions are occurring every 12 to 15 minutes c. Fetal fibronectin is present in vaginal secretions d. The cervix is effacing and dilated to 2 cm.

D Cervical changes such as shortened endocervical length, effacement, and dilation are predictors of imminent preterm labor. Changes in the cervix accompanied by regular contractions indicate labor at any gestation. Estriol is a form of estrogen produced by the fetus that is present in plasma at 9 weeks of gestation. Levels of salivary estriol have been shown to increase before preterm birth. Irregular, mild contractions that do not cause cervical change are not considered a threat. The presence of fetal fibronectin in vaginal secretions between 24 and 36 weeks of gestation could predict preterm labor, but it has only a 20% to 40% positive predictive value. Of more importance are other physiologic clues of preterm labor such as cervical changes.

Despite popular belief, there is a rare type of hemophilia that affects women of childbearing age. Von Willebrand disease is the most common of the hereditary bleeding disorders and can affect males and females alike. It results from a factor VIII deficiency and platelet dysfunction. Although factor VIII levels rise naturally during pregnancy, there is an increased risk for postpartum hemorrhage from birth until 4 weeks postpartum as levels of von Willebrand factor (vWf) and factor VIII fall. The treatment that should be considered first for the client with von Willebrand disease who experiences a postpartum hemorrhage is: A. Cryoprecipitate B. Factor VIII and vWf C. Hemabate D. Desmopressin

D Desmopressin is the primary treatment of choice. This hormone, which can be administered orally, nasally, and intravenously, promotes the release of factor VIII and vWf from storage. Treatment with cryoprecipitate or with plasma products such as factor VIII and vWf is acceptable, but because of the associated risk of possible viruses from donor blood products, other modalities are considered safer. Although the administration of the synthetic prostaglandin in Hemabate is known to promote contraction of the uterus during postpartum hemorrhage, it is not effective for the client who presents with a bleeding disorder.

When providing an infant with a gavage feeding, which infant assessment should be documented each time? a. Abdominal circumference after the feeding b. Heart rate and respirations before feeding c. Suck and swallow coordination d. Response to the feeding

D Documentation of a gavage feeding should include the size of the feeding tube, the amount and quality of the residual from the previous feeding, the type and quantity of the fluid instilled, and the infant's response to the procedure. Abdominal circumference is not measured after a gavage feeding. Although vital signs may be obtained before feeding, the infant's response to the feeding is more important. Similarly, some older infants may belearning to suck; the most important factor to document would still be the infant's response to the feeding, including the attempts to suck.

When providing an infant with a gavage feeding, what should the nurse document each time? A. The infant's abdominal circumference after the feeding B. The infant's heart rate and respirations C. The infant's suck and swallow coordination D. The infant's response to the feeding

D Documentation of a gavage feeding should include the size of the feeding tube, the amount and quality of the residual from the previous feeding, the type and quantity of the fluid instilled, and the infant's response to the procedure. Some older infants may be learning to suck, but the important factor to document is the infant's response to the feeding (including attempts to suck). Abdominal circumference is not measured after a gavage feeding. Vital signs may be obtained prior to feeding, but the infant's response is more important.

The nurse is performing an assessment on a client who thinks she may be experiencing preterm labor. Which information is the most important for the nurse to understand and share with the client? a. Because all women must be considered at risk for preterm labor and prediction is so variable, teaching pregnant women the symptoms of preterm labor probably causes more harm through false alarms b. Braxton Hicks contractions often signal the onset of preterm labor c. Because preterm labor is likely to be the start of an extended labor, a woman with symptoms can wait several hours before contacting the primary caregiver d. Diagnosis of preterm labor is based on gestational age, uterine activity, and progressive cervical change.

D Gestational age of 20 to 37 weeks, uterine contractions, and a cervix that is 80% effaced or dilated 2 cm indicates preterm labor. It is essential that nurses teach women how to detect the early symptoms of preterm labor. Braxton Hicks contractions resemble preterm labor contractions, but they are not true labor. Waiting too long to see a health care provider could result in essential medications failing to be administered. Preterm labor is not necessarily long-term labor.

Which statement related to the induction of labor is most accurate? a. Can be achieved by external and internal version techniques b. Is also known as atrial of labor (TOL) c. Is almost always performed for medical reasons d. Is rated for viability by a Bishop score

D Induction of labor is likely to be more successful with a Bishop score of 9 or higher for first-time mothers or 5 or higher for veterans. Version is the turning of the fetus to a better position by a physician for an easier or safer birth. A TOL is the observance of a woman and her fetus for several hours of active labor to assess the safety of vaginal birth. Two thirds of cases of induced labor are elective and not done for medical reasons.

When would an internal version be indicated to manipulate the fetus into a vertex position? a. Fetus from a breech to a cephalic presentation before labor begins b. Fetus from a transverse lie to a longitudinal lie before a cesarean birth c. Second twin from an oblique lie to a transverse lie before labor begins d. Second twin from a transverse lie to a breech presentation during a vaginal birth

D Internal version is used only during a vaginal birth to manipulate the second twin into a presentation that allows it to be vaginally born. For internal version to occur, the cervix needs to be completely dilated.

A nurse providing care to preterm infants should understand that nasogastric and orogastric tubes are used to: A. Help maintain body temperature B. Provide oxygen and ventilation C. Replace surfactants D. Feed the infants

D Nasogastric and orogastric tubes are used in gavage feeding, providing breast milk or formula directly to an infant unable to nipple feed. To help maintain body temperature, preterm infants should be placed on warmers. Oxygen, continuous positive airway pressure (CPAP), and a ventilator are used for O2 and ventilation. Surfactants are not replaced by using nasogastric or orogastric tubes.

A pregnant woman's amniotic membrane has ruptured and a prolapsed cord is suspected. Which intervention is the nurse's highest priority? A. Cover the cord in a sterile towel saturated with warm normal saline. B. Prepare the woman for a cesarean birth. C. Start oxygen by face mask. D. Place the woman in the knee-chest position.

D The woman is assisted into a position (e.g., modified Sims position, Trendelenburg position, or knee-chest position) in which gravity keeps the pressure of the presenting part off the cord. Relieving pressure on the cord is the nursing priority. The nurse may also use her gloved hand or two fingers to lift the presenting part off the cord. If the cord is protruding from the vagina it may be covered with a sterile towel soaked in saline. The nurse should administer O2 by facial mask at 8 to 10 L/min until delivery is complete. If the cervix is fully dilated, the nurse should prepare for immediate vaginal delivery. Cesarean birth is indicated only if cervical dilation is not complete.

On day 3 of life, a newborn continues to require 100% oxygen by nasal cannula. The parents ask if they may hold their infant during his next gavage feeding. Considering that this newborn is physiologically stable, what response should the nurse provide? a. "Parents are not allowed to hold their infants who are dependent on oxygen." b. "You may only hold your baby's hand during the feeding." c. "Feedings cause more physiologic stress; therefore, the baby must be closely monitored. I don't think you should hold the baby." d. "You may hold your baby during the feeding."

D Physical contact with the infant is important to establish early bonding. The nurse as the support person and teacher is responsible for shaping the environment and making the caregiving responsive to the needs of both the parents and the infant. Allowing the parents to hold their baby is the most appropriate response by the nurse. Parental interaction by holding should be encouraged during gavage feedings; nasal cannula oxygen therapy allows for easy feedings and psychosocial interactions. The parent can swaddle the infant or provide kangaroo care while gavage feeding their infant. Both swaddling and kangaroo care during feedings provide positive interactions for the infant and help the infant associate feedings with positive interactions.

Which assessment is least likely to be associated with a breech presentation? a. Meconium-stained amniotic fluid b. Fetal heart tones heard at or above the maternal umbilicus c. Preterm labor and birth d. Postterm gestation

D Postterm gestation is not likely to occur with a breech presentation. The presence of meconium in a breech presentation may be a result of pressure on the fetal wall as it traverses the birth canal. Fetal heart tones heard at the level of the umbilical level of the mother are a typical finding in a breech presentation because the fetal back would be located in the upper abdominal area. Breech presentations often occur in preterm births.

When evaluating the preterm infant, the nurse understands that compared with the term infant, what information is important for the nurse to understand? a. Few blood vessels visible through the skin b. More subcutaneous fat c. Well-developed flexor muscles d. Greater surface area in proportion to weight

D Preterm infants have greater surface area in proportion to their weight. More subcutaneous fat and well-developed muscles are indications of a more mature infant.

The obstetric provider has informed the nurse that she will be performing an amniotomy on the client to induce labor. What is the nurse's highest priority intervention after the amniotomy is performed? a. Applying clean linens under the woman b. Taking the client's vital signs c. Performing a vaginal examination d. Assessing the fetal heart rate (FHR)

D The FHR is assessed before and immediately after the amniotomy to detect any changes that might indicate cord compression or prolapse. Providing comfort measures, such as clean linens, for the client is important but not the priority immediately after an amniotomy. The woman's temperature should be checked every 2 hours after the rupture of membranes but not the priority immediately after an amniotomy. The woman would have had a vaginal examination during the procedure. Unless cord prolapse is suspected, another vaginal examination is not warranted. Additionally, FHR assessment provides clinical cues to a prolapsed cord.

Because of the premature infant's decreased immune functioning, what nursing diagnosis should the nurse include in a plan of care for a premature infant? a. Delayed growth and development b. Ineffectivethermoregulation c. Ineffective infant feeding pattern d. Risk for infection

D The nurse needs to understand that decreased immune functioning increases the risk for infection. Growth and development, thermoregulation, and feeding may be affected, although only indirectly.

Premature infants who exhibit 5 to 10 seconds of respiratory pauses followed by 10 to 15 seconds of compensatory rapid respiration are: A. Suffering from sleep or wakeful apnea B. Experiencing severe swings in blood pressure C. Trying to maintain a neutral thermal environment D. Breathing in a respiratory pattern common to premature infants

D The pattern of 5 to 10 seconds of respiratory pauses followed by 10 to 15 seconds of rapid respiration is called periodic breathing, which is common to premature infants. It may require nursing intervention such as oxygen and/or ventilation. Apnea is a cessation of respirations for 20 seconds or longer. An infant who presents with fluctuation in systemic blood pressure may have experienced a central nervous system injury. An infant attempting to maintain body temperature is likely to present with hypoglycemia, shivering, and mottled color.

In evaluating the effectiveness of magnesium sulfate for the treatment of preterm labor, which finding alerts the nurse to possible side effects? a. Urine output of 160 ml in 4 hours b. DTRs 2+ and no clonus c. Respiratory rate (RR) of 16 breaths per minute d. Serum magnesium level of 10 mg/dl

D The therapeutic range for magnesium sulfate management is 4 to 7.5 mg/dl. A serum magnesium level of 10 mg/dl could lead to signs and symptoms of magnesium toxicity, including oliguria and respiratory distress. Urine output of 160 ml in 4 hours, DTRs of 2+, and a RR of 16 breaths per minute are all normal findings.

According to research, which risk factor for postpartum depression (PPD) is likely to have the greatest effect on the client postpartum? a. Prenatal depression b. Single-mother status c. Low socioeconomic status d. Unplanned or unwanted pregnancy

a

What are the two classifications of PPH?

Early/Primary/Acute and Late/Secondary Early PPH is also known as primary, or acute, PPH; late PPH is known as secondary PPH. Unfortunately, PPH can occur with little warning and is often recognized only after the mother has profound symptoms.

Despite warnings, prenatal exposure to alcohol continues to far exceed exposure to illicit drugs. Which condition is rarely associated with fetal alcohol syndrome (FAS)? a. Respiratory conditions b. Intellectual impairment c. Neural development disorder d. Alcohol-related birth defects (ARBDs)

a

A nurse suspects that the laboring client has ruptured membranes. Which test should the nurse perform? Leopold maneuvers AROM Urine analysis Fern test

Fern test

Which information regarding amniotic fluid is important for the nurse to understand? a. Amniotic fluid serves as a source of oral fluid and a repository for waste from the fetus b. Volume of the amniotic fluid remains approximately the same throughout the term of a healthy pregnancy c. The study of fetal cells in amniotic fluid yields little information d. A volume of more than 2 L of amniotic fluid is associated with fetal renal abnormalities

a

Parents have been asked by the neonatologist to provide breast milk for their newborn son, who was born prematurely at 32 weeks of gestation. The nursewho instructs them regarding pumping, storing, and transporting the milk needs to assess their knowledge of lactation. Which statement is a valid basis for the instructions? a. Premature infants more easily digest breast milk than formula b. A glass of wine just before pumping will help reduce stress and anxiety c. The mother should only pump as much milk as the infant can drink d. The mother should pump every 2 to 3 hours, including during the night

a

A woman in the active phase of the first stage of labor is using a shallow pattern of breathing, which is about twice the normal adult breathing rate. She starts to complain about feeling lightheaded and dizzy and states that her fingers are tingling. What action should the nurse implement? Tell the woman to slow the pace of her breathing Administer oxygen via a mask or nasal cannula Help her breathe into a paper bag Notify the woman's physician

Help her breathe into a paper bag

nursing interventions for epidural

IV fluids to counteract hypotension lay woman on L side monitor maternal VS and FHR q5min for first 15 mins after epidural placed

Which presumptive sign or symptom of pregnancy would a client experience who is approximately 10 weeks of gestation? a. Amenorrhea b. Positive pregnancy test c. Chadwick sign d. Hegar sign

a

As part of the postpartum assessment, the nurse examines the breasts of a primiparous breastfeeding woman 1 day postpartum. What should the nurse identify as an expected finding? Little if any change Leakage of milk at let-down Swollen, warm and tender on palpation A few blisters and a bruise on each areola

Little if any change Breasts are essentially unchanged for the first 24 hours after birth. Colostrum is present and may leak from the nipples. Leakage of milk occurs after the milk comes in 72 to 96 hours after birth. Engorgement occurs at day 3 or 4 postpartum. A few blisters and a bruise indicate problems with the breastfeeding techniques being used.

On examining a woman who gave birth 5 hours ago, the nurse finds that the woman has completely saturated a perineal pad within 15 minutes. What is the first action to be taken by the nurse? Call the woman's primary health care provider Begin an IV infusion of Ringer's lactate solution Assess the woman's vital signs Massage the woman's fundus

Massage the woman's fundus The nurse should assess the uterus for atony. Uterine tone must be established to prevent excessive blood loss. The nurse may begin an IV infusion to restore circulatory volume, but this would not be the first action. Blood pressure is not a reliable indicator of impending shock from impending hemorrhage; assessing vital signs should not be the nurse's first action. The physician should be notified after the nurse completes assessment of the woman.

The most serious complication of an infant heelstick is necrotizing osteochondritis resulting from lancet penetration of the bone. What approach should the nurse take when performing the test to prevent this complication? a. Lancet should penetrate at the outer aspect of the heel b. Lancet should penetrate the walking surface of the heel c. Lancet should penetrate the ball of the foot d. Lancet should penetrate the area just below the fifth toe.

a

The nurse is evaluating the fetal monitor tracing of a client, who is in active labor. Suddenly the fetal heart rate (FHR) drop from its baseline of 125 down to 80. The nurse repositions the mother, provides oxygen, increases IV fluid, and performs a vaginal exam. The cervix has not changed. Five minutes have passed, and the FHR remains in the 80s. What additional measures should the nurse take? Insert a Foley catheter Notify the primary health care provider immediately Start oxytocin (Pitocin) Call for help

Notify the primary HCP immediately

A nurse is assessing a client who is 12 hours postpartum. Which finding would be a source of concern if observed by the nurse? Bradycardia—pulse rate of 55 beats/min Postural hypotension Pain in left calf with dorsiflexion of left foot Temperature of 38° C

Pain in left calf with dorsiflexion of left foot These findings indicate presence of Homans sign, are suggestive of thrombophlebitis, and should be investigated. Postural hypotension is an expected finding related to circulatory changes after birth. A heart rate of 55 beats/min is an expected finding in the initial postpartum period. A temperature of 38° C in the first 24 hours most likely indicates dehydration, which is easily corrected by increasing oral fluid intake.

What are the 5 P's of labor?

Passenger (baby) Passageway (birth canal) Powers (primary dilate and efface, secondary expell fetus) Psychological Position

A nurse is reviewing phases of maternal postpartum adjustment. Which behaviors should the nurse identify as being exhibited during the letting-go phase of maternal role adaptation? (Select all that apply) Being talkative and excited about becoming a mother Sexual intimacy relationship continuing Emergence of family unit Defining one's individual roles Dependent behaviors

Sexual intimacy relationship continuing Emergence of family unit Defining one's individual roles

Why might it be more difficult to diagnose appendicitis during pregnancy? a. The appendix is displaced upward and laterally, high and to the right b. The appendix is displaced upward and laterally, high and to the left c. The appendix is deep at the McBurney's point d. The appendix is displaced downward and laterally, low andto the right

a

The nurse caring for a newborn checks the record to note clinical findings that occurred last shift. Which finding related to the renal system would be of increased significance and require further action? a. The newborn has not voided in 24 hours b. The breastfed infant voided more often than a formula feed infant c. Brick dustwas noted on several diapers d. Weight loss from fluid loss and other normal factors has yet to be regained

a

Which renal system adaptation is an anticipated anatomic change of pregnancy? a. Increased urinary output makes pregnant women less susceptible to urinary infections b. Increased bladder sensitivity and then compression of the bladder by the enlarging uterus result in the urge to urinate even when the bladder is almost empty c. Renal (kidney) function is more efficient when the woman assumes a supine position d. Using diuretic agents during pregnancy can help keep kidney function regular

b

The nurse caring for a pregnant client is evaluating health teaching regarding fetal circulation. Which statement from the client reassures the nurse that the teaching has been effective? a. "Optimal fetal circulation is achieved when I am in the side-lying position." b. "Optimal fetal circulation is achieved when I am on my back with a pillow under my knees." c. "Optimal fetal circulation is achieved when the head of the bed is elevated." d. "Optimal fetal circulation is achieved when I am on my abdomen."

a

The various systems and organs of the fetus develop at different stages. Which statement is most accurate? a. Cardiovascular system is the first organ system to function in the developing human b. Hematopoiesis originating in the yolk sac begins in the liver at 10 weeks of gestation c. Body changes from straight to C-shape occurs at 8 weeks of gestation d. Gastrointestinal system is mature at 32 weeks of gestation

a

What is the correct term for the cheeselike, white substance that fuses with the epidermis and serves as a protective coating? a. Vernix caseosa b. Surfactant c. Caput succedaneum d. Acrocyanosis

a

What is the correct term used to describe the mucous plug that forms in the endocervical canal? a. Operculum b. Leukorrhea c. Funic souffle d. Ballottement

a

What kind of fetal anomalies are most often associated with oligohydramnios? a. Renal b. Cardiac c. Gastrointestinal d. Neurologic

a

When assessing a multiparous woman who has just given birth to an 8-pound boy, the nurse notes that the woman's fundus is firm and has become globular. A gush of dark red blood comes from her vagina. How does the nurse interpret this finding? A cervical tear occurred during the birth Clots have formed in the upper uterine segment The woman is beginning to hemorrhage The placenta has separated

The placenta has separated

Which condition is considered a medical emergency that requires immediate treatment? a. Inversion of the uterus b. Hypotonic uterus c. Idiopathic thrombocytopenic purpura (ITP) d. Uterine atony

a

A woman asks the nurse, "What protects my baby's umbilical cord from being squashed while the baby's inside of me?" What is the nurse's best response? a. "Your baby's umbilical cord is surrounded by connective tissue called Wharton's jelly, which prevents compression of the blood vessels." b. "Your baby's umbilical cord floats around in blood and amniotic fluid." c. "You don't need to be worrying about things like that." d. "The umbilical cord is a group of blood vessels that are very well protected by the placenta."

a

A woman in labor passes some thick meconium as her amniotic fluid ruptures. The client asks the nurse where the baby makes the meconium. What is the correctresponse by the nurse? a. Fetal intestines b. Fetal kidneys c. Amniotic fluid d. Placenta

a

Which statement bestdescribes the rationale for the physiologic anemia that occurs during pregnancy? a. Physiologic anemia involves an inadequate intake of iron b. Dilution of hemoglobin concentration occurs in pregnancy with physiologic anemia c. Fetus establishes the iron stores d. Decreased production of erythrocytes occur

b

A first-time father is changing the diaper of his 1-day-old daughter. He asks the nurse, "What is this brown, sticky stuff in her diaper?" What is the nurse's bestresponse? a ."That's meconium, which is your baby's first stool. It's normal." b. "That's transitional stool." c. "That means your baby is bleeding internally." d. "Oh, don't worry about that. It's okay."

a

A first-time mother at 18 weeks of gestation is in for her regularly scheduled prenatal visit. The client tells the nurse that she is afraid that she is going into premature labor because she is beginning to have regular contractions. The nurse explains that these are Braxton Hickscontractions. What other information is important for the nurse to share? a. Braxton Hickscontractions should be painless b. They may increase in frequency with walking c. These contractions might cause cervical dilation d. Braxton Hicks contractions will impede oxygen flow to the fetus

a

A new mother asks the nurse about the "white substance" covering her infant. How should the nurse explain the purpose of vernix caseosa? a. Vernix caseosa protects the fetal skin from the amniotic fluid. b. Vernix caseosa promotes the normal development of the peripheral nervous system c. Vernix caseosa allows the transport of oxygen and nutrients across the amnion d. Vernix caseosa regulates fetal temperature

a

A new mother asks the nurse what the "experts say" about the best way to feed her infant. Which recommendation of the American Academy of Pediatrics (AAP) regarding infant nutrition should be shared with this client? a. Infants should be given only human milk for the first 6 months of life b. Infants fed on formula should be started on solid food sooner than breastfed infants c. If infants are weaned from breast milk before 12 months, then they should receive cow's milk, not formula d. After 6 months, mothers should shift from breast milk to cow's milk

a

A new mother asks whether she should feed her newborn colostrum, because it is not "real milk." What is the nurse's most appropriate answer? a. Colostrum is high in antibodies, protein, vitamins, and minerals b. Colostrum is lower in calories than milk and should be supplemented by formula c. Giving colostrum is important in helping the mother learn how to breastfeed before she goes home d. Colostrum is unnecessary for newborns

a

A pregnant woman wants to breastfeed her infant; however, her partner is not convinced that there are any scientific reasons to do so. The nurse can give the couple printed information comparing breastfeeding and bottle feeding. Which statement regarding bottle feeding using commercially prepared infant formulas might influence their choice? a. Bottle feeding using a commercially prepared formula increases the risk that the infant will develop allergies b. Bottle feeding helps the infant sleep through the night c. Commercially prepared formula ensures that the infant is getting iron in a form that is easily absorbed d. Bottle feeding requires that multivitamin supplements be given to the infant

a

With regard to the father's acceptance of the pregnancy and preparation for childbirth, the maternity nurse should know that: a. The father goes through three phases of acceptance of his own b. The father's attachment to the fetus cannot be as strong as that of the mother because it does not start until after birth c. In the last 2 months of pregnancy, most expectant fathers suddenly get very protective of their established lifestyle and resist making changes to the home d. Typically men remain ambivalent about fatherhood right up to the birth of their child

a A father typically goes through three phases of acceptance: accepting the biologic fact, adjusting to the reality, and focusing on his role. Typically, the expectant father's ambivalence ends by the first trimester, and he progresses to adjusting to the reality of the situation and focusing on his role. The father-child attachment can be as strong as the mother-child relationship and can begin during pregnancy. In the last 2 months of pregnancy, many expectant fathers work hard to improve the environment of the home for the child.

A woman who has completed one pregnancy with a fetus (or fetuses) reaching the stage of fetal viability is called a: a. Primipara b. Primigravida c. Multipara d. Nulligravida

a A primipara is a woman who has completed one pregnancy with a viable fetus. To remember terms, keep in mind that gravida is a pregnant woman; para comes from parity, meaning a viable fetus; primi means first; multi means many; and null means none. A primigravida is a woman pregnant for the first time. A multipara is a woman who has completed two or more pregnancies with a viable fetus. A nulligravida is a woman who has never been pregnant.

A new mother states that her infant must be cold because the baby's hands and feet are blue. This common and temporary condition is called what? a. Acrocyanosis b. Erythema toxicum neonatorum c. Harlequin sign d. Vernix caseosa

a Acrocyanosis,or the appearance of slightly cyanotic hands and feet, is caused by vasomotor instability, capillary stasis, and a high hemoglobin level. Acrocyanosis is normal and intermittently appears over the first 7 to 10 days after childbirth. Erythema toxicum neonatorum (also called erythema neonatorum) is a transient newborn rash that resembles flea bites. The harlequin sign is a benign, transient color change in newborns. One half of the body is pale, and the other one half is ruddy or bluish-red with a line of demarcation. Vernix caseosa is a cheeselike, whitish substance that serves as a protective covering for the newborn.

A woman is undergoing a nipple-stimulated contraction stress test (CST). She is having contractions that occur every 3 minutes. The fetal heart rate (FHR) has a baseline heart rate of approximately 120 beats per minute without any decelerations. What is the correctinterpretation of this test? a. Negative b. Positive c. Satisfactory d. Unsatisfactory

a Adequate uterine activity necessary for a CST consists of three contractions in a 10-minute time frame. If no decelerations are observed in the FHR pattern with the contractions, then the findings are considered to be negative. A positive CST indicates thepresence of repetitive late FHR decelerations. The terms satisfactoryor unsatisfactory are not applicable.

A maternity nurse should be aware of which fact about the amniotic fluid? a. It serves as a source of oral fluid and as a repository for waste from the fetus b. The volume remains about the same throughout the term of a healthy pregnancy c. A volume of less than 300 ml is associated with gastrointestinal malformations d. A volume of more than 2 L is associated with fetal renal abnormalities

a Amniotic fluid also cushions the fetus and helps maintain a constant body temperature. Its volume changes constantly; too little fluid (oligohydramnios) is associated with renal abnormalities, and too much fluid (polyhydramnios) is associated with gastrointestinal and other abnormalities.

Which information is the highestpriority for the nurse to comprehend regarding the biophysical profile (BPP)? a. BPP is an accurate indicator of impending fetal well-being b.BPP is a compilation of health risk factors of the mother during the later stages of pregnancy c. BPP consists of a Doppler blood flow analysis and an amniotic fluid index (AFI) d. BPP involves an invasive form of an ultrasonic examination.

a An abnormal BPP score is one indication that labor should be induced. The BPP evaluates the health of the fetus, requires many different measures, and is a noninvasive procedure.

What is the correct definition of a spontaneous termination of a pregnancy (abortion)? a. Pregnancy is less than 20 weeks b. Fetus weighs less than 1000 g c. Products of conception are passed intact d. No evidence exists of intrauterine infection

a An abortion is the termination of pregnancy before the age of viability (20 weeks). The weight of the fetus is not considered because some older fetuses may have a low birth weight. A spontaneous abortion may be complete or incomplete and may be caused by many problems, one being intrauterine infection.

39-year-old primigravida woman believes that she is approximately 8 weeks pregnant, although she has had irregular menstrual periods all her life. She has a history of smoking approximately one pack of cigarettes a day; however, she tells the nurse that she is trying to cut down. Her laboratory data are within normal limits. What diagnostic technique would be useful at this time? a. Ultrasound examination b. Maternal serum alpha-fetoprotein (MSAFP) screening c. Amniocentesis d. Nonstress test (NST)

a An ultrasound examination could be performed to confirm the pregnancy and to determine the gestational age of the fetus. An MSAFP screening is performed at 16 to 18 weeks of gestation; therefore, it is too early in the woman's pregnancy to perform this diagnostic test. An amniocentesis is performed if the MSAFP levels are abnormal or if fetal or maternal anomalies are detected. An NST is performed to assess fetal well-being in the third trimester.

A nurse is evaluating several obstetric clients for their risk for cervical insufficiency. Which client would the nurse consider to be at greatest risk? a. Primip who undergoes a cervical cone biopsy for cervical dysplasia prior to the pregnancy b. Grandmultip who has previously had all vaginal deliveries without a problem c. Multip who had her previous delivery via C section due to cephalopelvic disproportion (CPD) d. Primipara

a Any client who has had previous surgical interventions (cone biopsy) is at greater risk for cervical insufficiency. There is no indication that a primip is at risk for cervical insufficiency. A grandmultip who has previously had vaginal deliveries without incidence is not necessarily at an increased risk for cervical insufficiency. A multip who has delivered via C section as a result of CPD would not necessarily be at an increased risk as the issue involves pelvic adequacy as determined by pelvic measurements in relationship to the fetus.

What is the highest priority nursing intervention when admitting a pregnant woman who has experienced a bleeding episode in late pregnancy? a. Assessing fetal heart rate (FHR) and maternal vital signs b. Performing a venipuncture for hemoglobin and hematocrit levels c. Placing clean disposable pads to collect any drainage d. Monitoring uterine contractions

a Assessment of the FHR and maternal vital signs will assist the nurse in determining the degree of the blood loss and its effect on the mother and fetus. The most important assessment is to check the well-being of both the mother and the fetus. The blood levels can be obtained later. Assessing future bleeding is important; however, the top priority remains mother/fetal well-being. Monitoring uterine contractions is important but not a top priority.

During a client's physical examination, the nurse notes that the lower uterine segment is soft on palpation. The nurse would document this finding as: a. Hegar sign b. McDonald sign c. Chadwick sign d. Goodell sign

a At approximately 6 weeks of gestation, softening and compressibility of the lower uterine segment occur; this is called the Hegar sign. The Chadwick sign is a blue-violet cervix caused by increased vascularity; it is seen around the fourth week of gestation. Softening of the cervical tip, which may be observed around the sixth week of pregnancy, is called the Goodell sign. (The McDonald's sign indicates a fast-food restaurant.)

The client is being induced in response to worsening preeclampsia. She is also receiving magnesium sulfate. It appears that her labor has not become active, despite several hours of oxytocin administration. She asks the nurse, "Why is this taking so long?"What is the nurse's most appropriate response? a. "Since the magnesium is competing with the oxytocin, your labor is slowed." b. "I don't know why it is taking so long." c. "The length of labor varies for different women." d. "Your baby is just being stubborn."

a Because magnesium sulfate is a tocolytic agent, its use may increase the duration of labor. The amount of oxytocin needed to stimulate labor may be more than that needed for the woman who is not receiving magnesium sulfate. The nurse should explain to the client the effects of magnesium sulfate on the duration of labor. Although the length of labor varies for different women, the most likely reason this woman's labor is protracted is the tocolytic effects of magnesium sulfate. The behavior of the fetus has no bearing on the length of labor.

Which intervention can nurses use to prevent evaporative heat loss in the newborn? a. Drying the baby after birth, and wrapping the baby in a dry blanket b. Keeping the baby out of drafts and away from air conditioners c. Placing the baby away from the outside walls and windows d. Warming the stethoscope and the nurse's hands before touching the baby

a Because the infant is wet with amniotic fluid and blood, heat loss by evaporation quickly occurs.

Which intervention is most important when planning care for a client with severe gestational hypertension? a. Induction of labor is likely, as near term as possible b. If at home, the woman should be confined to her bed, even with mild gestational hypertension c. Special diet low in protein and salt should be initiated immediately d. Vaginal birth is still an option, even in severe cases

a By 34 weeks of gestation, the risk of continuing the pregnancy may be considered greater than the risks of a preterm birth. Strict bed rest is controversial for mild cases; some women in the hospital are even allowed to move around. Diet and fluid recommendations are essentially the same as for healthy pregnant women, although some authorities have suggested a diet high in protein. Women with severe gestational hypertension should expect a cesarean delivery.

The nurse sees a woman for the first time when she is 30 weeks pregnant. The woman has smoked throughout the pregnancy, and fundal height measurements now are suggestive of growth restriction in the fetus. In addition to ultrasound to measure fetal size, what would be another tool useful in confirming the diagnosis? a. Doppler blood flow analysis b. Contraction stress test (CST) c. Amniocentesis d. Daily fetal movement counts

a Doppler blood flow analysis allows the examiner to study the blood flow noninvasively in the fetus and the placenta. It is a helpful tool in the management of high risk pregnancy due to intrauterine growth restriction (IUGR), diabetes mellitus, multiple fetuses, or preterm labor. Because of the potential risk of inducing labor and causing fetal distress, a CST is not performed in a woman whose fetus is preterm. Indications for an amniocentesis include diagnosis of genetic disorders or congenital anomalies, assessment of pulmonary maturity, and the diagnosis of fetal hemolytic disease, not IUGR. Fetal kick count monitoring is performed to monitor the fetus in pregnancies complicated by conditions that may affect fetal oxygenation. Although it may be a useful tool at some point later in this woman's pregnancy, it is not used to diagnose IUGR.

The nurse sees a woman for the first time when she is 30 weeks pregnant. The client has smoked throughout the pregnancy, and fundal height measurements now are suggestive of intrauterine growth restriction (IUGR) in the fetus. In addition to ultrasound to measure fetal size, what is another tool useful in confirming the diagnosis? a. Doppler blood flow analysis b. Contraction stress test (CST) c. Amniocentesis d. Daily fetal movement counts

a Doppler blood flow analysis allows the examiner to study the blood flow noninvasively in the fetus and the placenta. It is a helpful tool in the management of high-risk pregnancies because of IUGR, diabetes mellitus, multiple fetuses, or preterm labor. Because of the potential risk of inducing labor and causing fetal distress, a CST is not performed on a woman whose fetus is preterm. Indications for an amniocentesis include diagnosis of genetic disorders or congenital anomalies, assessment of pulmonary maturity, and the diagnosis of fetal hemolytic disease, not IUGR. Fetal kick count monitoring is performed to monitor the fetus in pregnancies complicated by conditions that may affect fetal oxygenation. Although this may be a useful tool at some point later in this woman's pregnancy, it is not used to diagnose IUGR.

The nurse should be cognizant of which statement regarding the unique qualities of human breast milk? a. Frequent feedings during predictable growth spurts stimulate increased milk production b. Milk of preterm mothers is the same as the milk of mothers who gave birth at term c. Milk at the beginning of the feeding is the same as the milk at the end of the feeding d. Colostrum is an early, less concentrated, less rich version of mature milk

a Growth spurts (at 10 days, 3 weeks, 6 weeks, and 3 months) usually last 24 to 48 hours, after which the infants resume normal feeding

A woman experiencing severe preeclampsia has been receiving magnesium sulfate by intravenous infusion for 8 hours. The nurse assesses the client and documents the following findings: temperatureof 37.1° C, pulse rate of 96 beats per minute, respiratory rate of 24 breaths per minute, BP of 155/112 mm Hg, 3+ DTRs, and no ankle clonus. The nurse calls the provider with an update. The nurse should anticipate an order for which medication? a. Hydralazine b. Magnesium sulfate bolus c. Diazepam d. Calcium gluconate

a Hydralazine is an antihypertensive medication commonly used to treat hypertension in severe preeclampsia. Typically, it is administered for a systolic BP higher than 160 mm Hg or a diastolic BP higher than 110 mm Hg. An additional bolus of magnesium sulfate may be ordered for increasing signs of CNS irritability related to severe preeclampsia (e.g., clonus) or if eclampsia develops. Diazepam is sometimes used to stop or shorten eclamptic seizures. Calcium gluconate is used as the antidote for magnesium sulfate toxicity. The client is not currently displaying any signs or symptoms of magnesium toxicity.

A pregnant woman reports that she is still playing tennis at 32 weeks of gestation. The nurse would be most concerned regarding what this woman consumes during and after tennis matches. Which is the most important? a. Several glasses of fluid b. Extra protein sources, such as peanut butter c. Salty foods to replace lost sodium d. Easily digested sources of carbohydrate

a If no medical or obstetric problems contraindicate physical activity, pregnant women should get 30 minutes of moderate physical exercise daily. Liberal amounts of fluid should be consumed before, during, and after exercise, because dehydration can trigger premature labor. Also the woman's calorie and carbohydrate intake should be sufficient to meet the increased needs of pregnancy and the demands of exercise. All pregnant women should consume the necessary amount of protein in their diet, regardless of level of activity. Many pregnant women of this gestation tend to retain fluid, possibly contributing to hypertension and swelling. An adequate fluid intake prior to and after exercise should be sufficient without trying to replace sodium.

Many parents-to-be have questions about multiple births. Maternity nurses should be able to tell them that: a. Rates of twinning and other multiple births are increasing because of the use of fertility drugs and delayed childbearing b. Dizygotic twins (two fertilized ova) have the potential to be conjoined twins c. Identical twins are more common in Caucasian families d. Fraternal twins are same gender, usually male

a If the parents-to-be are older and have taken fertility drugs, they would be very interested in this information. Conjoined twins are monozygotic; they are from a single fertilized ovum in which division occurred very late. Identical twins show no racial or ethnic preference; fraternal twins are more common among African-American women. Fraternal twins can be different genders or the same gender. Identical twins are the same gender.

Part of the health assessment of a newborn is observing the infant's breathing pattern. What is the predominate pattern of newborn's breathing? a. Abdominal with synchronous chest movements b. Chest breathing with nasal flaring c. Diaphragmatic with chest retraction d. Deep with a regular rhythm

a In a normal infant respiration, the chest and abdomen synchronously rise and infant breaths are shallow and irregular. Breathing with nasal flaring is a sign of respiratory distress. Diaphragmatic breathing with chest retraction is also a sign of respiratory distress.

A woman with severe preeclampsia is being treated with an IV infusion of magnesium sulfate. Which finding would the nurse identify as indicating that the treatment is successful? a. Seizures do not occur b. Diuresis reduces fluid retention c. Blood pressure is reduced to prepregnant baseline d. Deep tendon reflexes become hypotonic

a Magnesium sulfate is a central nervous system (CNS) depressant given primarily to prevent seizures. A temporary decrease in blood pressure can occur but is not the purpose of administering this medication. Hypotonia is a sign of an excessive serum level of magnesium. It is critical that calcium gluconate be on hand to counteract the depressant effects of magnesium toxicity. Diuresis is not an expected outcome of magnesium sulfate administration.

Which analysis of maternal serum may predict chromosomal abnormalities in the fetus? a. Multiple-marker screening b. Lecithin-sphingomyelin ratio (L/S ratio) c. Biophysical profile (BPP) d. Blood type and crossmatch of maternal and fetal serum

a Maternal serum can be analyzed for abnormal levels of alpha-fetoprotein, human chorionic gonadotropin, and estriol. The multiple-marker screening may predict chromosomal defects in the fetus. The L/S ratio is used to determine fetal lung maturity. ABPP is used for evaluating fetal status during the antepartum period. Five variables are used, but none is concerned with chromosomal problems. The blood type and crossmatch would not predict chromosomal defects in the fetus.

Which neonatal complications are associated with hypertension in the mother? a. Intrauterine growth restriction (IUGR) and prematurity b. Seizures and cerebral hemorrhage c. Hepatic or renal dysfunction d. Placental abruption and DI

a Neonatal complications are related to placental insufficiency and include IUGR, prematurity, and necrotizing enterocolitis. Seizures and cerebral hemorrhage are maternal complications. Hepatic and renal dysfunction are maternal complications of hypertensive disorders in pregnancy. Placental abruption and DIC are conditions related to maternal morbidity and mortality.

The nurse caring for a pregnant woman knows that her health teaching regarding fetal circulation has been effective when the woman reports that she has been sleeping: a. In a side-lying position b. On her back with a pillow under her knees c. With the head of the bed elevated d. On her abdomen

a Optimal circulation is achieved when the woman is lying at rest on her side. Decreased uterine circulation may lead to intrauterine growth restriction. Previously it was believed that the left lateral position promoted maternal cardiac output, thereby enhancing blood flow to the fetus. However, it is now known that either side-lying position enhances uteroplacental blood flow. If a woman lies on her back with the pressure of the uterus compressing the vena cava, blood return to the right atrium will be diminished. Although having a pillow under the knees is recommended and ideal for later in pregnancy, the woman must still maintain a lateral tilt to the pelvis to avoid compression of the vena cava. Many women will find lying on the abdomen uncomfortable as pregnancy advances.

Which statement concerning neurologic and sensory development in the fetus is correct? a. Brain waves have been recorded on an electroencephalogram as early as the end of the first trimester (12 weeks of gestation) b. Fetuses respond to sound by 24 weeks of gestation and can be soothed by the sound of the mother's voice c. Eyes are first receptive to light at 34 to 36 weeks of gestation d. At term, the fetal brain is at least one third the size of an adult brain

b

Screening at 24 weeks of gestation reveals that a pregnant woman is experiencing gestational diabetes mellitus (GDM). In planning her care, the nurse and the client mutually agree that an expected outcome is to prevent injury to the fetus because of GDM. This fetus is at the greatest risk for which condition? a. Macrosomia b.Congenital anomalies of the central nervous system c. Preterm birth d. Low birth weight

a Poor glycemic control later in pregnancy increases the rate of fetal macrosomia. Poor glycemic control during the preconception time frame and into the early weeks of the pregnancy is associated with congenital anomalies. Preterm labor or birth is more likely to occur with severe diabetes and is the greatest risk in women with pregestational diabetes. Increased weight, or macrosomia, is the greatest risk factor for this fetus.

What nursing diagnosis is the most appropriate for a woman experiencing severe preeclampsia? a. Potential for injuryto mother and fetus, related to central nervous system (CNS) irritability b. Potential for reduced gas exchange c. Potential for inadequate fluid volume,related to increased sodium retention secondary to the administration of magnesium sulfate d. Potential for increased cardiac output,related to the use of antihypertensive drugs

a Potential for injury is the most appropriate nursing diagnosis for this client scenario. Gas exchange is more likely to become reduced, attributable to pulmonary edema. A potential for increased, not decreased, fluid volume, related to increased sodium retention, and a potential for decreased, not increased, cardiac output, related to the use of antihypertensive drugs, also is increased.

What is the most common medical complication of pregnancy? a. Hypertension b. Hyperemesis gravidarum c. Hemorrhagic complications d. Infections

a Preeclampsia and eclampsia are two noted deadly forms of hypertension. A large percentage of pregnant women will have nausea and vomiting, but a relatively few will have the severe form called hyperemesis gravidarum. Hemorrhagic complications are the second most common medical complication of pregnancy; hypertension is the most common. Infection is a risk factor for preeclampsia.

In caring for the woman with disseminated intravascular coagulation (DIC), which order should the nurse anticipate? a. Administration of blood b. Preparation of the client for invasive hemodynamic monitoring c. Restriction of intravascular fluids d. Administration of steroids

a Primary medical management in all cases of DIC involves a correction of the underlying cause, volume replacement, blood component therapy, optimization of oxygenation and perfusion status, and continued reassessment of laboratory parameters. Central monitoring would not be initially ordered in a client with DIC because it could contribute to more areas of bleeding. Management of DIC would include volume replacement, not volume restriction. Steroids are not indicated for the management of DIC.

Which statement concerning the complication of maternal diabetes is the most accurate? a. Diabetic ketoacidosis (DKA) can lead to fetal death at any time during pregnancy b. Hydramnios occurs approximately twice as often in diabetic pregnancies than in nondiabetic pregnancies c. Infections occur about as often and are considered about as serious in both diabetic and nondiabetic pregnancies d. Even mild-to-moderate hypoglycemic episodes can have significant effects on fetal well-being

a Prompt treatment of DKA is necessary to save the fetus and the mother. Hydramnios occurs 10 times more often in diabetic pregnancies. Infections are more common and more serious in pregnant women with diabetes. Mild-to-moderate hypoglycemic episodes do not appear to have significant effects on fetal well-being.

Which newborn reflex is elicited by stroking the lateral sole of the infant's foot from the heel to the ball of the foot? a. Babinski b. Tonic neck c. Stepping d. Plantar grasp

a The Babinski reflex causes the toes to flare outward and the big toe to dorsiflex. The tonic neck reflex (also called the fencing reflex) refers to the posture assumed by newborns when in a supine position. The stepping reflex occurs when infants are held upright with their heel touching a solid surface and the infant appears to be walking. Plantar grasp reflex is similar to the palmar grasp reflex; when the area below the toes is touched, the infant's toes curl over the nurse's finger.

Which information is an important consideration when comparing the contraction stress test (CST) with the nonstress test (NST)? a. The NST has no known contraindications b. The CST has fewer false-positive results when compared with the NST c. The CST is more sensitive in detecting fetal compromise, as opposed to the NST d. The CST is slightly more expensive than the NST.

a The CST has several contraindications. The NST has a high rate of false-positive results and is less sensitive than the CST but relatively inexpensive.

The nurse is preparing to administer a hepatitis B virus (HBV) vaccine to a newborn. Which intervention by the nurse is correct? a. Obtaining a syringe with a 25-gauge, 5/8-inch needle for medication administration b. Confirming that the newborn's mother has been infected with the HBV c. Assessing the dorsogluteal muscle as the preferred site for injection d. Confirming that the newborn is at least 24 hours old

a The HBV vaccine should be administered in the vastus lateralis muscle at childbirth with a 25-gauge, 5/8-inch needle and is recommended for all infants. If the infant is born to an infected mother who is a chronic HBV carrier, then the hepatitis vaccine and HBVimmunoglobulin should be administered within 12 hours of childbirth.

In caring for an immediate postpartum client, the nurse notes petechiae and oozing from her intravenous (IV) site. The client would be closely monitored for which clotting disorder? a. Disseminated intravascular coagulation (DIC) b. Amniotic fluid embolism (AFE) c. Hemorrhage d. HELLP syndrome

a The diagnosis of DIC is made according to clinical findings and laboratory markers. A physical examination reveals unusual bleeding. Petechiae may appear around a blood pressure cuff on the woman's arm. Excessive bleeding may occur from the site of slight trauma such as venipuncture sites. These symptoms are not associated with AFE, nor is AFE a bleeding disorder. Hemorrhage occurs for a variety of reasons in the postpartum client. These symptoms are associated with DIC. Hemorrhage would be a finding associated with DIC and is not a clotting disorder in and of itself. HELLP syndrome isnot a clotting disorder, but it may contribute to the clotting disorder DIC.

Which statement regarding the probable signs of pregnancy is most accurate? a. Determined by ultrasound b. Observed by the health care provider c. Reported by the client d. Confirmed by diagnostic tests

b

Which behavior indicates that a woman is "seeking safe passage" for herself and her infant? a. She keeps all prenatal appointments b. She "eats for two" c. She drives her car slowly d. She wears only low-heeled shoes

a The goal of prenatal care is to foster a safe birth for the infant and mother. Keeping all prenatal appointments is a good indication that the woman is indeed seeking "safe passage." Eating properly, driving carefully, using proper body mechanics, and wearing appropriate footwear during pregnancy are healthy measures that all pregnant women should take.

Which major neonatal complication is carefully monitored after the birth of the infant of a diabetic mother? a. Hypoglycemia b. Hypercalcemia c. Hypobilirubinemia d. Hypoinsulinemia

a The neonate is at highest risk for hypoglycemia because fetal insulin production is accelerated during pregnancy to metabolize excessive glucose from the mother. At birth, the maternal glucose supply stops and the neonatal insulin exceeds the available glucose, thus leading to hypoglycemia. Hypocalcemia is associated with preterm birth, birth trauma, and asphyxia, all common problems of the infant of a diabetic mother.Excess erythrocytes are broken down after birth, and large amounts of bilirubin are released into the neonate's circulation, with resulting hyperbilirubinemia. Because fetal insulin production is accelerated during pregnancy, hyperinsulinemia develops in the neonate.

A pregnant woman's biophysical profile (BPP) score is 8. She asks the nurse to explain the results. How should the nurse respond initially? a. "The test results are within normal limits." b. "Immediate delivery by cesarean birth is being considered." c. "Further testing will be performed to determine the meaning of this score." d. "An obstetric specialist will evaluate the results of this profile and, within the next week, will inform you of your options regarding delivery."

a The normal biophysical score ranges from 8 to 10 points if the amniotic fluid volume is adequate. A normal score allows conservative treatment of high-risk clients. Delivery can be delayed if fetal well-being is indicated. Scores less than 4 should be investigated, and delivery could be initiated sooner than planned. The results of the BPP are usually available immediately after the procedure is performed. Since this score is within normal range, no further testing is required at this time.

With regard to medications, herbs, shots, and other substances normally encountered, the maternity nurse should be aware that during pregnancy: a. Prescription and over-the-counter (OTC) drugs that otherwise are harmless can be made hazardous by metabolic deficiencies of the fetus b. The greatest danger of drug-caused developmental deficits in the fetus is seen in the final trimester c. Killed-virus vaccines (e.g., tetanus) should not be given, but live-virus vaccines (e.g., measles) are permissible d. No convincing evidence exists that secondhand smoke is potentially dangerous to the fetus

a The statement in A is especially true for new medications and combinations of drugs. The greatest danger of drug-caused developmental defects exists in the interval from fertilization through the first trimester, when a woman may not realize that she is pregnant. Live-virus vaccines should be part of postpartum care; killed-virus vaccines may be administered during pregnancy. Secondhand smoke is associated with fetal growth restriction and increases in infant mortality.

Nursing follow-up care often includes home visits for the new mother and her infant. Which information related to home visits is correct? a. Ideally, the visit is scheduled within 72 hours after discharge b. Home visits are available in all areas c. Visits are completed within a 30-minute time frame d. Blood draws are not a part of the home visit

a This timing allows early assessment and intervention for problems with feedings, jaundice, newborn adaptation, and maternal-infant interaction. Because of geographic distances, home visits are not available in all locales. Visits are usually 60 to 90 minutes in length to allow enough time for assessment and teaching. When jaundice is found, the nurse can discuss the implications and check the transcutaneous bilirubin level or draw blood for testing.

A nurse is reviewing clinical diagnoses of preeclampsia and eclampsia. Which statement should the nurse be aware of? a. Preeclampsia results in decreased function in such organs as the placenta, kidneys, liver, and brain b. Preeclampsia is a condition of the first trimester; eclampsia is a condition of the second and third trimesters c. The causes of preeclampsia and eclampsia are well documented d. Severe preeclampsia is defined as preeclampsia plus proteinuria

a Vasospasms diminish the diameter of blood vessels, which impedes blood flow to all organs. Preeclampsia occurs after week 20 of gestation and can run the duration of the pregnancy. The causes of preeclampsia and eclampsia are unknown, although several have been suggested. Preeclampsia includes proteinuria; severe cases are characterized by greater proteinuria or any of nine other conditions.

Which assessments are included in the fetal biophysical profile (BPP)? Select all that apply a. Fetal movement b. Fetal tone c. Fetal heart rate d. Amniotic fluid index (AFI) e. Placental grade

a, b, c, d

Intrauterine growth restriction (IUGR) is associated with which pregnancy-related risk factors? Select all that apply a. Poor nutrition b. Multifetal pregnancy c. Gestational hypertension d. Premature rupture of membranes e. Smoking

a, b, c, e

Cell-free deoxyribonucleic acid (cfDNA) screening is a new method of noninvasive prenatal testing (NIPT) that has recently become available in the clinical setting. This technology can provide a definitive diagnosis of which findings? Select all that apply a. Fetal Rh status b. Fetal gender c. Maternally transmitted gene disorder d. Paternally transmitted gene disorder e. Trisomy 21

a, b, d, e

The parents of a newborn ask the nurse how much the newborn can see. The parents specifically want to know what type of visual stimuli they should provide for their newborn. What information provided by the nurse would be most useful to these new parents? a. "Infants can see very little until approximately 3 months of age." b. "Clearest visual distance is 8 to 12 inches and they can distinguish patterns; preferring complex ones." c. "The infant's eyes must be protected. Infants enjoy looking at brightly colored stripes." d. "It's important to shield the newborn's eyes. Overheadlights help them see better."

b

The nurse suspects that a client has early signs of ectopic pregnancy. The nurse should be observing the client for which signs or symptoms? Select all that apply a. Pelvic pain b. Abdominal pain c. Unanticipated heavy bleeding d. Vaginal spotting or light bleeding e. Missed period

a, b, d, e A missed period or spotting can be easily mistaken by the client as an early sign of pregnancy. More subtle signs depend on exactly where the implantation occurs. The nurse must be thorough in her assessment because pain is not a normal symptom of early pregnancy. As the fallopian tube tears open and the embryo is expelled, the client often exhibits severe pain accompanied by intraabdominal hemorrhage, which may progress to hypovolemic shock with minimal or even no external bleeding. In approximately one half of women, shoulder and neck pain results from irritation of the diaphragm from the hemorrhage.

Diabetes refers to a group of metabolic diseases characterized by hyperglycemia resulting from defects in insulin action, insulin secretion, or both. Over time, diabetes causes significant changes in the microvascular and macrovascular circulations. What do these complications include? Select all that apply a. Atherosclerosis b. Retinopathy c. Intrauterine fetal death (IUFD) d. Nephropathy e. Neuropathy f. Autonomic neuropathy

a, b, d, e These structural changes will most likely affect a variety of systems, including the heart, eyes, kidneys, and nerves. IUFD (stillbirth) remains a major complication of diabetes in pregnancy; however, this is a fetal complication.

One of the most important components of the physical assessment of the pregnant client is the determination of blood pressure (BP). Which techniques are important in obtaining accurate BP readings? Select all that apply a. The client should be seated b. The client's arm should be placed at the level of the heart c. An electronic BP device should be used d. The cuff should cover a minimum of 60% of the upper arm e. The same arm should be used for every reading

a, b, e BP readings are easily affected by maternal position. Ideally, the client should be seated. An alternative position is left lateral recumbent with the arm at the level of the heart. The arm should always be held in a horizontal position at approximately the level of the heart. The same arm should be used at every visit. The manual sphygmomanometer is the most accurate device. If manualand electronic devices are used in the care setting, then the nurse must use caution when interpreting the readings. A proper size cuff should cover at least 80% of the upper arm or be approximately 1.5 times the length of the upper arm.

A serious but uncommon complication of undiagnosed or partially treated hyperthyroidism is a thyroid storm, which may occur in response to stress such as infection, birth, or surgery. What are the signs and symptoms of this emergency disorder? Select all that apply a. Fever b. Hypothermia c. Restlessness d. Bradycardia e. Hypertension

a, c Fever, restlessness, tachycardia, vomiting, hypotension, and stupor are symptomsof a thyroid storm. Fever, not hypothermia; tachycardia, not bradycardia; and hypotension, not hypertension, are symptoms of thyroid storm.

Relating to the fetal circulatory system, which special characteristics allow the fetus to obtain enough oxygen from the maternal blood? (Select all that apply.) a. Fetal hemoglobin (Hb) carries 20% to 30% more oxygen than maternal Hb b. Fetal Hb carries40% to 50% more oxygen than maternal Hb c. Hb concentration is 50% higher than that of the mother d. Fetal heart rate is 110 to 160 beats per minute e. Fetal heart rate is 160 to 200 beats per minute

a, c, d

The diagnosis of pregnancy is based on which positive signs of pregnancy? (Select all that apply.) a. Identification of fetal heartbeat b. Palpation of fetal outline c. Visualization of the fetus d. Verification of fetal movement e. Positive hCG test

a, c, d

Which adverse prenatal outcomes are associated with the HELLP syndrome? Select all that apply a. Placental abruption b. Placenta previa c. Renal failure d. Cirrhosis e. Maternal and fetal death

a, c, e The HELLP syndrome is associated with an increased risk for adverse perinatal outcomes, including placental abruption, acute renal failure, subcapsular hepatic hematoma, hepatic rupture, recurrent preeclampsia, preterm birth, and fetal and maternal death. The HELLP syndrome is associated with an increased risk for placental abruption, not placenta previa. It is also associated with an increased risk for hepatic hematoma, not cirrhosis.

A client who has undergone a dilation and curettage (D&C)for early pregnancy loss is likely to be discharged the same day. The nurse must ensure that her vital signs are stable, that bleeding has been controlled, and that the woman has adequately recovered from the administration of anesthesia. To promote an optimal recovery, what information should discharge teaching include? Select all that apply a. Iron supplementation b. Resumption of intercourse at 6 weeks post-procedure c. Referral to a support group, if necessary d. Expectation of heavy bleeding for at least 2 weeks e. Emphasizing the need for rest

a, c, e The woman should be advised to consume a diet high in iron and protein. For many women, iron supplementation also is necessary. The nurse should acknowledge that the client has experienced a loss, however early. She can be taught to expect mood swings and possibly depression. Referral to a support group, clergy, or professional counseling may be necessary. Discharge teaching should emphasize the need for rest. Nothing should be placed in the vagina for 2 weeks after the procedure, including tampons and vaginal intercourse. The purpose of this recommendation is to prevent infection. Should infection occur, antibiotics may be prescribed. The client should expect a scant, dark discharge for 1 to 2 weeks. Should heavy, profuse, or bright bleeding occur, she should be instructed to contact her health care provider.

A 31-year-old woman believes that she may be pregnant. She took an over-the-counter (OTC) pregnancy test 1 week ago after missing her period; the test was positive. During her assessment interview, the nurse asks about the woman's last menstrual period and whether she is taking any medications. The client states that she takes medicine for epilepsy. She has been under considerable stress lately at work and has not been sleeping well. Her physical examination does not indicate that she is pregnant. She has an ultrasound scan, which confirms that she is not pregnant. What is the most likely cause of the false-positive pregnancy test result? a. The pregnancy test was taken too early b. Anticonvulsant medications may cause the false-positive test result c. The woman has a fibroid tumor d. She has been under considerable stress and has a hormone imbalance

b

A client is seen at the clinic at 14 weeks of gestation for a follow-up appointment. At which level does the nurse expect to palpate the fundus? a. Nonpalpable above the symphysis at 14 weeks of gestation b. Slightly above the symphysis pubis c. At the level of the umbilicus d. Slightly above the umbilicus

b

During a prenatal examination, a woman reports having two cats at home. The nurse informs her that she should not be cleaning the litter box while she is pregnant. The client questions the nurse as to why. What is the nurse's most appropriate response? a. "Your cats could be carrying toxoplasmosis. This is a zoonotic parasite that can infect you and have severe effects on your unborn child." b. "You and your baby can be exposed to the HIV in your cats' feces." c. "It's just gross. You should make your husband clean the litter boxes." d. "Cat feces are known to carry Escherichia coli, which can cause a severe infection in you and your baby."

a. "Your cats could be carrying toxoplasmosis. This is a zoonotic parasite that can. Toxoplasmosis is a multisystem disease caused by the protozoal Toxoplasma gondii parasite, commonly found in cats, dogs, pigs, sheep, and cattle. Approximately 30% of women who contract toxoplasmosis during gestation transmit the disease to their offspring. Clinical features ascribed to toxoplasmosis include hydrocephalus or microcephaly, chorioretinitis, seizures, or cerebral calcifications. HIV is not transmitted by cats. Although cleaning the litter boxes is "just gross," this statement is not appropriate, fails to answer the client's question, and is not the nurse's best response. E. coli is found in normal human fecal flora and is not transmitted by cats.

Which substance, when abused during pregnancy, is the most significant cause of cognitive impairment and dysfunction in the infant? a. Alcohol b. Tobacco c. Marijuana d. Heroin

a. Alcohol. Alcohol abuse during pregnancy is recognized as one of the leading causes of neurodevelopmental disorders in the United States. Alcohol is a teratogen; maternal ethanol abuse during gestation can lead to identifiable fetal alcohol spectrum disorders that include alcohol-related neurodevelopmental disorders. Cigarette smoking is linked to adverse pregnancy outcomes; the risk for placenta previa, placenta abruption, and premature rupture of membranes is twice that of nonsmokers. Marijuana is the most common illicit drug used by pregnant women. Marijuana crosses the placenta, and its use during pregnancy can result in shortened gestation and a higher incidence of IUGR. Heroin crosses the placenta and often results in IUGR, stillbirth, and congenital anomalies.

A pregnant woman arrives at the birth unit in labor at term, having had no prenatal care. After birth, her infant is noted to be small for gestational age with small eyes and a thin upper lip. The infant also is microcephalic. Based on her infant's physical findings, this woman should be questioned about her use of which substance during pregnancy? a. Alcohol b. Cocaine c. Heroin d. Marijuana

a. Alcohol. The description of the infant suggests fetal alcohol syndrome, which is consistent with maternal alcohol consumption during pregnancy. Fetal brain, kidney, and urogenital system malformations have been associated with maternal cocaine ingestions. Heroin use in pregnancy frequently results in intrauterine growth restriction (IUGR). The infant may have a shrill cry and sleep-cycle disturbances and may exhibit with poor feeding, tachypnea, vomiting, diarrhea, hypothermia or hyperthermia, and sweating. Studies have found a higher incidence of meconium staining in infants born of mothers who used marijuana during pregnancy.

A 3.8-kg infant was vaginally delivered at 39 weeks after a 30-minute second stage. A nuchal cord was found at delivery. After birth, the infant is noted to have petechiae over the face and upper back. Which information regarding petechiae is most accurate and should be provided to the parents? a. Are benign if they disappear within 48 hours of birth b. Result from increased blood volume c. Should always be further investigated d. Usually occur with a forceps-assisted delivery

a. Are benign if they disappear within 48 hours of birth. Petechiae, or pinpoint hemorrhagic areas, acquired during childbirth may extend over the upper portion of the trunk and face. These lesions are benign if they disappear within 2 days of childbirth and no new lesions appear. Petechiae may result from decreased platelet formation. In this situation, the presence of petechiae is most likely a soft-tissue injury resulting from the nuchal cord at birth. Unless the lesions do not dissipate in 2 days, alarming the family is not necessary. Petechiae usually occur with a breech presentation vaginal birth.

Epidural block

anesthetic injected into epidural space no sensation from waist to knees admin if dilated 4cm or more main side effect: maternal hypotension Causes longer second stage of labor, decreased urge to push, N/V, bladder distention

The use of methamphetamine (meth) has been described as a significant drug problem in the United States. The nurse who provides care to this client population should be cognizant of what regarding methamphetamine use? a. Methamphetamines are similar to opiates b. Methamphetamines are stimulants with vasoconstrictive characteristics c. Methamphetamines should not be discontinued during pregnancy d. Methamphetamines are associated with a low rate of relapse

b

A new father is ready to take his wife and newborn son home. He proudly tells the nurse who is discharging them that within the next week he plans to start feeding the infant cereal between breastfeeding sessions. Which information should the nurse provide regarding this feeding plan? a. "Feeding solid foods before your son is 4 to 6 months old may decrease your son's intake of sufficient calories." b. "Feeding solid foods between breastfeeding sessions before your son is 4 to 6 months old will lead to an early cessation of breastfeeding." c. "Your feeding plan will help your son sleep through the night." d. "Feeding solid foods before your son is 4 to 6 months old will limit his growth."

b

A new father wants to know what medication was put into his infant's eyes and why it is needed. How does the nurse explain the purpose of the erythromycin ophthalmic ointment? a. Erythromycin ophthalmic ointment destroys an infectious exudate caused by Staphylococcusthat could make the infant blind b. This ophthalmic ointment prevents gonorrheal infection of the infant's eyes, potentially acquired from the birth canal c. Erythromycin prevents potentially harmful exudate from invading the tear ducts of the infant's eyes, leading to dry eyes d. This ointment prevents the infant's eyelids from sticking together and helps the infant see

b

A new mother recalls from prenatal class that she should try to feed her newborndaughter when she exhibits feeding readiness cues rather than waiting until the baby is frantically crying. Which feeding cue would indicate that the baby is ready to eat? a. Waves her arms in the air b. Makes sucking motions c. Has the hiccups d. Stretches out her legs straight

b

A patient in her first trimester complains of nausea and vomiting. She asks, "Why does this happen?" What is the nurse's bestresponse? a. "Nausea and vomiting are due to an increase in gastric motility." b. "Nausea and vomiting may be due to changes in hormones." c. "Nausea and vomiting are related to an increase in glucose levels." d. "Nausea and vomiting are caused by a decrease in gastric secretions."

b

A primiparous woman is watching her newborn sleep. She wants him to wake up and respond to her. The mother asks the nurse how much he will sleep every day. What is an appropriate response by the nurse? a. "He will only wake up to be fed, and you should notbother him between feedings." b. "The newborn sleeps approximately 17 hours a day, with periods of wakefulness gradually increasing." c. "He will probably follow your same sleep and wake patterns, and you can expect him to be awake soon." d. "He is being stubborn by not waking up when you want him to. You should try to keep him awake during the daytime so that he will sleep through the night."

b

A woman who is 16 weeks pregnant asks the nurse, "Is it possible to tell by ultrasound if the baby is a boy or girl yet?" What is the bestanswer? a. "A baby's sex is determined as soon as conception occurs." b. "The baby has developed enough to enable us to determine the sex by examining the genitals through an ultrasound scan." c. "Boys and girls look alike until approximately 20 weeks after conception, and then they begin to look different." d. "It might be possible to determine your baby's sex, but the external organs look very similar right now."

b

At 1 minute after birth a nurse assesses an infant and notes a heart rate of 80 beats per minute, some flexion of extremities, a weak cry, grimacing, and a pink body but blue extremities. Which Apgar score does the nurse calculate based upon these observations and signs? a. 4 b. 5 c. 6 d. 7

b

Cardiac output increases up to 50% by the 32nd week of pregnancy. What is the rationale for this change? a. To compensate for the decreased renal plasma flow b. To provide adequate perfusion of the placenta c. To eliminate metabolic wastes of the mother d. To prevent maternal and fetal dehydration

b

It is extremely rare for a woman to die in childbirth; however, it can happen. In the United States, the annual occurrence of maternal death is 12 per 100,000 cases of live birth. What are the leading causes of maternal death? a. Embolism and preeclampsia b. Falls and motor vehicle accidents (MVAs) c. Hemorrhage and infection d. Underlying chronic conditions

b

Of which physiologic alteration of the uterus during pregnancy is it important for the nurse to alert the patient? a. Lightening occurs near the end of the second trimester as the uterus rises into a different position b. Woman's increased urinary frequency in the first trimester is the result of exaggerated uterine anti-reflexion caused by softening c. Braxton Hicks contractions become more painful in the third trimester, particularly if the woman tries to exercise d. Uterine souffle is the movement of the fetus

b

The breastfeeding mother should be taught a safe method to remove the breast from the baby's mouth. Which suggestion by the nurse is most appropriate? a. Slowly remove the breast from the baby's mouth when the infant has fallen asleep and the jaws are relaxed b. Break the suction by inserting your finger into the corner of the infant's mouth c. A popping sound occurs when the breast is correctly removed from the infant's mouth d. Elicit the Moro reflex to wake the baby and remove the breast when the baby cries

b

The musculoskeletal system adapts to the changes that occur throughout the pregnancy. Which musculoskeletal alteration should the client expect? a. Her center of gravity will shift backward b. She will have increased lordosis c. She will have increased abdominal muscle tone d. She will notice decreased mobility of her pelvic joints

b

The nurse is providing health education to a pregnant client regarding the cardiovascular system. Which information is correctand important to share? a. A pregnant woman experiencing disturbed cardiac rhythm requires close medical and obstetric observation no matter how healthy she may appear otherwise b. Changes in heart size and position and increases in blood volume create auditory changes from 20 weeks of gestation to term c. Palpitations are twice as likely to occur in twin gestations d. All of the above changes will likely occur

b

The nurse knows that teaching about the natural family planning method of contraception was effective when the couple responds that an ovum is considered fertile for which period of time? a. 6 to 8 hours b. 24 hours c. 2 to 3 days d. 1 week

b

The pancreas forms in the foregut during the 5th to 8th week of gestation. A client with poorly controlled gestational diabetes asks the nurse what the effects of her condition will be on the fetus. What is the best response by the nurse? Poorly controlled maternal gestational diabetes will: a. produce fetal hypoglycemia b. result in a macrocosmic fetus c. result in a microcosmic fetus d. enhance lung maturation

b

What type of cultural concern is the most likely deterrent to many women seeking prenatal care? a. Religion b. Modesty c. Ignorance d. Belief that physicians are evil

b A concern for modesty is a strong deterrent to many women seeking prenatal care; for some women, exposing body parts, especially to a man, is considered a major violation of modesty. There are other deterrents. Even if the prenatal care described is familiar to a woman, some practices may conflict with the beliefs and practices of a subculture group or religion to which she belongs. For many cultural groups a physician is deemed appropriate only in times of illness. Because pregnancy is considered a normal process and the woman is in a state of health, the services of a physician are considered inappropriate. Many cultural variations are found in prenatal care, so ignorance is not likely to be a deterrent to women seeking prenatal care.

With regard to nutritional needs during lactation, a maternity nurse should be aware that: a. The mother's intake of vitamin C, zinc, and protein now can be lower than during pregnancy b. Caffeine consumed by the mother accumulates in the infant, who therefore may be unusually active and wakeful c. Critical iron and folic acid levels must be maintained d. Lactating women can go back to their pre-pregnant calorie intake

b A lactating woman needs to avoid consuming too much caffeine. Vitamin C, zinc, and protein levels need to be moderately higher during lactation than during pregnancy. The recommendations for iron and folic acid are somewhat lower during lactation. Lactating women should consume about 500 kcal more than their prepregnancy intake, at least 1800 kcal daily overall.

A nurse is reviewing the clinical diagnosis of ectopic pregnancy. Which location should the nurse identify as being the most common location for this occurrence? a. Fimbriae b. Ampulla c. Uterine fundus d. Cervical os

b A pregnancy within the uterus would be considered a normal pregnancy. Implantation of the pregnancy at the cervical os would be a significant abnormality. The majority of ectopic pregnancies, approximately 80%, are located in the ampulla or largest portion of the tube.

The client being cared for has severe preeclampsia and is receiving a magnesium sulfate infusion. Which new finding would give the nurse cause for concern? a. Sleepy, sedated affect b. Respiratory rate of 10 breaths per minute c. Deep tendon reflexes (DTRs) of 2+ d. Absent ankle clonus

b A respiratory rate of 10 breaths per minute indicates the client is experiencing respiratory depression from magnesium toxicity. Because magnesium sulfate is a CNS depressant, the client will most likely become sedated when the infusion is initiated. DTRs of 2+ and absent ankle clonus are normal findings.

A woman presents to the emergency department complaining of bleeding and cramping. The initial nursing history is significant for a last menstrual period 6 weeks ago. On sterile speculum examination, the primary health care provider finds that the cervix is closed. The nurse bases the anticipated plan of care for this woman as it relates to a probable diagnosis of which type of spontaneous abortion? a. Incomplete b. Threatened c. Septic d. Inevitable

b A woman with a threatened abortion presents with spotting, mild cramps, and no cervical dilation. Heavy bleeding, mild to severe cramping, and cervical dilation are the presentation for both incomplete abortion and inevitable abortion. A woman with a septic abortion presents with malodorous bleeding and, typically, a dilated cervix.

The nurse is explaining the benefits associated with breastfeeding to a new mother. Which statement by the nurse would provide conflicting information to the client? a. Women who breastfeed have a decreased risk of breast cancer b. Breastfeeding is an effective method of birth control c. Breastfeeding increases bone density d. Breastfeeding may enhance postpartum weight loss

b Although breastfeeding delays the return of fertility, it is not an effective birth control method.

A woman who is 32 weeks pregnant is informed by the nurse that a danger sign of pregnancy could be: a. Constipation b. Alteration in the pattern of fetal movement c. Heart palpitations d. Edema in the ankles and feet at the end of the day

b An alteration in the pattern or amount of fetal movement may indicate fetal jeopardy. Constipation is a normal discomfort of pregnancy that occurs in the second and third trimesters. Heart palpitations are a normal change related to pregnancy; they are most likely to occur during the second and third trimesters. As the pregnancy progresses, edema in the ankles and feet at the end of the day is not uncommon.

Which information should the nurse take into consideration when planning care for a postpartum client with cardiac disease? a. The plan of care for a postpartum client is the same as the plan for any pregnant woman b. The plan of care includes rest, stool softeners, and monitoring of the effect of activity c. The plan of care includes frequent ambulating, alternating with active range-of-motion exercises d. The plan of care includes limiting visits with the infant to once per day

b Bed rest may be ordered, with or without bathroom privileges. Bowel movements without stress or strain for the woman are promoted with stool softeners, diet, and fluids. Care of the woman with cardiac disease in the postpartum period is tailored to the woman's functional capacity. The woman will be on bed rest to conserve energy and to reduce the strain on the heart. Although the woman may need help caring for the infant, breastfeeding and infant visits are not contraindicated.

An 18-year-old client who has reached 16 weeks of gestation was recently diagnosed with pre-gestational diabetes. She attends her centering appointment accompanied by one of her girlfriends. This young woman appears more concerned about how her pregnancy will affect her social life than her recent diagnosis of diabetes. Several nursing diagnoses are applicable to assist in planning adequate care. What is the most appropriate diagnosis at this time? a. Potential for injury to the fetus related to birth trauma b. Lack of understanding related to diabetic pregnancy management c. Lack of understanding related to insulin administration d. Potential for injury to the mother related to hypoglycemia or hyperglycemia

b Before a treatment plan is developed or goals for the outcome of care are outlined, this client must come to an understanding of diabetes and the potential effects on herpregnancy. She appears more concerned about changes to her social life than adopting a new self-care regimen. Potential for injury to the fetus related to either placental insufficiency or birth trauma may come later in the pregnancy. At this time, the client is having difficulty acknowledging the adjustments that she needs to make to her lifestyle to care for herself during pregnancy. The client may not yet be on insulin. Insulin requirements increase with gestation. The importance of glycemic control must be part of health teaching for this client. However, she has not yet acknowledged that changes to her lifestyle need to be made and may not participate in the plan of care until understanding takes place.

A perinatal nurse is giving discharge instructions to a woman, status post-suction, and curettage secondary to a hydatidiform mole. The woman asks why she must take oral contraceptives for the next 12 months. What is the bestresponse by the nurse? a. "If you get pregnant within 1 year, the chance of a successful pregnancy is very small. Therefore, if you desire a future pregnancy, it would be better for you to use the most reliable method of contraception available." b. "The major risk to you after a molar pregnancy is a type of cancer that can be diagnosed only by measuring the same hormone that your body produces during pregnancy. If you were to get pregnant, then it would make the diagnosis of this cancer more difficult." c. "If you can avoid a pregnancy for the next year, the chance of developing a second molar pregnancy is rare. Therefore, to improve your chance of a successful pregnancy, not getting pregnant at this time is best." d. "Oral contraceptives are the only form of birth control that will prevent a recurrence of a molar pregnancy."

b Beta-human chorionic gonadotropin (beta-hCG) hormone levels are drawn for 1 yearto ensure that the mole is completely gone. The chance of developing choriocarcinoma after the development of a hydatidiform mole is increased. Therefore, the goal is to achieve a zero human chorionic gonadotropin (hCG) level. If the woman were to become pregnant, then it may obscure the presence of the potentially carcinogenic cells. Women should be instructed to use birth control for 1 year after treatment for a hydatidiform mole. The rationale for avoiding pregnancy for 1 year is to ensure that carcinogenic cells are not present. Any contraceptive method except an intrauterine device (IUD) is acceptable.

Cardiovascular system changes occur during pregnancy. Which finding would be considered normal for a woman in her second trimester? a. Less audible heart sounds (S1, S2) b. Increased pulse rate c. Increased blood pressure d. Decreased red blood cell (RBC) production

b Between 14 and 20 weeks of gestation, the pulse increases about 10 to 15 beats/min, which persists to term. Splitting of S1 and S2 is more audible. In the first trimester blood pressure usually remains the same as the prepregnancy level, but it gradually decreases up to about 20 weeks of gestation. During the second trimester both the systolic and diastolic pressures decrease by about 5 to 10 mm Hg. Production of RBCs accelerates during pregnancy.

Women with inadequate weight gain during pregnancy are at higher risk of giving birth to an infant with: a. Spina bifida b. Intrauterine growth restriction c. Diabetes mellitus d. Down syndrome

b Both normal-weight and underweight women with inadequate weight gain have an increased risk of giving birth to an infant with intrauterine growth restriction. Spina bifida is not associated with inadequate maternal weight gain; an adequate amount of folic acid has been shown to reduce the incidence of this condition. Diabetes mellitus is not related to inadequate weight gain. A mother with gestational diabetes is more likely to give birth to a large-for-gestational age infant. Down syndrome is the result of trisomy 21, not inadequate maternal weight gain.

Which statement best describes the transition period between intrauterine and extrauterine existence for the newborn? a. Consists of four phases, two reactive and two of decreased responses b. Lasts from birth to day 28 of life c. Applies to full-term births only d. Varies by socioeconomic status and the mother's age

b Changes begin immediately after birth; the cutoff time when the transition is considered over (although the baby keeps changing) is 28 days. This transition period has three phases: first reactivity, decreased response, and second reactivity. All newborns experience this transition period, regardless of age or type of birth.

A client with maternal phenylketonuria (PKU) has come to the obstetrical clinic to begin prenatal care. Why would this preexisting condition result in the need for closer monitoring during pregnancy? a. PKU is a recognized cause of preterm labor b. The fetus may develop cognitive problems c. A pregnant woman is more likely to die without strict dietary control d. Women with PKU are usually mentally handicapped and should not reproduce

b Children born to women with untreated PKU are more likely to be born with mental retardation, microcephaly, congenital heart disease, and low birth weight. Maternal PKU has no effect on labor. Women without dietary control of PKU are more likely to miscarry or bear a child with congenital anomalies. Screening for undiagnosed maternal PKU at the first prenatal visit may be warranted, especially in individuals with a family history of the disorder, with low intelligence of an uncertain cause, or who have given birth to microcephalic infants.

Diabetes in pregnancy puts the fetus at risk in several ways. Nurses should be aware that: a. With good control of maternal glucose levels, sudden and unexplained stillbirth is no longer a major concern b. The most important cause of perinatal loss in diabetic pregnancy is congenital malformations c. Infants of mothers with diabetes have the same risks for respiratory distress syndrome because of the careful monitoring d. At birth, the neonate of a diabetic mother is no longer in any greater risk

b Congenital malformations account for 30% to 50% of perinatal deaths in diabetic pregnancies. Even with good control, sudden and unexplained stillbirth remains a major concern. Infants of diabetic mothers are at increased risk for respiratory distress syndrome, and the transition to extrauterine life is often marked by hypoglycemia and other metabolic abnormalities.

What is an appropriate indicator for performing a contraction stress test? a. Increased fetal movement and small for gestational age b. Maternal diabetes mellitus and postmaturity c. Adolescent pregnancy and poor prenatal care d. History of preterm labor and intrauterine growth restriction

b Decreased fetal movement is an indicator for performing a contraction stress test; the size (small for gestational age) is not an indicator. Although adolescent pregnancy and poor prenatal care are risk factors for poor fetal outcomes, they are not indicators for performing a contraction stress test. Intrauterine growth restriction is an indicator; history of a previous stillbirth, not preterm labor, is another indicator.

Which laboratory marker is indicative of disseminated intravascular coagulation (DIC)? a. Bleeding time of 10 minutes b. Presence of fibrin split products c. Thrombocytopenia d. Hypofibrinogenemia

b Degradation of fibrin leads to the accumulation of multiple fibrin clots throughout the body's vasculature. Bleeding time in DIC is normal. Low platelets may occur but are not indicative of DIC because they may be the result from other coagulopathies. Hypofibrinogenemia occurs with DIC.

The nurse is circulating during a cesarean birth of a preterm infant. The obstetrician requests that cord clamping be delayed. What is the rationale for this directive? a. To reducethe risk for jaundice b. To reduce the risk of intraventricular hemorrhage c. To decrease total blood volume d. To improve the ability to fight infection

b Delayed cord clamping provides the greatest benefits to the preterm infant. These benefits include a significant reduction in intraventricular hemorrhage, a reduced need for a blood transfusion, and improved blood cell volume. The risk of jaundice can increase, requiring phototherapy. Although no difference in the newborn's infection fighting ability occurs, iron status is improved, which can provide benefits for 6 months.

The priority assessment in evaluating a pregnant woman with severe nausea and vomiting is: a. Fasting blood glucose level b. Ketonuria c. Bilirubin d. WBC count

b Determination of ketonuria would be a critical assessment that would lead towards determination of hyperemesis. A pregnant patient with severe nausea and vomiting may have hyperemesis gravidarum and as such requires critical monitoring to determine the nature of the problem. An FBS measurement, although informative, would not be the priority assessment at this time, nor would a bilirubin measurement. A WBC count would indicate the possibility of an infectious source but it would not be a priority assessment in terms of the patient's presentation.

A pregnant woman experiencing nausea and vomiting should: a. Drink a glass of water with a fat-free carbohydrate before getting out of bed in the morning b. Eat small, frequent meals (every 2 to 3 hours) c. Increase her intake of high-fat foods to keep the stomach full and coated d. Limit fluid intake throughout the day

b Eating small, frequent meals is a correct suggestion for a pregnant woman experiencing nausea and vomiting. She should avoid consuming fluids early in the day or when nauseated, but should compensate by drinking fluids at other times. She should also reduce her intake of fried foods and other fatty foods.

A woman is 8 months pregnant. She tells the nurse that she knows her baby listens to her, but her husband thinks she is imagining things. Which response by the nurse is most appropriate? a. "Many women imagine what their baby is like." b. "A baby in utero does respond to the mother's voice." c. "You'll need to ask the doctor if the baby can hear yet." d. "Thinking that your baby hears will help you bond with the baby."

b Fetuses respond to sound by 24 weeks. The fetus can be soothed by the sound of the mother's voice, and the nurse should instruct the mother so. Although statement A is accurate, it is not the most appropriate response. Statement D is not appropriate because it gives the mother impression that her baby cannot hear her and belittles her interpretation of her fetus's behaviors.

An Maternal Serum Alpha-Fetoprotein Screening (MSAFP) screening indicates an elevated level of alpha-fetoprotein. The test is repeated, and again the level is reported as higher than normal. What is the next step in the assessment sequence to determine thewell-being of the fetus? a. Percutaneous umbilical blood sampling (PUBS) b. Ultrasound c. Biophysical profile (BPP) d. Amniocentesis

b If MSAFP findings are abnormal, then follow-up procedures include genetic counseling for families with a history of NTD, repeated MSAFP screenings, an ultrasound examination, and possibly amniocentesis. Indications for the use of PUBS include prenatal diagnosis of inherited blood disorders, karyotyping of malformed fetuses, detection of fetal infection, determination of the acid-base status of fetuses with IUGR, and assessment and treatment of isoimmunization and thrombocytopenia in the fetus. A BPP is a method of assessing fetal well-being in the third trimester. Before an amniocentesis, the client would have an ultrasound for direct visualization of the fetus.

Which maternal condition always necessitates delivery by cesarean birth? a. Marginal placenta previa b. Complete placenta previa c. Ectopic pregnancy d. Eclampsia

b In complete placenta previa, the placenta completely covers the cervical os. A cesarean birth is the acceptable method of delivery. The risk of fetal death occurring is due to preterm birth. If the previa is marginal (i.e., 2 cm or greater away from the cervical os), then labor can be attempted. A cesarean birth is not indicated for an ectopic pregnancy. Labor can be safely induced if the eclampsia is under control.

Which of the following findings is not likely to be seen in a pregnant patient who has hypothyroidism? a. Miscarriage b. Macrosomia c. Gestational hypertension d. Placental abruption

b Infants born to mothers with hypothyroidism are more likely to be of low birth weight or preterm; these outcomes can be improved with early diagnosis and treatment. Hypothyroidism is often associated with both infertility and an increased risk of miscarriage. Pregnant women with hypothyroidism are more likely to experience both preeclampsia and gestational hypertension. Placental abruption and stillbirth are risks associated with hypothyroidism.

The nurse who is caring for a woman hospitalized for hyperemesis gravidarum would expect the initial treatment to involve what? a. Corticosteroids to reduce inflammation b. Intravenous (IV) therapy to correct fluid and electrolyte imbalances c. Antiemetic medication, such as pyridoxine, to control nausea and vomiting d. Enteral nutrition to correct nutritional deficits

b Initially, the woman who is unable to down clear liquids by mouth requires IV therapy to correct fluid and electrolyteimbalances. Corticosteroids have been successfully used to treat refractory hyperemesis gravidarum, but they are not the expected initial treatment for this disorder. Pyridoxine is vitamin B6, not an antiemetic medication. Promethazine, a common antiemetic, may be prescribed. In severe cases of hyperemesis gravidarum, enteral nutrition via a feeding tube may be necessary to correct maternal nutritional deprivation but is not the initial treatment for this client.

What is the nurse's initial action while caring for an infant with a slightly decreased temperature? a. Immediately notify the physician b. Place a cap on the infant's head, and have the mother perform kangaroo care c. Tell the mother that the infant must be kept in the nursery and observed for the next 4 hours d. Change the formula; a decreased body temperature is a sign of formula intolerance

b Keeping the head well covered with a cap prevents further heat loss from the head and placing the infant skin-to-skin against the mother should increase the infant's temperature.

Which client is at greatest risk for early postpartum hemorrhage (PPH)? a. Primiparous woman (G 2, P 1-0-0-1) being prepared for an emergency cesarean birth for fetal distress b. Woman with severe preeclampsia on magnesium sulfate whose labor is being induced c. Multiparous woman (G 3, P 2-0-0-2) with an 8-hour labor d. Primigravida in spontaneous labor with preterm twins

b Magnesium sulfate administration during labor poses a risk for PPH. Magnesium acts as a smooth muscle relaxant, thereby contributing to uterine relaxation and atony.

What is the correctterminology for an abortion in which the fetus dies but is retained within the uterus? a. Inevitable abortion b. Missed abortion c. Incomplete abortion d. Threatened abortion

b Missed abortion refers to the retention of a dead fetus in the uterus. An inevitable abortion means that the cervix is dilating with the contractions. An incomplete abortion means that not all of the products of conception were expelled. With a threatened abortion, the woman has cramping and bleeding but no cervical dilation.

In understanding and guiding a woman through her acceptance of pregnancy, a maternity nurse should be aware that: a. Nonacceptance of the pregnancy very often equates to rejection of the child b. Mood swings are most likely the result of worries about finances and a changed lifestyle, as well as profound hormonal changes c. Ambivalent feelings during pregnancy are usually seen only in emotionally immature or very young mothers d. Conflicts such as not wanting to be pregnant or childrearing and career-related decisions need not be addressed during pregnancy because they will resolve themselves naturally after birth

b Mood swings are natural and are likely to affect every woman to some degree. A woman may dislike being pregnant, refuse to accept it, and still love and accept the child. Ambivalent feelings about pregnancy are normal for mature or immature women, young or older. Conflicts about desire to perform childrearing and career-related concerns, however, need to be resolved; the baby's arrival ends the pregnancy but not all the issues.

A pregnant woman is being discharged from the hospital after the placement of a cervical cerclage because of a history of recurrent pregnancy loss, secondary to an incompetent cervix. Which information regarding postprocedural care should the nurse emphasize in the discharge teaching? a. Any vaginal discharge should be immediately reported to her health care provider b. The presence of any contractions, rupture of membranes (ROM), or severe perineal pressure should be reported c. The client will need to arrange for care at home, because her activity level will be restricted d. The client will be scheduled for a cesarean birth

b Nursing care should stress the importance of monitoring for the signs and symptoms of preterm labor. Vaginal bleeding needs to be reported to her primary health care provider. Bed rest is an element of care. However, the woman may stand for periods of up to 90 minutes, which allows her the freedom to see her physician. Home uterine activity monitoring may be used to limit the woman's need for visits and to monitor her status safely at home. The cerclage can be removed at 37 weeks of gestation (to prepare for a vaginal birth), or a cesarean birth can be planned.

A new mother with a thyroid disorder has come for a lactation follow-up appointment. Which thyroid disorder is a contraindication for breastfeeding? a.Hyperthyroidism b. Phenylketonuria (PKU) c. Hypothyroidism d. Thyroid storm

b PKU is a cause of mental retardation in infants; mothers with PKU pass on phenylalanine and therefore should elect not to breastfeed. A woman with either hyperthyroidism or hypothyroidism would have no reason not to breastfeed. A thyroid storm is a complication of hyperthyroidism and is not a contraindication to breastfeeding.

In contrast to placenta previa, what is the mostprevalent clinical manifestation of abruptio placentae? a. Bleeding b. Intense abdominal pain c. Uterine activity d. Cramping

b Pain is absent with placenta previa and may be agonizing with abruptio placentae. Bleeding may be present in varying degrees for both placental conditions. Uterine activity and cramping may be present with both placental conditions.

The nurse is planning the care for a laboring client with diabetes mellitus. This client is at greater risk for which clinical finding? a. Oligohydramnios b. Polyhydramnios c. Postterm pregnancy d. Chromosomal abnormalities

b Polyhydramnios or amniotic fluid in excess of 2000 ml is 10 times more likely to occur in the client with diabetes mellitus rather than in nondiabetic pregnancies. This complication places the mother at risk for premature rupture of membranes, premature labor, and postpartum hemorrhage. Prolonged rupture of membranes, IUGR, intrauterine fetal death, and renal agenesis (Potter syndrome) place the client at risk for developing oligohydramnios. Anencephaly, placental insufficiency, and perinatal hypoxia contribute to the risk for postterm pregnancy. Maternal age older than 35 years and balanced translocation (maternal and paternal) are risk factors for chromosomal abnormalities.

A nurse teaches a pregnant woman about the presumptive, probable, and positive signs of pregnancy. The woman demonstrates understanding of the nurse's instructions if she states that a positive sign of pregnancy is: a. A positive pregnancy test result b. Fetal movement palpated by the nurse-midwife c. Braxton Hicks contractions d. Quickening

b Positive signs of pregnancy are those that are attributed to the presence of a fetus, such as hearing the fetal heartbeat and palpating fetal movement. A positive pregnancy test result and Braxton Hicks contractions are probable signs of pregnancy. Quickening is a presumptive sign of pregnancy.

Preconception counseling is critical in the safe management of diabetic pregnancies. Which complication is commonly associated with poor glycemic control before and during early pregnancy? a. Frequent episodes of maternal hypoglycemia b. Miscarriage c. Hydramnios d. Hyperemesis gravidarum

b Preconception counseling is particularly important since strict metabolic control before conception and in the early weeks of gestation is instrumental in decreasing the risk of miscarriages. Frequent episodes of maternal hypoglycemia may occur during the first trimester (not before conception) as a result of hormonal changes and the effects on insulin production and use. Hydramnios occurs approximately 10 times more often in diabetic pregnancies than in nondiabetic pregnancies. Typically, it is observed inthe third trimester of pregnancy. Hyperemesis gravidarum may exacerbate hypoglycemic events because the decreased food intake by the mother and glucose transfer to the fetus contribute to hypoglycemia.

A nurse is reviewing care for pregnant women. Which clinical diagnosis would the nurse identify as being the most common medical complication of pregnancy? a. Hemorrhagic complications b. Hypertension c. Infections d. Hyperemesis gravidarum

b Preeclampsia and eclampsia are two noted, deadly forms of hypertension, which is the most common medical complication of pregnancy. A large percentage of pregnant women have nausea and vomiting, but a relative few have the severe form called hyperemesis gravidarum. Hemorrhagic complications are the second most common medical complication of pregnancy.

Which time span delineates the appropriate length for a normal pregnancy? a. 9 lunar months, 8.5 calendar months, 39 weeks, 272 days b. 10 lunar months, 9 calendar months, 40 weeks, 280 days c. 9 calendar months, 10 lunar months, 42 weeks, 294 days d. 9 calendar months, 38 weeks, 266 days

b Pregnancy lasts approximately 10 lunar months, 9 calendar months, 40 weeks, 280 days. Length of pregnancy is computed from the first day of the last menstrual period (LMP) until the day of birth. Nine lunar months is just short of a term pregnancy, and 294 days is longer than the average length of a pregnancy and would be considered postterm. Because conception occurs approximately 2 weeks after the first day of the LMP, the length described in D represents the postconception age of 266 days or 38 weeks. Postconception age is used in the discussion of fetal development.

A 41-week pregnant multigravida arrives at the labor and delivery unit after testing indicated that her fetus could be experiencing some difficulties in utero. Which diagnostic tool yields more detailed information about the condition of the fetus? a .Ultrasound for fetal anomalies b. Biophysical profile (BPP) c. MSAFP screening d. Percutaneous umbilical blood sampling (PUBS)

b Real-time ultrasound permits a detailed assessment of the physical and physiologic characteristics of the developing fetus and a cataloging of normal and abnormal biophysical responses to stimuli. The BPP is a noninvasive, dynamic assessment of a fetus that is based on acute and chronic markers of fetal disease. An ultrasound for fetal anomalies would most likely have occurred earlier in the pregnancy. It is too late in the pregnancy to perform an MSAFP. Furthermore, it does not provide information related to fetal well-being. Indications for PUBS include prenatal diagnosis or inherited blood disorders, karyotyping of malformed fetuses, detection of fetal infection, determination of the acid-base status of the fetus with IUGR, and assessment and treatment ofisoimmunization and thrombocytopenia in the fetus.

Signs of a threatened abortion (miscarriage) are noted in a woman at 8 weeks of gestation. Which intervention would the nurse identify as being appropriate for this type of abortion? a. Comfort the woman by telling her that if she loses this baby, she may attempt to get pregnant again in 1 month b. Prepare the woman for an ultrasound and blood work c. Put the woman on bed rest for at least 1 week and reevaluate d. Prepare the woman for a dilation and curettage (D&C)

b Repetitive transvaginal ultrasounds and measurement of human chorionic gonadotropin (hCG) and progesterone levels may be performed to determine whether the fetus is alive and within the uterus. Bed rest is recommended for 48 hours initially. D&C is not considered until signs of the progress to inevitable abortion are noted or the contents are expelled and incomplete. If the pregnancy is lost, the woman should be guided through the grieving process. Telling the client that she can get pregnant again soon is not a therapeutic response because it discounts the importance of this pregnancy.

Which information about variations in the infant's blood counts is important for the nurse to explain to the new parents? a. A somewhat lower-than-expected red blood cell count could be the result of a delay in clamping the umbilical cord b. An early high white blood cell (WBC) count is normal at birth and should rapidly decrease c. Platelet counts are higher in the newborn than in adults for the first few months d. Even a modest vitamin K deficiency means a problem with the blood's ability to properly clot

b The WBC count is normally high on the first day of birth and then rapidly declines. Delayed cord clamping results in an increase in hemoglobin and the red blood cell count. The platelet count is essentially the same for newborns and adults. Clotting is sufficient to prevent hemorrhage unless the deficiency of vitamin K is significant.

A nurse is reviewing the complication of HELLP syndrome. Which finding should the nurse be aware of? a. It can be diagnosed by a nurse alert to its symptoms b. Is characterized by hemolysis, elevated liver enzymes, and low platelets c. It is a mild form of preeclampsia d. Is associated with preterm labor but not perinatal mortality

b The acronym HELLP stands for hemolysis (H), elevated liver enzymes (EL), and low platelets (LP). HELLP syndrome is a variant of severe preeclampsia. It is difficult to identify, because the symptoms often are not obvious. It must be diagnosed in the laboratory. Preterm labor is greatly increased with HELLP syndrome, and so is perinatal mortality.

The nurse is aware that theinitiation of breastfeeding is most effective during the first 30 minutes after birth. What is the correct term for this phase of alertness? a. Transition period b. First period of reactivity c. Organizational stage d. Second period of reactivity

b The first period of reactivity is the first phase of transition and lasts up to 30 minutes after birth. The infant is highly alert during this phase. The transition period is the phase between intrauterine and extrauterine existence. An organizational stage is not a valid stage. The second period of reactivity occurs approximately between 4 and 8 hours after birth, after a period of sleep.

The nurse is completing a physical examination of the newborn 24 hours after birth. Which component of the evaluation is correct? a. The parents are excused to reduce their normal anxiety b. The nurse can gauge the neonate's maturity level by assessing his or her general appearance c. Once often neglected, blood pressure is now routinely checked d. When the nurse listens to the neonate's heart, the S1and S2 sounds can be heard; the S1 sound is somewhat higher in pitch and sharper than the S2 sound

b The nurse is looking at skin color, alertness, cry, head size, and other features. The parents' presence actively involves them in child care and gives the nurse the chance to observe their interactions. Blood pressure is not usually taken unless cardiac problems are suspected. The S2 sound is higher and sharper than the S1 sound.

A 26-year-old pregnant woman, gravida 2, para 1-0-0-1, is 28 weeks pregnant when she experiences bright red, painless vaginal bleeding. On her arrival at the hospital, which diagnostic procedure will the client most likely have performed? a. Amniocentesis for fetal lung maturity b. Transvaginal ultrasound for placental location c. Contraction stress test (CST) d. Internal fetal monitoring

b The presence of painless bleeding should always alert the health care team to the possibility of placenta previa, which can be confirmed through ultrasonography. Amniocentesis is not performed on a woman who is experiencing bleeding. In the event of an imminent delivery, the fetus is presumed to have immature lungs at this gestational age, and the mother is given corticosteroids to aid in fetal lung maturity. A CST is not performed at a preterm gestational age. Furthermore, bleeding is a contraindication toa CST. Internal fetal monitoring is also contraindicated in the presence of bleeding.

A 26-year-old primigravida has come to the clinic for her regular prenatal visit at 12 weeks. She appears thin and somewhat nervous. She reports thatshe eats a well-balanced diet, although her weight is 5 pounds less than it was at her last visit. The results of laboratory studies confirm that she has a hyperthyroid condition. Based on the available data, the nurse formulates a plan of care. Which nursing diagnosis is most appropriate for the client currently? a. Disrupted fluid balance b. Inadequate nutrition c. Excessive nutrition d. Disrupted sleep

b This client's clinical cues include weight loss, which supports a nursing diagnosis of "Inadequate nutrition." No clinical signs or symptoms support a nursing diagnosis of disrupted fluid balance. This client reports weight loss not weight gain. Although the client reports nervousness, the most appropriate nursing diagnosis, based on the client's other clinical symptoms, is "Inadequate nutrition."

Which statement about multifetal pregnancy is not accurate? a. The expectant mother often experiences anemia because the fetuses have a greater demand for iron b. Twin pregnancies come to term with the same frequency as single pregnancies c. The mother should be counseled to increase her nutritional intake and gain more weight d. Backache and varicose veins are often more pronounced

b Twin pregnancies often end in prematurity; serious efforts should be made to bring the pregnancy to term. A woman with a multifetal pregnancy often experiences anemia because of the increased demands of two fetuses; this issue should be monitored closely throughout her pregnancy. The client may need nutrition counseling to ensure that she gains more weight than what is needed for a singleton birth. The considerable uterine distention in multifetal pregnancy is likely to cause backache and leg varicosities; maternal support hose should be recommended.

In terms of the incidence and classification of diabetes, which information should the nurse keep in mind when evaluating clients during their ongoing prenatal appointments? a. Type 1 diabetes is most common b. Type 2 diabetes often goes undiagnosed c. Gestational diabetes mellitus (GDM) means that the woman will receive insulin treatment until 6 weeks after birth d. Type 1 diabetes may become type 2 during pregnancy

b Type 2 diabetes often goes undiagnosed because hyperglycemia gradually develops and is often not severe. Type 2, sometimes called adult-onset diabetes, is the most common type of diabetes. GDM refers to any degree of glucose intolerance first recognized during pregnancy; insulin may or may not be needed. People do not go back and forth between type 1 and type 2 diabetes.

Which clinical finding is a major use of ultrasonography in the first trimester? a. Amniotic fluid volume b. Presence of maternal abnormalities c. Placental location and maturity d. Cervical length

b Ultrasonography can detect certain uterine abnormalities such as bicornuate uterus, fibroids, and ovarian cysts. Amniotic fluid volume, placental location and maturity, and cervical length are not available via ultrasonography until the second or third trimester.

A woman at 39 weeks of gestation with a history of preeclampsia is admitted to the labor and birth unit. She suddenly experiences increased contraction frequency to every 1 to 2 minutes; dark red vaginal bleeding; and a tense, painful abdomen. What complication should the nurse suspect? a. Eclamptic seizure b. Placental abruption c. Placenta previa d. Rupture of the uterus

b Uterine tenderness in the presence of increasing tone may be the earliest finding of premature separation of the placenta (abruptio placentae or placental abruption). Women with hypertension are at increased risk for an abruption. Eclamptic seizures are evidenced by the presence of generalized tonic-clonic convulsions. Uterine rupture manifests with hypotonic uterine activity, signs of hypovolemia, and in many cases the absence of pain, and placenta previa with bright red, painless vaginal bleeding

Which cardiovascular changes cause the foramen ovale to close at birth? a. Increased pressure in the right atrium b. Increased pressure in the left atrium c. Decreased blood flow to the left ventricle d. Changes in the hepatic blood flow

b With the increase in the blood flow to the left atrium from the lungs, the pressure is increased, and the foramen ovale is functionally closed. The pressure in the right atrium decreases at birth and is higher during fetal life. Blood flow increases to the left ventricle after birth. The hepatic blood flow changes but is not the reason for the closure of the foramen ovale.

A woman who is at 36 weeks of gestation is having a nonstress test. Which statement indicates her correct understanding of the test? a. "I will need to have a full bladder for the test to be done accurately." b. "I should have my husband drive me home after the test because I may be nauseated." c. "This test will help to determine whether the baby has Down syndrome or a neural tube defect." d. "This test observes for fetal activity and an acceleration of the fetal heart rate to determine the well-being of the baby."

b, c Ultrasound is used prior to the procedure as a visualization aid to assist with insertion of transabdominal needle. There is no need to assess the urine for bleeding as this is not considered to be a typical presentation or complication.

In reviewing the history of a woman who wants to become pregnant, which medication profile would indicate a potential concern relative to toxic exposure? Select all that apply a. Tylenol OTC occasionally for a headache; twice last week b. Anticonvulsant for seizure disorder c. Lithium for bipolar disorder d. Coumadin for atrial fibrillation e. Multivitamins once a day

b, c, d A patient being treated with an anticonvulsant or lithium is at risk for toxic effects during pregnancy. Warfarin (Coumadin) can put a patient at risk during pregnancy. Although acetaminophen (Tylenol) can have toxic effects on the liver, the reported frequency is not a concern at this time. Taking multivitamins is a healthy recommended option.

When would the best timeframe be to establish gestational age based on ultrasound? a. At term b. 8 weeks c. Between 14 and 22 weeks d. 36 weeks

b, c, d The fetal alarm signal is reached when no fetal movements are noted for a period of 12 hours.

A nurse is working with a diabetic patient who recently found out she is pregnant. In coordinating an interdisciplinary team to help manage the patient throughout the pregnancy, the nurse would include: select all that apply a. Family practice physician b. Dietician c. Perinatologist d. Occupational therapist e. Nephrologist f. Speech therapist

b, c, e An internal medicine practitioner rather than family practice physician would be included on the interdisciplinary care team. A dietician would be included to help the patient with dietary planning, a perinatologist to take care of the maternal-fetal unit, and a nephrologist to monitor renal function. There is no need for an occupational therapist or a speech therapist unless other issues arise.

A pregnant patient is experiencing some integumentary changes and is concerned that they may represent abnormal findings. The nurse provides information to the patient that the following findings would be considered "normal abnormal" findings during pregnancy so that she should not be alarmed. Select all that apply. a. Facial edema b. Melasma c. Linea nigra d. Superficial thrombophlebitis e. Vascular spiders f. Allodynia

b, c, e Facial edema is a concern because it can represent toxemia of pregnancy. Superficial thrombophlebitis is a concern because it can represent a risk factor for development of a DVT during pregnancy. The presentation of allodynia (pain upon normal touch) is considered to be a significant finding and requires additional investigation. Melasma (also known as the mask of pregnancy or chloasma), linea nigra (a hyperpigmentation line extending from the fundus to the symphysis pubis), and the presence of vascular spiders are all considered to be normal abnormal findings in pregnancy.

A pregnant patient experiences thyroid storm following delivery of her infant. What interventions would the nurse anticipate to be ordered by the physician? a. Restriction of intravenous fluids to prevent fluid overload b. Administration of O2 c. Antipyretics d. Synthroid e. PTU

b, c, e Oxygen would be provided, antipyretics would be given to reduce fever, and PTU would be administered. IV fluids would be administered to the patient in order to reverse the hypotension that the patient would be experiencing. Synthroid would not be given because it is used to treat hypothyroidism, and with thyroid storm, the patient is suffering from hyperthyroidism.

Which findings could be considered to be a barrier to a pregnant woman seeking prenatal care? Select all that apply. a. Patient would prefer to be cared for by a midwife instead of a physician. b. Economic cost of health care. c. Patient's cultural beliefs do not include prenatal care as being valued. d. Patient speaks several languages. e. Patient had a bad experience the last time she went to a doctor for care.

b, c, e Economic factors can delay the onset of health care treatment. A patient's cultural beliefs and values may be a barrier to seeking prenatal care if her culture does not perceive any inherent value in prenatal care. If the patient had a bad prior experience with a health care provider, it may be a barrier to seeking future care. The fact that this patient is multilingual does not necessarily represent a barrier to seeking prenatal care. Although the patient may prefer to be cared for by a midwife, this fact cannot be considered a barrier to seeking prenatal care because it demonstrates a patient's choice.

A newborn in the neonatal intensive care unit (NICU) is dying of a massive infection. The parents speak to the neonatologist, who informs them of their son's prognosis. When the father sees his son, he says, "He looks just fine to me. I can't understand what all this is about." What is the most appropriate response or reaction by the nurse at this time? a. "Didn't the physician tell you about your son's problems?" b. "This must be a difficult time for you. Tell me how you're doing." c. Quietly stand beside the infant's father. d. "You'll have to face up to the fact that he is going to die sooner or later."

b. "This must be a difficult time for you. Tell me how you're doing." The phase of intense grief can be very difficult, especially for fathers. Parents should be encouraged to share their feelings during the initial steps in the grieving process. This father is in a phase of acute distress and is reaching out to the nurse as a source of direction in his grieving process. Shifting the focus is not in the best interest of the parent. Nursing actions may help the parents actualize the loss of their infant through a sharing and verbalization of their feelings of grief. Telling the father that his son is going to die sooner or later is dispassionate and an inappropriate statement on the part of the nurse.

13. A client gives birth to a stillborn infant. At first, she appears stunned by the news, cries a little, and then asks the nurse to call her mother. What is the proper term for the phase of bereavement that this client is experiencing? a. Anticipatory grief b. Acute distress c. Intense grief d. Reorganization

b. Acute distress. The immediate reaction to news of a perinatal loss or infant death encompasses a period of acute distress. Disbelief and denial can occur. However, parents also feel very sad and depressed. Intense outbursts of emotion and crying are normal. However, a lack of affect, euphoria, and calmness may occur and may reflect numbness, denial, or personal ways of coping with stress. Anticipatory grief applies to the grief related to a potential loss of an infant. The parent grieves in preparation of the infant's possible death, although he or she clings to the hope that the child will survive. Intense grief occurs in the first few months after the death of the infant. This phase encompasses many different emotions, including loneliness, emptiness, yearning, guilt, anger, and fear. Reorganization occurs after a long and intense search for meaning. Parents are better able to function at work and home, experience a return of self-esteem and confidence, can cope with new challenges, and have placed the loss in perspective.

What information regarding a fractured clavicle is most important for the nurse to take into consideration when planning the infant's care? a. Prone positioning facilitates bone alignment. b. No special treatment is necessary. c. Parents should be taught range-of-motion exercises. d. The shoulder should be immobilized with a splint.

b. No special treatment is necessary. Fractures in newborns generally heal rapidly. Except for gentle handling, no accepted treatment for a fractured clavicle exists. Movement should be limited, and the infant should be gently handled. Performing range-of-motion exercises on the infant is not necessary. A fractured clavicle does not require immobilization with a splint.

During a follow-up home visit, the nurse plans to evaluate whether parents have progressed to the second stage of grieving (phase of intense grief). Which behavior would the nurse not anticipate finding? a. Guilt, particularly in the mother b. Numbness or lack of response c. Bitterness or irritability d. Fear and anxiety, especially about getting pregnant again

b. Numbness or lack of response. The second phase of grieving encompasses a wide range of intense emotions, including guilt, anger, bitterness, fear, and anxiety. What the nurse would hope not to see is numbness or unresponsiveness, which indicates that the parents are still in denial or shock.

The nurse is performing a gestational age and physical assessment on the newborn. The infant appears to have an excessive amount of saliva. This clinical finding may be indicative of what? a. Excessive saliva is a normal finding in the newborn b. Excessive saliva in a neonate indicates that the infant is hungry c. It may indicate that the infant has a tracheoesophageal fistula or esophageal atresia d. Excessive saliva may indicate that the infant has a diaphragmatic hernia

c

Parents have asked the nurse about organ donation after that infant's death. Which information regarding organ donation is important for the nurse to understand? a. Federal law requires the medical staff to ask the parents about organ donation and then to contact their state's organ procurement organization (OPO) to handle the procedure if the parents agree. b. Organ donation can aid grieving by giving the family an opportunity to see something positive about the experience. c. Most common donation is the infant's kidneys. d. Corneas can be donated if the infant was either stillborn or alive as long as the pregnancy went full term.

b. Organ donation can aid grieving by giving the family an opportunity to see something positive about the experience. Evidence indicates that organ donation can promote healing among the surviving family members. The federal Gift of Life Act made state OPOs responsible for deciding whether to request a donation and for making that request. The most common donation is the cornea. For cornea donation, the infant must have been born alive at 36 weeks of gestation or later.

Which finding would indicate to the nurse that the grieving parents have progressed to the reorganization phase of grieving? a. The parents say that they "feel no pain." b. The parents are discussing sex and a future pregnancy c. The parents have abandoned those moments of "bittersweet grief." d. The parents' questions have progressed from "Why?" to "Why us?"

b. The parents are discussing sex and a future pregnancy. Many couples have conflicting feelings about sexuality and future pregnancies. A little painis always present, certainly beyond the first year when recovery begins to peak. Bittersweet grief describes the brief grief response that occurs with reminders of a loss, such as anniversary dates. Most couples never abandon these reminders. Recovery is ongoing. Typically, a couple's search for meaning progresses from "Why?" in the acute phase to "Why me?" in the intense phase to "What does this loss mean to my life?" in the reorganizational phase.

The nurse should be cognizant of which condition related to skeletal injuries sustained by a neonate during labor or childbirth? a. Newborn's skull is still forming and fractures easily. b. Unless a blood vessel is involved, linear skull fractures heal without special treatment. c. Clavicle fractures often need to be set with an inserted pin for stability. d. Other than the skull, the most common skeletal injuries are to leg bones.

b. Unless a blood vessel is involved, linear skull fractures heal without special treatment. Approximately 70% of neonatal skull fractures are linear. Because the newborn skull is flexible, considerable force is required to fracture it. Clavicle fractures need no special treatment. The clavicle is the bone most often fractured during birth.

A client arrives for her initial prenatal examination. This is her first child. She asks the nurse, "How does my baby get air inside my uterus?" What is the correctresponse by the nurse? a. "The baby's lungs work in utero to exchange oxygen and carbon dioxide." b. "The baby absorbs oxygen from your blood system." c. "The placenta provides oxygen to the baby and excretes carbon dioxide into your bloodstream." d. "The placenta delivers oxygen-rich blood through the umbilical artery to the baby's abdomen."

c

A newborn is jaundiced and is receiving phototherapy via ultraviolet bank lights. What is the mostappropriate nursing intervention when caring for an infant with hyperbilirubinemia and receiving phototherapy? a. Applying an oil-based lotion to the newborn's skin to prevent dying and cracking b. Limiting the newborn's intake of milk to prevent nausea, vomiting, and diarrhea c. Placing eye shields over the newborn's closed eyes d. Changing the newborn's position every 4 hours

c

A nurse is assessing a newborn girl who is 2 hours old. Which finding warrants a call to the health care provider? a. Blood glucose of 45 mg/dl using a Dextrostix screening method b. Heart rate of 160 beats per minute after vigorously crying c. Laceration of the cheek d. Passage of a dark black-green substance from the rectum

c

A pregnant woman at 25 weeks of gestation tells the nurse that she dropped a pan last week and her baby jumped at the noise. Which response by the nurse is most accurate? a. "That must have been a coincidence; babies can't respond like that." b. "The fetus is demonstrating the aural reflex." c. "Babies respond to sound starting at approximately 24 weeks of gestation." d. "Let me know if it happens again; we need to report that to your midwife."

c

A woman who has recently given birth reports pain and tenderness in her leg. On physical examination, the nurse notices warmth and redness over an enlarged, hardened area. Which condition should the nurse suspect, and how will it be confirmed? a. Disseminated intravascular coagulation (DIC); asking for laboratory tests b. von Willebrand disease (vWD); noting whether bleeding times have been extended c. Thrombophlebitis; using real-time and color Doppler ultrasound d. Idiopathic or immune thrombocytopenic purpura (ITP); drawing blood for laboratory analysis

c

A woman's cousin gave birth to an infant with a congenital heart anomaly. The woman asks the nurse when such anomalies occur during development. Which response by the nurse is most accurate? a. "We don't really know when such defects occur." b. "It depends on what caused the defect." c. "Defects occur between the third and fifth weeks of development." d. "They usually occur in the first 2 weeks of development."

c

An infant boy was delivered minutes ago. The nurse is conducting the initial assessment. Part of the assessment includes the Apgar score. When should the Apgar assessment be performed? a. Only if the newborn is in obvious distress b. Once by the obstetrician, just after the birth c. At least twice, 1 minute and 5 minutes after birth d. Every 15 minutes during the newborn's first hour after birth

c

As the nurse assists a new mother with breastfeeding, the client asks, "If formula is prepared to meet the nutritional needs of the newborn, what is in breast milk that makes it better?" What is the nurse's best response? a. More calories b. Essential amino acids c. Important immunoglobulins d. More calcium

c

The nurse is providing education to a client regarding the normal changes of the breasts during pregnancy. Which statement regarding these changes is correct? a. The visibility of blood vessels that form an intertwining blue network indicates a possible infection of the tubercles b. The mammary glands do not develop until 2 weeks before labor c. Lactation is inhibited until the progesterone level declines after birth d. Colostrum is the yellowish oily substance used to lubricate the nipples for breastfeeding

c

What is the rationale for the administration of vitamin K to the healthy full-term newborn? a. Most mothers have a diet deficient in vitamin K, which results in the infant being deficient b. Vitamin K prevents the synthesis of prothrombin in the liver and must be administered by injection c. Bacteria that synthesize vitamin K are not present in the newborn's intestinal tract d. The supply of vitamin K in the healthy full-term newborn is inadequate for at least 3 to 4 months and must be supplemented

c

When a woman is diagnosed with postpartum depression (PPD) withpsychotic features, what is the nurse's primary concern in planning the client's care? a. Displaying outbursts of anger b. Neglecting her hygiene c. Harming her infant d. Losing interest in her husband

c

Which action by the mother will initiate the milk ejection reflex (MER)? a. Wearing a firm-fitting bra b. Drinking plenty of fluids c. Placing the infant to the breast d. Applying cool packs to her breast

c

Which clinical finding in a primiparous client at 32 weeks of gestation might be an indication of anemia? a. Ptyalism b. Pyrosis c. Pica d. Decreased peristalsis

c

Which information should the nurse provide to a breastfeeding mother regarding optimal self-care? a. She will need an extra 1000 calories a day to maintain energy and produce milk b. She can return to prepregnancy consumption patterns of any drinks as long as she gets enough calcium c. She should avoid trying to lose large amounts of weight d. She must avoid exercising because it is too fatiguing

c

Which is the initial treatment for the client diagnosed with von Willebrand disease (vWD) who experiences a postpartum hemorrhage (PPH)? a. Cryoprecipitate b. Factor VIII and von Willebrand factor (vWf) c. Desmopressin d. Hemabate

c

Which is the most accurate description of postpartum depression (PPD) without psychotic features? a. Postpartum baby blues requiring the woman to visit with a counselor or psychologist b. Condition that is more common among older Caucasian women because they have higher expectations c. Distinguishable by pervasive sadness along with mood swings d. Condition that disappears without outside help

c

Which statement accuratelydescribes an appropriate-for-gestational age (AGA) weight assessment? a. AGA weight assessment falls between the 25th and 75th percentiles for the infant's age b. AGA weight assessment depends on the infant's length and the size of the newborn's head c. AGA weight assessment falls between the 10th and 90th percentiles for the infant's age d. AGA weight assessment is modified to consider intrauterine growth restriction (IUGR)

c

Which statement regarding the development of the respiratory system is a high priority for the nurse to understand? a. The respiratory system does not begin developing until after the embryonic stage b. The infant's lungs are considered mature when the L/S ratio is 1:1, at approximately 32 weeksof gestation c. Maternal hypertension can reduce maternal-placental blood flow, accelerating lung maturity d. Fetal respiratory movements are not visible on ultrasound scans until at least 16 weeks of gestation

c

Which substance used during pregnancy causes vasoconstriction and decreased placental perfusion, resulting in maternal and neonatal complications? a. Alcohol b. Caffeine c. Tobacco d. Chocolate

c

While examining a newborn, the nurse notes uneven skinfolds on the buttocks and a clunk when performing the Ortolani maneuver. These findings are likely indicative of what? a. Polydactyly b. Clubfoot c. Hip dysplasia d. Webbing

c

While providing care to the maternity client, the nurse should be aware that one of these anxiety disorders is likely to be triggered by the process of labor and birth. Which disorder fits this criterion? a. Phobias b. Panic disorder c. Post traumatic stress disorder (PTSD) d. Obsessive-compulsive disorder (OCD)

c

Which hematocrit (HCT) and hemoglobin (HGB) results represent the lowest acceptable values for a woman in the third trimester of pregnancy? a. 38% HCT; 14 g/dL HGB b. 35% HCT; 13 g/dL HGB c. 33% HCT; 11 g/dL HGB d. 32% HCT; 10.5 g/dL HGB

c 38% HCT; 14 g/dL HGB and 35% HCT; 13 g/dL HGB are within normal limits in a nonpregnant woman. 33% HCT; 11 g/dL HGB represents the lowest acceptable values during the first and the third trimesters, and 32% HCT; 10.5 g/dl HGB represents the lowest acceptable values for the second trimester, when the hemodilution effect of blood volume expansion is at its peak.

In the past, factors to determine whether a woman was likely to develop a high-risk pregnancy were primarily evaluated from a medical point of view. A broader, more comprehensive approach to high-risk pregnancy has been adopted today. Four categories have now been established, based on the threats to the health of the woman and the outcome of pregnancy. Which category should notbe included in this group? a. Biophysical b. Psychosocial c. Geographic d. Environmental

c A geographic category is correctly referred to as sociodemographicrisk. These factors stem from the mother and her family. Ethnicity may be one of the risks to pregnancy; however, it is not the only factor in this category. Low income, lack of prenatal care, age, parity, and marital status also are included. Biophysical is one of the broad categories used for determining risk. These include genetic considerations, nutritional status, and medical and obstetric disorders. Psychosocial risks include smoking, caffeine, drugs, alcohol, and psychologic status. All of these adverse lifestyles can have a negative effect on the health of the mother or fetus. Environmental risks are risks that can affect both fertility and fetal development. These include infections, chemicals, radiation, pesticides, illicit drugs, and industrial pollutants.

Which preexisting factor is known to increase the risk of gestational diabetes mellitus (GDM)? a. Underweight before pregnancy b. Maternal age younger than 25 years c. Previous birth of large infant d. Previous diagnosis of type 2 diabetes mellitus

c A previous birth of a large infant suggests GDM. Obesity (body mass index [BMI] of 30 or greater) creates a higher risk for gestational diabetes. A woman younger than 25 years is not generally at risk for GDM. The person with type 2 diabetes mellitus already has diabetes and thus will continue to have it after pregnancy. Insulin may be required during pregnancy because oral hypoglycemia drugs are contraindicated during pregnancy.

A woman arrives at the emergency department with reports of bleeding and cramping. The initial nursing history is significant for a last menstrual period 6 weeks ago. On sterile speculum examination, the primary care provider finds that the cervix is closed. The anticipated plan of care for this woman would be based on a probable diagnosis of which type of spontaneous abortion? a. Incomplete b. Inevitable c. Threatened d. Septic

c A woman with a threatened abortion has spotting, mild cramps, and no cervical dilation. A woman with an incomplete abortion would have heavy bleeding, mild-to-severe cramping, and cervical dilation. An inevitable abortion demonstrates the same symptoms as an incomplete abortion: heavy bleeding, mild-to-severe cramping, and cervical dilation. A woman with a septic abortion has malodorous bleeding and typically a dilated cervix.

A woman is 6 weeks pregnant. She has had a previous spontaneous abortion at 14 weeks of gestation and a pregnancy that ended at 38 weeks with the birth of a stillborn girl. What is her gravidity and parity according to the GTPAL system? a. 2-0-0-1-1 b. 2-1-0-1-0 c. 3-1-0-1-0 d. 3-0-1-1-0

c According to the GPTAL system, this woman's gravidity and parity information is calculated as follows:G: Total number of times the woman has been pregnant (she is pregnant for the third time) T: Number of pregnancies carried to term (only one pregnancy resulted in a fetus at term)P: Number of pregnancies that resulted in a preterm birth (none)A: Abortions or miscarriages before the period of viability (she has had one)L: Number of children born who are currently living (she has no living children)3-1-0-1-0 is the correct calculation of this woman's gravidity and parity.

While discussing the societal impacts of breastfeeding, the nurse should be cognizant of the benefits and educate the client accordingly. Which statement as part of this discussion would be incorrect? a. Breastfeeding requires fewer supplies and less cumbersome equipment b. Breastfeeding saves families money c. Breastfeeding costs employers in terms of time lost from work d. Breastfeeding benefits the environment

c Actually, less time is lost to work by breastfeeding mothers, in part because infants are healthier.

At 35 weeks of pregnancy, a woman experiences preterm labor. Although tocolytic medications are administered and she is placed on bed rest, she continues to experience regular uterine contractions and her cervix is beginning to dilate and efface. What is an important test for fetal well-being at this time? a. Percutaneous umbilical blood sampling (PUBS) b. Ultrasound for fetal size c. Amniocentesis for fetal lung maturity d. Non stress test (NST)

c Amniocentesis is performed to assess fetal lung maturity in the event of a preterm birth. The fluid is examined to determine the lecithin to sphingomyelin (L/S) ratio. Indications for PUBS include prenatal diagnosis or inherited blood disorders, karyotyping of malformed fetuses, detection of fetal infection, determination of the acid-base status of the fetus with IUGR, and assessment and treatment of isoimmunization and thrombocytopenia in the fetus. Determination of fetal size by ultrasound is typically performed during the second trimester and is not indicated in this scenario. An NST measures the fetal response to fetal movement in a noncontracting mother.

A woman with worsening preeclampsia is admitted to the hospital's labor and birth unit. The physician explains the plan of care for severe preeclampsia, including the induction of labor, to the woman and her partner. Which statement by the partner leads the nurse to believe that the couple needs further information? a. "I will help her use the breathing techniques that we learned in our childbirth classes." b. "I will give her ice chips to eat during labor." c. "Since we will be here for a while, I'll ask my mother, to bring our toddler to visit." d. "I will stay with her during her labor, just as we planned."

c Arranging a visit with their toddler indicates that the partner does not understand the importance of the quiet, subdued environment that is needed to prevent this condition from worsening. Implementing breathing techniques is indicative of adequate knowledge related to pain management during labor. Administering ice chips indicates an understanding of nutritional needs during labor. Staying with his partner during labor demonstrates the husband's support and is appropriate.

A client in the third trimester has just undergone an amniocentesis to determinefetal lung maturity. Which statement regarding this testing is important for the nurse in formulating a care plan? a. Because of new imaging techniques, an amniocentesis should have been performed in the first trimester b. Despite the use of ultrasonography, complications still occur in the mother or infant in 5% to 10% of cases c. Administration of Rho(D) immunoglobulin may be necessary d. The presence of meconium in the amniotic fluid is always a cause for concern

c As a result of the possibility of fetomaternal hemorrhage, administration of Rho(D) immunoglobulin is the standard of practice after amniocentesis for women who are Rh negative. Amniocentesis is possible after the 14th week of pregnancy when the uterus becomes an abdominal organ. Complications occur in less than 1% of cases; many have been minimized or eliminated through the use of ultrasonography. Meconium in the amniotic fluid before the beginning of labor is not usually a problem.

The nurse should be cognizant of which important information regarding the gastrointestinal (GI) system of the newborn? a. The newborn's cheeks are full because of normal fluid retention b. The nipple of the bottle or breast must be placed well inside the baby's mouth because teeth have been developing in utero, and one or more may even be through c. Regurgitation during the first day or two can be reduced by burping the infant and slightlyelevating the baby's head d. Bacteria are already present in the infant's GI tract at birth because they traveled through the placenta

c Avoiding overfeeding can also reduce regurgitation. The newborn's cheeks are full because of well-developedsucking pads. Teeth do develop in utero, but the nipple is placed deep because the baby cannot move food from the lips to the pharynx. Bacteria are not present at birth, but they soon enter through various orifices.

A pregnant woman at 21 weeks of gestation has an elevated blood pressure of 140/98. Past medical history reveals that the woman has been treated for hypertension. On the basis of this information, the nurse would classify this client as having: a. Gestational hypertension b. Chronic hypertension c. Superimposed preeclampsia d. Preeclampsia

c Because this client already has a medical history of hypertension and is now exhibiting hypertension after the 20th week of gestation, she would be considered to have superimposed pre-eclampsia. Pre-eclampsia would be the classification in a client without a history of hypertension who was hypertensive following the 20th week of pregnancy. Gestational hypertension occurs after the 20th week of pregnancy in a client who was previously normotensive. Even though the client has chronic hypertension, the fact that she is now pregnant determines that she would be classified as having superimposed pre-eclampsia.

A pregnant woman has maternal phenylketonuria (PKU) and is interested in whether or not she will be able to breastfeed her baby. Which reaction by the nurse indicates accurate information? a. The patient can breastfeed the baby as long as she continues to maintain a PKU-restricted diet. b. The patient should alternate breastfeeding with bottle feeding in order to reduce PKU levels provided to the baby. c. The patient should be advised to not breastfeed the infant because her breast milk will contain large amounts of phenylalanine. d. The patient can breastfeed for the first 3 months without any untoward effects on the infant.

c Breastfeeding is not advised for a patient who has maternal PKU, because phenylalanine levels are high in such a patient's breast milk. Dietary restriction will not limit the amount of this substance in breast milk. Alternating feeding sources is not advised either.

What condition indicates concealed hemorrhage when the client experiences abruptio placentae? a. Decrease in abdominal pain b. Bradycardia c. Hard, board like abdomen d. Decrease in fundal height

c Concealed hemorrhage occurs when the edges of the placenta do not separate. The formation of a hematoma behind the placenta and subsequent infiltration of the blood into the uterine muscle results in a very firm, boardlike abdomen. Abdominal pain may increase. The client will have shock symptoms that include tachycardia. As bleeding occurs, the fundal height increases.

A woman arrives for evaluation of signs and symptoms that include a missed period, adnexal fullness, tenderness, and dark red vaginal bleeding. On examination, the nurse notices an ecchymotic blueness around the woman's umbilicus. What does this finding indicate? a. Normal integumentary changes associated with pregnancy b. Turner sign associated with appendicitis c. Cullen sign associated with a ruptured ectopic pregnancy d. Chadwick sign associated with early pregnancy

c Cullen sign, the blue ecchymosis observed in the umbilical area, indicates hematoperitoneum associated with an undiagnosed ruptured intraabdominal ectopic pregnancy. Linea nigra on the abdomen is the normal integumentary change associated with pregnancy and exhibits a brown pigmented, vertical line on the lower abdomen. Turner sign is ecchymosis in the flank area, often associated with pancreatitis. A Chadwick sign is a blue-purple cervix that may be seen during or around the eighth week of pregnancy.

During a prenatal visit, the nurse is explaining dietary management to a woman diagnosed with pre-gestational diabetes. Which statement by the client reassures the nurse that teaching has been effective? a. "I will need to eat 600 more calories per day because I am pregnant." b. "I can continue with the same diet as before pregnancy as long as it is well balanced." c. "Diet and insulin needs change during pregnancy." d. "I will plan my diet based on the results of urine glucose testing."

c Diet and insulin needs change during the pregnancy in direct correlation to hormonal changes and energy needs. In the third trimester, insulin needs may double or even quadruple. The diet is individualized to allow for increased fetal and metabolic requirements, with consideration of such factors as prepregnancy weight and dietary habits, overall health, ethnic background, lifestyle, stage of pregnancy, knowledge of nutrition, and insulin therapy. Energy needs are usually calculated on the basis of 30 to 35calories per kilogram of ideal body weight. Dietary management during a diabetic pregnancy must be based on blood, not urine, glucose changes.

Which important component of nutritional counseling should the nurse include in health teaching for a pregnant woman who is experiencing cholecystitis? a. Assess the woman's dietary history for adequate calories and proteins b. Teach the woman that the bulk of calories should come from proteins c. Instruct the woman to eat a low-fat diet and to avoid fried foods d. Instruct the woman to eat a low-cholesterol, low-salt diet

c Eating a low-fat diet and avoiding fried foods is appropriate nutritional counseling for this client. Caloric and protein intake do not predispose a woman to the development of cholecystitis. The woman should be instructed to limit protein intake and choose foods that are high in carbohydrates. A low-cholesterol diet may be the result of limiting fats. However, a low-salt diet is not indicated.

While assessing the integument of a 24-hour-old newborn, the nurse notes a pink papular rash with vesicles superimposed on the thorax, back, and abdomen. What action is the highest priority for the nurse to take immediately? a. Immediately notify the health care professional. b. Move the newborn to an isolation nursery. c. Document the finding as erythema toxicum neonatorum. d. Take the newborn's temperature and obtain a culture of one of the vesicles.

c Erythema toxicum neonatorum (or erythema neonatorum) is a newborn rash that resembles flea bites. Documentation of the condition is the priority. Notification of the health care provider, isolation of the newborn, or additional interventions are not necessary when erythema toxicum neonatorum is present.

A pregnant woman demonstrates understanding of the nurse's instructions regarding relief of leg cramps if she: a. Wiggles and points her toes during the cramp b. Applies cold compresses to the affected leg c. Extends her leg and dorsiflexes her foot during the cramp d. Avoids weight bearing on the affected leg during the cramp

c Extending the leg and dorsiflexing the foot are the appropriate relief measure for a leg cramp. Pointing the toes can aggravate rather than relieve the cramp. Application of heat is recommended. Bearing weight on the affected leg can help relieve the leg cramp, so it should not be avoided.

Which presumptive sign (felt by woman) or probable sign (observed by the examiner) of pregnancy is not matched with another possible cause(s)? a. Amenorrhea—stress, endocrine problems b. Quickening—gas, peristalsis c. Goodell sign—cervical polyps d. Chadwick sign—pelvic congestion

c Goodell sign might be the result of pelvic congestion, not polyps. Amenorrhea sometimes can be caused by stress, vigorous exercise, early menopause, or endocrine problems. Quickening can be gas or peristalsis. Chadwick sign might be the result of pelvic congestion.

The labor of a pregnant woman with preeclampsia is going to be induced. Before initiating the oxytocin infusion, the nurse reviews the woman's latest laboratory test findings, which reveal a platelet count of 90,000 mm3, an elevated aspartate aminotransaminase (AST) level, and a falling hematocrit. The laboratory results are indicative of which condition? a. Eclampsia b. Disseminated intravascular coagulation (DIC) syndrome c. Hemolysis, elevated liver enzyme levels, and low platelet levels (HELLP) syndrome d. Idiopathic thrombocytopenia

c HELLP syndrome is a laboratory diagnosis for a variant of severe preeclampsia that involves hepatic dysfunction characterized by hemolysis (H), elevated liver (EL) enzymes, and low platelets (LP). Eclampsia is determined by the presence of seizures. DIC isa potential complication associated with HELLP syndrome. Idiopathic thrombocytopenia is the presence of low platelets of unknown cause and is not associated with preeclampsia.

To manage her diabetes appropriately and to ensure a good fetal outcome, how would the pregnant woman with diabetes alter her diet? a. Eat six small equal meals per day b. Reduce the carbohydrates in her diet c. Eat her meals and snacks on a fixed schedule d. Increase her consumption of protein.

c Having a fixed meal schedule will provide the woman and the fetus with a steady blood sugar level, provide a good balance with insulin administration, and help prevent complications. Having a fixed meal schedule is more important than the equal division of food intake. Approximately 45% of the food eaten should be in the form of carbohydrates.

Which statement regarding the laboratory test for glycosylated hemoglobin Alcis correct? a. The laboratory test for glycosylated hemoglobin Alcis performed for all pregnant women, not only those with or likely to have diabetes b. This laboratory test is a snapshot of glucose control at the moment c. This laboratory test measures the levels of hemoglobin Alc, which should remain at less than 7% d. This laboratory test is performed on the woman's urine, not her blood

c Hemoglobin Alclevels greater than 7% indicate an elevated glucose level during the previous 4 to 6 weeks. This extra laboratory test is for diabetic women and defines glycemic control over the previous 4 to 6 weeks. Glycosylated hemoglobin level tests are performed on the blood.

A woman diagnosed with marginal placenta previa gave birth vaginally 15 minutes ago. Which complication should the nurse anticipate as the being the greatest risk for thie client? a. Thrombophlebitis b. Infection c. Hemorrhage d. Urinary retention

c Hemorrhage is the most immediate risk because the lower uterine segment has limited ability to contract to reduce blood loss. Infection is a risk because of the location of the placental attachment site; however, it is not a priority concern at this time. Placenta previa poses no greater risk for urinary retention or thrombophlebitis than does a normally implanted placenta.

Which statement regarding the nutrient needs of breastfed infants is correct? a. Breastfed infants need extra water in hot climates b. During the first 3 months, breastfed infants consume more energy than formula-fed infants c. Breastfeeding infants should receive 400 IU of oral vitamin D drops daily d. Vitamin K injections at birth are not necessary for breastfed infants

c Human milk contains only small amounts of vitamin D. All infants who are breastfed should receive 400 International Units of vitamin D each day.

A woman with severe preeclampsia has been receiving magnesium sulfate by IV infusion for 8 hours. The nurse assesses the woman and documents the following findings: temperature 37.1° C, pulse rate 96 beats/min, respiratory rate 24 breaths/min, blood pressure 155/112 mm Hg, 3+ deep tendon reflexes, and no ankle clonus. Which physician order should the nurse anticipate? a. Diazepam b. Calcium gluconate c. Hydralazine d. Magnesium sulfate bolus

c Hydralazine is an antihypertensive commonly used to treat hypertension in severe preeclampsia. An additional bolus of magnesium sulfate may be ordered for increasing signs of central nervous system irritability related to severe preeclampsia (e.g., clonus) or if eclampsia develops. Diazepam sometimes is used to stop or shorten eclamptic seizures. Calcium gluconate is used as the antidote for magnesium sulfate toxicity. The client is not currently displaying any signs or symptoms of magnesium toxicity.

The indirect Coombs' test is a screening tool for Rh incompatibility. An amniocentesis may be a necessary next step it the titer is greater than what? a. 1:2 b. 1:4 c. 1:8 d. 1:12

c If the maternal titer for Rh antibodies is greater 1:8, then an amniocentesis is indicated to determine the level of bilirubin in the amniotic fluid. This testing will determine the severity of fetal hemolytic anemia.

A pregnant woman at term is transported to the emergency department (ED) after a severe vehicular accident. The obstetric nurse responds and rushes to the ED with a fetal monitor. Cardiopulmonary arrest occurs as the obstetric nurse arrives. What is the highest priority for the trauma team? a. Obtaining IV access, and starting aggressive fluid resuscitation b. Quickly applying the fetal monitor to determine whether the fetus viability c. Starting cardiopulmonary resuscitation (CPR) d. Transferring the woman to the surgical unit for an emergency cesarean delivery in case the fetus is still alive

c In a situation of severe maternal trauma, the systematic evaluation begins with a primary survey and the initial ABCs (airway, breathing, and circulation) of resuscitation. CPR is initiated first, followed by intravenous (IV) replacement fluid. After immediate resuscitation and successful stabilization measures, a more detailed secondary survey of the mother and fetus should be accomplished. Attempts at maternal resuscitation are made, followed by a secondary survey of the fetus. In the presence of multisystem trauma, a cesarean delivery may be indicated to increase the chance for maternal survival

Which minerals and vitamins are usually recommended to supplement a pregnant woman's diet? a. Fat-soluble vitamins A and D b. Water-soluble vitamins C and B6 c. Iron and folate d. Calcium and zinc

c Iron generally should be supplemented, and folic acid supplements often are needed because folate is so important. Fat-soluble vitamins should be supplemented as a medical prescription, as vitamin D might be for lactose-intolerant women. Water-soluble vitamin C sometimes is consumed in excess naturally; vitamin B6 is prescribed only if the woman has a very poor diet. Zinc is sometimes supplemented; most women get enough calcium.

How would the nurse differentiate a meconium stool from a transitional stool in the healthy newborn? a. Observed at age 3 days b. Is residue of a milk curd c. Passes in the first 12to 24 hours of life d. Is lighter in color and looser in consistency

c Meconium stool is usually passed in the first 12 hours of life, and 99% of newborns have their first stool within 48 hours. If meconium is not passed by 48 hours, then obstruction is suspected. Meconium stool is the first stool of the newborn and is made up of matter remaining in the intestines during intrauterine life. Meconium is dark and sticky.

The nurse is preparing to administer methotrexate to the client. This hazardous drug is most often used for which obstetric complication? a. Complete hydatidiform mole b. Missed abortion c. Unruptured ectopic pregnancy d. Abruptio placentae

c Methotrexate is an effective nonsurgical treatment option for a hemodynamically stable woman whose ectopic pregnancy is unruptured and measures less than 4 cm in diameter. Methotrexate is not indicated or recommended as a treatment option for a complete hydatidiform mole, for a missed abortion, or for abruptio placentae.

The nurse is evaluating the new mother's knowledge about appropriate infant bottle feeding techniques. Which statement by the client reassures the nurse that correct learning has taken place? a. "Since reaching 2 weeks of age, I add rice cereal to my daughter's formula to ensure adequate nutrition." b. "I warm the bottle in my microwave oven." c. "I burp my daughter during and after the feeding and as needed." d. "I refrigerate any leftover formula for the next feeding."

c Most infants swallow air when fed from a bottle and should be given a chance to burp several times during and after the feeding.

A nurse providing care for the antepartum woman should understand that the contraction stress test (CST): a. Sometimes uses vibroacoustic stimulation. b. Is an invasive test; however, contractions are stimulated. c. Is considered to have a negative result if no late decelerations are observed with the contractions. d. Is more effective than nonstress test (NST) if the membranes have already been ruptured.

c No late decelerations indicate a positive CST result. Vibroacoustic stimulation is sometimes used with NST. CST is invasive if stimulation is performed by IV oxytocin but not if by nipple stimulation. CST is contraindicated if the membranes have ruptured.

A pregnant woman has been diagnosed with oligohydramnios. Which presentation would the nurse suspect to find on physical examination? a. Fetus is in breech position b. FHR baseline is within normal range c. Fetus with possible renal problems d. Increased fundal height

c Oligohydramnios reflects a decrease in the amount of amniotic fluid and is associated with renal abnormalities in the fetus and compromised fetal well-being. The position of the fetus is due to gestational age and the maternal uterine environment. FHR may be within normal range because it is affected by gestational age and fetal well-being. An increase in fundal height would be associated with polyhydramnios and/or gestational age assessment.

The management of the pregnant client who has experienced a pregnancy loss depends on the type of miscarriage and the signs and symptoms. While planning care for a client who desires outpatient management after a first-trimester loss, what would the nurse expect the plan to include? a. Dilation and curettage (D&C) b. Dilation and evacuation (D&E) c. Misoprostol d. Ergot products

c Outpatient management of a first-trimester loss is safely accomplished by the intravaginal use of misoprostol for up to 2 days. If the bleeding is uncontrollable, vital signs are unstable, or signs of infection are present, then a surgical evacuation should be performed. D&C is a surgical procedure that requires dilation of the cervix and scraping of the uterine walls to remove the contents of pregnancy. This procedure is commonly performed to treat inevitable or incomplete abortion and should be performed in a hospital. D&E is usually performed after 16 weeks of pregnancy. The cervix is widely dilated, followed by removal of the contents of the uterus. Ergot products such as Methergine or Hemabate may be administered for excessive bleeding after miscarriage.PTS:1DIF:

What marks on a baby's skin may indicate an underlying problem that requires notification of a physician? a. Mongolian spots on the back b. Telangiectatic nevi on the nose or nape of the neck c. Petechiae scattered over the infant's body d. Erythema toxicum neonatorum anywhere on the body

c Petechiae (bruises) scattered over the infant's body should be reported to the pediatrician because they may indicate underlying problems such as infection or low platelet count. Mongolian spots are bluish-black spots that resemble bruises but gradually fade over months and have no clinical significance. Telangiectatic nevi (stork bites, angel kisses) fade by the second year and have no clinical significance. Erythema toxicum neonatorum is an appalling-looking rash; however, it has no clinical significance and requires no treatment.

Several metabolic changes occur throughout pregnancy. Which physiologic adaptation of pregnancy will influence the nurse's plan of care? a. Insulin crosses the placenta to the fetus only in the first trimester, after which the fetus secretes its own b. Women with insulin-dependent diabetes are prone to hyperglycemia during the first trimester because they are consuming more sugar c. Duringthe second and third trimesters, pregnancy exerts a diabetogenic effect that ensures an abundant supply of glucose for the fetus d. Maternal insulin requirements steadily decline during pregnancy.

c Pregnant women develop increased insulin resistance during the second and third trimesters. Insulin never crosses the placenta; the fetus starts making its own around the 10th week. As a result of normal metabolic changes during pregnancy, insulin-dependent women are prone to hypoglycemia (low levels). Maternal insulin requirements may double or quadruple by the end of pregnancy.

In caring for the woman with disseminated intravascular coagulation (DIC), what order should the nurse anticipate? a. Administration of steroids b. Preparation of the woman for invasive hemodynamic monitoring c. Administration of blood d. Restriction of intravascular fluids

c Primary medical management in all cases of DIC involves correction of the underlying cause, volume replacement (not volume restriction), blood component therapy, optimization of oxygenation and perfusion status, and continued reassessment of laboratory parameters. Central monitoring would not be ordered initially in a woman with DIC because it could contribute to more areas of bleeding. Steroids are not indicated for the management of DIC.

A primigravida is being monitored at the prenatal clinic for preeclampsia. Which finding is of greatestconcern to the nurse? a. Blood pressure (BP) increase to 138/86 mm Hg b. Weight gain of 0.5 kg during the past 2 weeks c. Dipstick value of 3+ for protein in her urine d. Pitting pedal edema at the end of the day

c Proteinuriais defined as a concentration of 1+ or greater via dipstick measurement. A dipstick value of 3+ alerts the nurse that additional testing or assessment should be performed. A 24-hour urine collection is preferred over dipstick testing attributable to accuracy. Generally, hypertension is defined as a BP of 140/90 mm Hg or an increase in systolic pressure of 30 mm Hg or diastolic pressure of 15 mm Hg. Preeclampsia may be demonstrated as a rapid weight gain of more than 2 kg in 1 week. Edema occurs in many normal pregnancies, as well as in women with preeclampsia. Therefore, the presence of edema is no longer considered diagnostic of preeclampsia.

A pregnant woman has been receiving a magnesium sulfate infusion for treatment of severe preeclampsia for 24 hours. On assessment, the nurse finds the following vital signs: temperature 37.3° C, pulse rate 88 beats per minute, respiratory rate 10 breaths per minute, BP 148/90 mm Hg, absent deep tendon reflexes (DTRs), and no ankle clonus. The client complains, "I'm so thirsty and warm." What is the nurse's immediateaction? a. To call for an immediate magnesium sulfate level b. To administer oxygen c. To discontinue the magnesium sulfate infusion d. To prepare to administer hydralazine

c Regardless of the magnesium level, the client is displaying the clinical signs and symptoms of magnesium toxicity. The first action by the nurse should be to discontinue the infusion of magnesium sulfate. In addition, calcium gluconate, the antidote for magnesium, may be administered. Hydralazine is an antihypertensive drug commonly used to treat hypertension in severe preeclampsia. Typically, hydralazine is administered for a systolic BP higher than 160 mm Hg or a diastolic BP higher than 110 mm Hg.

A breastfeeding woman develops engorged breasts at 3 days postpartum. What action will help this client achieve her goal of reducing the engorgement? a. Skip feedings to enable her sore breasts to rest b. Avoid using a breast pump c. Breastfeed her infant every 2-3 hours d. Reduce her fluid intake for 24 hours

c Skipping feedings may cause further swelling and discomfort. If the infant does not adequately feed and empty the breast, then the mother may pump to extract the milk and relieve some of the discomfort.

At a 2-month well-baby examination, it was discovered that an exclusively breastfed infant had only gained 10 ounces in the past 4weeks. The mother and the nurse develop a feeding plan for the infant to increase his weight gain. Which change in dietary management will assist the client in meeting this goal? a. Begin solid foods b. Have a bottle of formula after every feeding c. Have one extra breastfeeding session every 24 hours d. Start iron supplements

c Solid foods should not be introduced to an infant for at least 4 to 6 months. Bottle feeding may cause nipple confusion and may limit the supply of milk. Iron supplements have no bearing on weight gain.

A woman who is 30 weeks of gestation arrives at the hospital with bleeding. Which differential diagnosis would notbe applicable for this client? a. Placenta previa b. Abruptio placentae c. Spontaneous abortion d. Cord insertion

c Spontaneous abortion is another name for miscarriage; it occurs, by definition, early in pregnancy. Placenta previa is a well-known reason for bleeding late in pregnancy. The premature separation of the placenta (abruptio placentae) is a bleeding disorder that can occur late in pregnancy. Cord insertion may cause a bleeding disorder that can also occur late in pregnancy.

Which statement most accurately describes the HELLP syndrome? a. Mild form of preeclampsia b. Diagnosed by a nurse alert to its symptoms c. Characterized by hemolysis, elevated liver enzymes, and low platelets d. Associated with preterm labor but not perinatal mortality

c The acronym HELLP stands for hemolysis (H), elevated liver (EL) enzymes, and low platelets (LP). The HELLP syndrome is a variant of severe preeclampsia and is difficult to identify because the symptoms are not often obvious. The HELLP syndrome must be diagnosed in the laboratory. Preterm labor is greatly increased; therefore, so is perinatal mortality.

A 30-year-old gravida 3, para 2-0-0-2 is at 18 weeks of gestation. Which screening test should the nurse recommend be ordered for this client? a. Biophysical profile (BPP) b. Chorionic villi sampling c. Maternal Serum Alpha-Fetoprotein Screening (MSAFP) screening d. Screening for diabetes mellitus

c The biochemical assessment MSAFP test is performed from week 15 to week 20 of gestation (weeks 16 to 18 are ideal). A BPP is a method of biophysical assessment of fetal well-being in the third trimester. Chorionic villi sampling is a biochemical assessment of the fetus that should be performed from the 10th to 12th weeks of gestation. Screening for diabetes mellitus begins with the first prenatal visit.

The nurse is cognizant of which information related to the administration of vitamin K? a. Vitamin K is important in the production of red blood cells b. Vitamin K is necessary in the production of platelets c. Vitamin K is not initially synthesized because of a sterile bowel at birth d. Vitamin K is responsible for the breakdown of bilirubin and the prevention of jaundice

c The bowel is initially sterile in the newborn, and vitamin K cannot be synthesized until food is introduced into the bowel. Vitamin K is necessary to activateblood-clotting factors. The platelet count in term newborns is near adult levels. Vitamin K is necessary to activate prothrombin and other blood-clotting factors.

A woman at 35 weeks of gestation has had an amniocentesis. The results reveal that surface-active phospholipids are present in the amniotic fluid. The nurse is aware that this finding indicates that: a. The fetus is at risk for Down syndrome b. The woman is at high risk for developing preterm labor c. The lung are mature d. Meconium is present in the amniotic fluid

c The detection of the presence of pulmonary surfactants, surface-active phospholipids, in amniotic fluid has been used to determine fetal lung maturity, or the ability of the lungs to function after birth. This occurs at approximately 35 weeks of gestation. This result is unrelated to Down syndrome and in no way indicates risk for preterm labor. Meconium should not be present in the amniotic fluid.

In the past, factors to determine whether a woman was likely to have a high risk pregnancy were evaluated primarily from a medical point of view. A broader, more comprehensive approach to high risk pregnancy has been adopted. There are now four categories based on threats to the health of the woman and the outcome of pregnancy. Which of the options listed here is not included as a category? a. Biophysical b. Psychosocial c. Geographic d. Environmental

c The fourth category is correctly referred to as the sociodemographic risk category. The factors stem from the mother and her family. Ethnicity may be one of the risks to pregnancy; however, it is not the only factor in this category. Low income, lack of prenatal care, age, parity, and marital status are included. Biophysical is one of the broad categories used for determining risk. It includes genetic considerations, nutritional status, and medical and obstetric disorders. Psychosocial risks include smoking, caffeine, drugs, alcohol, and psychologic status. All of these adverse lifestyles can have a negative effect on the health of the mother or fetus. Environmental risks are those that can affect fertility and fetal development. They include infections, chemicals, radiation, pesticides, illicit drugs, and industrial pollutants.

Which information related to the newborn's developing cardiovascular system should the nurse fully comprehend? a. The heart rate of a crying infant may rise to 120 beats per minute b. Heart murmurs heard after the first few hours are a cause for concern c. The point of maximal impulse (PMI) is often visible on the chest wall d. Persistent bradycardia may indicate respiratory distress syndrome (RDS)

c The newborn's thin chest wall often allows the PMI to be observed. The normal heart rate for infants who are not sleeping is 110 to 160 beats per minute. However, a crying infant could temporarily have a heart rate of 180 beats per minute. Heart murmurs during the first few days of life have no pathologic significance; however, an irregular heart rate beyond the first few hours should be further evaluated. Persistent tachycardia may indicate RDS; bradycardia may be a sign of congenital heart blockage

A pregnant woman at 10 weeks of gestation jogs three or four times per week. She is concerned about the effect of exercise on the fetus. The nurse should tell her: a. "You don't need to modify your exercising any time during your pregnancy." b. "Stop exercising, because it will harm the fetus." c. "You may find that you need to modify your exercise to walking later in your pregnancy, around the seventh month." d. "Jogging is too hard on your joints; switch to walking now."

c The nurse should inform the woman that she may need to reduce her exercise level as the pregnancy progresses. Typically, running should be replaced with walking around the seventh month of pregnancy. Physical activity promotes a feeling of well-being in pregnant women. It improves circulation, promotes relaxation and rest, and counteracts boredom. Simple measures should be initiated to prevent injuries, such as warm-up and stretching exercises, to prepare the joints for more strenuous exercise.

A woman's cousin gave birth to an infant with a congenital heart anomaly. The woman asks the nurse when such anomalies occur during development. Which response by the nurse is most accurate? a. "We don't really know when such defects occur." b. "It depends on what caused the defect." c. "They occur between the third and fifth weeks of development." d. "They usually occur in the first 2 weeks of development."

c The nurse would be aware of when such defects occur. Regardless of the cause, the heart is vulnerable during its period of development, the third to fifth weeks. The cardiovascular system is the first organ system to function in the developing human. Blood vessel and blood formation begins in the third week, and the heart is developmentally complete in the fifth week.

An expectant father confides in the nurse that his pregnant wife, at 10 weeks of gestation, is driving him crazy. "One minute she seems happy, and the next minute she is crying over nothing at all. Is there something wrong with her?" The nurse's best response is: a. "This is normal behavior and should begin to subside by the second trimester." b. "She may be having difficulty adjusting to pregnancy; I will refer her to a counselor I know." c. "This is called emotional lability and is related to hormone changes and anxiety during pregnancy. The mood swings will eventually subside as she adjusts to being pregnant." d. "You seem impatient with her. Perhaps this is precipitating her behavior."

c The statement in C is the most appropriate response because it gives an explanation and a time frame for when the mood swings may stop. The statement in A is an appropriate response but it does not answer the father's question. Mood swings are a normal finding in the first trimester; the woman does not need counseling. The statement in D is judgmental and not appropriate.

Which finding on a prenatal visit at 10 weeks of gestation might suggest a hydatidiform mole? a. Complaint of frequent mild nausea b. Blood pressure of 120/80 mm Hg c. Fundal height measurement of 18 cm d. History of bright red spotting for 1 day, weeks ago

c The uterus in a hydatidiform molar pregnancy is often larger than would be expected on the basis of the duration of the pregnancy. Nausea increases in a molar pregnancy because of the increased production of hCG. A woman with a molar pregnancy may have early-onset pregnancy-induced hypertension. In the client's history, bleeding is normally described as brownish.

Which client exhibits the greatest number of risk factors associated with the development of preeclampsia? a. 30-year-old obese Caucasian with her third pregnancy b. 41-year-old Caucasian primigravida c. 19-year-old African American who is pregnant with twins d. 25-year-old Asian American whose pregnancy is the result of donor insemination

c Three risk factors are present in the 19-year-old African-American client. She has African-American ethnicity, is at the young end of the age distribution, and has a multiple pregnancy. In planning care for this client, the nurse must frequently monitor her BP and teach her to recognize the early warning signs of preeclampsia. The 30-year-old obese Caucasian client has only has one known risk factor: obesity. Age distribution appears to be U-shaped, with women younger than 20 years of age and women older than 40 years of age being at greatest risk. Preeclampsia continues to be more frequently observed in primigravidas; this client is a multigravida woman. Two risk factors are present for the 41-year-old Caucasian primigravida client. Her age and status as a primigravida place her at increased risk for preeclampsia. Caucasian women are at a lower risk than are African-American women. The 25-year-old Asian-American client exhibits only one risk factor. Pregnancies that result from donor insemination, oocyte donation, and embryo donation are at an increased risk of developing preeclampsia.

A patient has undergone an amniocentesis for evaluation of fetal well-being. Which intervention would be included in the nurse's plan of care after the procedure? Select all that apply. a. Perform ultrasound to determine fetal positioning b. Observe the patient for possible uterine contractions c. Administer RhoGAM to the patient if she is Rh negative d. Perform a minicatheterization to obtain a urine specimen to assess for bleeding

c Ultrasound determination of gestational age dating is best done between 14 and 22 weeks. It is less reliable after that period because of variability in fetal size. Standard sets of measurements relative to gestational age are noted around 10 to after 12 weeks and include crown-rump length (after 10), biparietal diameter (after 12), femur length, and head and abdominal circumferences.

A patient who is pregnant already has Type 2 diabetes with a hemoglobin A1c value of 7. The nurse would categorize this patient as having: a. Gestational diabetes b. Insulin-dependent diabetes complicated by pregnancy c. Pregestational diabetes mellitus d. Non-insulin-dependent diabetes with complications

c Pregestational diabetes mellitus is a term used to describe patients with type 1 or type 2 diabetes in whom diabetes existed prior to pregnancy. Gestational diabetes occurs when a woman becomes diabetic during the pregnancy state. Type 2 diabetes is non-insulin-dependent. None of the information presented indicates complications at this point, because the hemoglobin A1c is within normal range, signifying adequate glycemic control

During pregnancy, many changes occur as a direct result of the presence of the fetus. Whichof these adaptations meet this criterion? (Select all that apply.) a. Leukorrhea b. Development of the operculum c. Quickening d. Ballottement e. Lightening

c, d, e

Hypothyroidism occurs in 2 to 3 pregnancies per 1000. Because severe hypothyroidism is associated with infertility and miscarriage, it is not often seen in pregnancy. Regardless of this fact, the nurse should be aware of the characteristic symptoms of hypothyroidism. Which do they include?Select all that apply a. Hot flashes b. Weight loss c. Lethargy d. Decrease in exercise capacity e. Cold intolerance

c, d, e Symptoms include weight gain, lethargy, decrease in exercise capacity, and intolerance to cold. Other presentations might include constipation, hoarseness, hair loss, and dry skin. Thyroid supplements are used to treat hyperthyroidism in pregnancy.

A nurse is reviewing lab results for a client diagnosed with preeclampsia. Which laboratory values would the nurse expect to be present? Select all that apply a. Hemoglobin 8g/dL b. Burr cells c. LDH 100 units/L d. Platelet count of 75,000 e. BUN 25 mg/dL

c, d, e Thrombocytopenia below 100,000, an increase in LDH, and an increase in BUN would be noted. Hemoglobin levels would be increased, but 8 g/dL reflects a decreased level.Burr cells would not be present in preeclampsia but would in HELLP syndrome.

A nurse is reviewing the clnical diagnosis of early pregnancy loss. Which of the following presentations should the nurse identify as being associated with early pregnancy loss, occurring in less than 12 weeks gestation? Select all that apply. a. Infection b. Cystitis c. Chromosomal abnormalities d. Antiphospholipid syndrome e. Caffeine use f. Hypothyroidism

c, d, f 50% of early pregnancy loss results from genetic abnormalities. Hypothyroidism and antiphospholipid syndrome are associated with early pregnancy loss. Caffeine use is associated with second-trimester losses as a result of maternal behavior. Infection is not a likely source of early pregnancy loss. Cystitis in not associated with early pregnancy loss.

A primigravida has just delivered a healthy infant girl. The nurse is about to administer erythromycin ointment in the infant's eyes when the mother asks, "What is that medicine for?" How should the nurse respond? a. "It is an eye ointment to help your baby see you better." b. "It is to protect your baby from contracting herpes from your vaginal tract." c. "Erythromycin is prophylactically given to prevent a gonorrheal infection." d. "This medicine will protect your baby's eyes from drying out over the next few days."

c. "Erythromycin is prophylactically given to prevent a gonorrheal infection." With the prophylactic use of erythromycin, the incidence of gonococcal conjunctivitis has declined to less than 0.5%. Eye prophylaxis is administered at or shortly after birth to prevent ophthalmia neonatorum. Erythromycin has no bearing on enhancing vision, is used to prevent an infection caused by gonorrhea, not herpes, and is given to prevent infection, not for lubrication.

Which options for saying "good-bye" would the nurse want to discuss with a woman who is diagnosed with having a stillborn girl? a. The nurse should not discuss any options at this time; plenty of time will be available after the baby is born. b. "Would you like a picture taken of your baby after birth?" c. "When your baby is born, would you like to see and hold her?" d. "What funeral home do you want notified after the baby is born?"

c. "When your baby is born, would you like to see and hold her?" Mothers and fathers may find it helpful to see their infant after delivery. The parents' wishes should be respected. Interventions and support from the nursing and medical staff after a prenatal loss are extremely important in the healing of the parents. The initial intervention should be directly related to the parents' wishes concerning seeing or holding their dead infant. Although information about funeral home notification may be relevant, this information is not the most appropriate option at this time. Burial arrangements can be discussed after the infant is born.

The nurse is evaluating a neonate who was delivered 3 hours ago by vacuum-assisted delivery. The infant has developed a cephalhematoma. Which statement is most applicable to the care of this neonate? a. Intracranial hemorrhage (ICH) as a result of birth trauma is more likely to occur in the preterm, low-birth-weight infant. b. Subarachnoid hemorrhage (the most common form of ICH) occurs in term infants as a result of hypoxia. c. In many infants, signs of hemorrhage in a full-term infant are absent and diagnosed only through laboratory tests. d. Spinal cord injuries almost always result from vacuum-assisted deliveries.

c. In many infants, signs of hemorrhage in a full-term infant are absent and diagnosed only through laboratory tests. Abnormalities in lumbar punctures or red blood cell counts, for instance, or in visuals on computed tomographic (CT) scans might reveal a hemorrhage. ICH as a result of birth trauma is more likely to occur in the full-term, large infant. Subarachnoid hemorrhage in term infants is a result of trauma; in preterm infants, it is a result of hypoxia. Spinal cord injuries are almost always from breech births; however, spinal cord injuries are rare today because cesarean birth is used for breech presentation.

Which statement most accurately describes complicated grief? a. Occurs when, in multiple births, one child dies and the other or others live b. Is a state during which the parents are ambivalent, as with an abortion c. Is an extremely intense grief reaction that persists for a long time. d. Is felt by the family of adolescent mothers who lose their babies

c. Is an extremely intense grief reaction that persists for a long time. Parents showing signs of complicated grief should be referred for counseling. Multiple births, in which not all of the babies survive, create a complicated parenting situation but not complicated bereavement. Abortion can generate complicated emotional responses, but these responses do not constitute complicated bereavement. Families of lost adolescent pregnancies may have to deal with complicated issues, but these issues are not complicated bereavement.

Providing care for the neonate born to a mother who abuses substances can present a challenge for the health care team. Nursing care for this infant requires a multi system approach. What is the first step in the provision of care for the infant? a. Pharmacologic treatment b. Reduction of environmental stimuli c. Neonatal abstinence syndrome (NAS) scoring d. Adequate nutrition and maintenance of fluid and electrolyte balance

c. Neonatal abstinence syndrome (NAS) scoring. NAS describes the cohort of symptoms associated with drug withdrawal in the neonate. The NAS system evaluates CNS, metabolic, vasomotor, respiratory, and gastrointestinal (GI) disturbances. This evaluation tool enables the health care team to develop an appropriate plan of care. The infant is scored throughout his or her length of stay, and the treatment plan is adjusted accordingly. Pharmacologic treatment is based on the severity of the withdrawal symptoms, which are determined by using a standard assessment tool. Medications of choice are morphine, phenobarbital, diazepam, or diluted tincture of opium. Swaddling, holding, and reducing environmental stimuli are essential in providing care to the infant who is experiencing withdrawal. These nursing interventions are appropriate for the infant who displays CNS disturbances. Poor feeding is one of the GI symptoms common to this client population. Fluid and electrolyte balance must be maintained, and adequate nutrition provided. These infants often have a poor suck reflex and may need to be fed via gavage.

For an infant experiencing symptoms of drug withdrawal, which intervention should be included in the plan of care? a. Administering chloral hydrate for sedation b. Feeding every 4 to 6 hours to allow extra rest between feedings c. Snugly swaddling the infant and tightly holding the baby d. Playing soft music during feeding

c. Snugly swaddling the infant and tightly holding the baby. The infant should be snugly wrapped to reduce self-stimulation behaviors and to protect the skin from abrasions. Phenobarbital or diazepam may be administered to decrease central nervous system (CNS) irritability. The infant should be fed in small, frequent amounts and burped well to diminish aspiration and maintain hydration. The infant should not be stimulated (such as with music), because stimulation will increase activity and potentially increase CNS irritability.

During the initial acute distress phase of grieving, parents still must make unexpected and unwanted decisions about funeral arrangements and even naming the baby. What is the nurse's role at this time? a. To take over as much as possible to relieve the pressure b. To encourage the grandparents to take over c. To ensure that the parents, themselves, approve the final decisions d. To leave them alone to work things out

c. To ensure that the parents, themselves, approve the final decisions. The nurse is always the client's advocate. Nurses can offer support and guidance and yet leave room for the same from grandparents. In the end, however, nurses should let the parents make the final decisions. For the nurse to be able to present options regarding burial and autopsy, among other issues, in a sensitive and respectful manner is essential. The nurse should assist the parents in any way possible; however, taking over all arrangements is not the nurse's role. Grandparents are often called on to help make the difficult decisions regarding funeral arrangements or the disposition of the body because they have more life experiences with taking care of these painful yet required arrangements. Some well-meaning relatives may try to take over all decision-making responsibilities. The nurse must remember that the parents, themselves, should approve all of the final decisions. During this time of acute distress, the nurse should be present to provide quiet support, answer questions, obtain information, and act as a client advocate.

A woman who is 8 months pregnant asks the nurse, "Does my baby have any antibodies to fight infection?" What is the mostappropriate response by the nurse? a. "Your baby has all the immunoglobulins necessary: immunoglobulin G (IgG), immunoglobulin M (IgM),and immunoglobulin A (IgA)." b. "Your baby won't receive any antibodies until he is born and you breastfeed him." c. "Your baby does not have any antibodies to fight infection." d. "Your baby has IgG and IgM."

d

As part of the infant discharge instructions, the nurse is reviewing the use of the infant car safety seat. Which information is the highest priority for the nurse to share? a. Infant carriers are okay to use until an infant car safety seat can be purchased b. For traveling on airplanes, buses, and trains, infant carriers are satisfactory c. Infant car safety seats are used for infants only from birth to 15 pounds d. Infant car seats should be rear facing and placed in the back seat of the car

d

Numerous changes in the integumentary system occur during pregnancy. Which change persists after birth? a. Epulis b. Chloasma c. Telangiectasia d. Striae gravidarum

d

Screening for critical congenital heart disease (CCHD) was added to the uniform screening panel in 2011. The nurse hasexplained this testing to the new mother. Which action by the nurse related to this test is correct? a. Screening is performed when the infant is 12 hours of age b. Testing is performed with an electrocardiogram c. Oxygen (O2) is measured in both hands and in the right foot d. A passing result is an O2 saturation of >95%

d

To reassure and educate their pregnant clients regarding changes in their blood pressure, nurses should be cognizant of what? a. A blood pressure cuff that is too small produces a reading that is too low; a cuff that is too large produces a reading that is too high b. Shifting the client's position and changing from arm to arm for different measurements produces the most accurate composite blood pressure reading at each visit c. Systolic blood pressure slightly increases as the pregnancy advances; diastolic pressure remains constant d. Compression of the iliac veins and inferior vena cava by the uterus contributes to hemorrhoids in the later stage of a term pregnancy

d

What is the most dangerous effect on the fetus of a mother who smokes cigarettes while pregnant? a. Genetic changes and anomalies b. Extensive CNS damage c. Fetal addiction to the substance inhaled d. Intrauterine growth restriction

d

Which gastrointestinalalteration of pregnancy is a normalfinding? a. Insufficient salivation (ptyalism) is caused by increases in estrogen b. Acid indigestion (pyrosis) begins early but declines throughout pregnancy c. Hyperthyroidism often develops (temporarily) because hormone production increases d. Nausea and vomiting rarely have harmful effects on the fetus and may be beneficial

d

Which hormone is essential for maintaining pregnancy? a. Estrogen b. hCG c. Oxytocin d. Progesterone

d

A postpartum woman telephones the provider regarding her 5-day-old infant. The client is not scheduled for another weight check until the infant is 14 days old. The new mother is worried about whether breastfeeding is going well. Which statement indicates that breastfeeding is effective formeeting the infant's nutritional needs? a. Sleeps for 6 hours at a time between feedings b. Has at least one breast milk stool every 24 hours c. Gains 1 to 2 ounces per week d. Has 3 bowel movements and eight wet diapers per day

d After day 4, when the mother's milk comes in, the infant should have six to eight wet diapers every 24 hours. Typically, infants sleep 2 to 4 hours between feedings, depending on whether they are being fed on a 2-to 3-hour schedule or cluster-fed. The infant's sleep pattern is not an indication whether the infant is breastfeeding well. The infant should have a minimum of three bowel movements in a 24-hour period. Breastfed infants typically gain15 to 30 g/day.

Which statement regarding the condition referred to as a miscarriage is most accurate? a. A miscarriage is a natural pregnancy loss before labor begins b. It occurs in fewer than 5% of all clinically recognized pregnancies c. Careless maternal behavior, such as poor nutrition or excessive exercise, can be a factor in causing a miscarriage d. If a miscarriage occurs before the 12th week of pregnancy, then it may be observed only as moderate discomfort and blood loss

d Before the sixth week, the only evidence might be a heavy menstrual flow. After the 12th week, more severe pain, like that of labor, is likely. Miscarriage is a natural pregnancy loss, but it occurs, by definition, before 20 weeks of gestation, before the fetus is viable. Miscarriages occur in approximately 10% to 15% of all clinically recognized pregnancies. Miscarriages can be caused by several disorders or illnesses outside the mother's control or knowledge.

Nurses should be aware of the strengths and limitations of various biochemical assessments during pregnancy, including that: a. Chorionic villus sampling (CVS) is becoming more popular because it provides early diagnosis. b. Screening for maternal serum alpha-fetoprotein (MSAFP) levels is recommended only for women at risk for neural tube defects. c. Percutaneous umbilical blood sampling (PUBS) is one of the quad-screen tests for Down syndrome. d. MSAFP is a screening tool only; it identifies candidates for more definitive procedures.

d CVS does provide a rapid result, but it is declining in popularity because of advances in noninvasive screening techniques. MSAFP screening is recommended for all pregnant women. MSAFP, not PUBS, is part of the quad-screen tests for Down syndrome. MSAFP is a screening tool, not a diagnostic tool. Further diagnostic testing is indicated after an abnormal result.

When counseling a client about getting enough iron in her diet, the maternity nurse should tell her that: a. Milk, coffee, and tea aid iron absorption if consumed at the same time as iron b. Iron absorption is inhibited by a diet rich in vitamin C c. Iron supplements are permissible for children in small doses d. Constipation is common with iron supplements

d Constipation can be a problem with iron supplements. Milk, coffee, and tea actually inhibit iron absorption when consumed at the same time as iron. Vitamin C promotes iron absorption. Children who ingest iron can get very sick and even die.

A 22-year-old woman pregnant with a single fetus had a preconception body mass index (BMI) of 24. When she was seen in the clinic at 14 weeks of gestation, she had gained 1.8 kg (4 lbs) since conception. How would the nurse interpret this finding? a. This weight gain indicates possible gestational hypertension b. This weight gain indicates that the woman's infant is at risk for intrauterine growth restriction (IUGR) c. This weight gain cannot be evaluated until the woman has been observed for several more weeks d. The woman's weight gain is appropriate for this stage of pregnancy

d During the first trimester, the average total weight gain is only 1 to 2.5 kg. The desirable weight gain during pregnancy varies among women. Weight gain should take place throughout the pregnancy. The optimal rate depends on the stage of the pregnancy. The primary factor to consider in making a weight gain recommendation is the appropriateness of the prepregnancy weight for the woman's height. A commonly used method of evaluating the appropriateness of weight for height is the BMI. This woman's BMI is within the normal range, and she has gained the appropriate amount of weight for her size at this point in her pregnancy. Although the statements in A through C are accurate, they do not apply to this client.

What is one of the initial signs and symptoms of puerperal infection in the postpartum client? a. Fatigue continuing for longer than 1 week b. Pain with voiding c. Profuse vaginal lochia with ambulation d. Temperature of 38° C (100.4° F) or higher on 2 successive days

d Fatigue is a late finding associated with infection. Pain with voiding may indicate a urinary tract infection (UTI), but it is not typically one of the earlier symptoms of infection. Profuse lochia may be associated with endometritis, but it is not the first symptom associated with infection.

A nurse is discussing the storage of breast milk with a mother whose infant is preterm and in the special care nursery. Which storage practice indicates that the mother requires additional teaching? a. Keeping breast milk in the deep freezer for up to 6 months b. Freezing breast milk for up to 3 months." c. Allowing breast milk to sit at room temperature for up to 4 hours d. Storing breast milk in the refrigerator for up to 10 days."

d Human milk for the healthy or preterm hospitalized infant can be kept in the refrigerator for up to 8 days or in the freezer for up to 3 months, but only for 4 hours or less at room temperature.

A woman with preeclampsia has a seizure. What is the nurse's highest priority during a seizure? a. To insert an oral airway b. To suction the mouth to prevent aspiration c. To administer oxygen by mask d. To stay with the client and call for help

d If a client becomes eclamptic, then the nurse should stay with the client and call for help. Nursing actions during a convulsion are directed toward ensuring a patent airway and client safety. Insertion of an oral airway during seizure activity is no longer the standard of care. The nurse should attempt to keep the airway patent by turning the client's head to the side to prevent aspiration. Once the seizure has ended, it may be necessary to suction the client's mouth. Oxygen is administered after the convulsion has ended.

In order to reassure and educate pregnant clients about changes in their blood pressure, maternity nurses should be aware that: a. A blood pressure cuff that is too small produces a reading that is too low; a cuff that is too large produces a reading that is too high b. Shifting the client's position and changing from arm to arm for different measurements produces the most accurate composite blood pressure reading at each visit c. The systolic blood pressure increases slightly as pregnancy advances; the diastolic pressure remains constant d. Compression of the iliac veins and inferior vena cava by the uterus contributes to hemorrhoids in the later stage of term pregnancy

d In addition to hemorrhoids, compression of the iliac veins and inferior vena cava by the uterus also leads to varicose veins in the legs and vulva. The tightness of a cuff that is too small produces a reading that is too high; similarly, the looseness of a cuff that is too large results in a reading that is too low. Because maternal positioning affects readings, each blood pressure measurement should be obtained in the same arm and with the woman in the same position. The systolic blood pressure generally remains constant but may decline slightly as pregnancy advances. The diastolic blood pressure first drops and then gradually increases.

Nurses should be aware of the strengths and limitations of various biochemical assessments during pregnancy. Which statement regarding monitoring techniques is the most accurate? a. Chorionic villus sampling (CVS) is becoming more popular because it provides early diagnosis. b.Maternal serum AFP (MSAFP) screening is recommended only for women atrisk for neural tube defects (NTDs) c. Percutaneous umbilical blood sampling (PUBS) is one of the triple-marker tests for Down syndrome d. Maternal serum AFP (MSAFP) is a screening tool only; it identifies candidates for more definitive diagnostic procedures.

d MSAFP is a screening tool, not a diagnostic tool. CVS provides a rapid result, but it is declining in popularity because of advances in noninvasive screening techniques. An MSAFP screening is recommended for all pregnant women. MSAFP screening, not PUBS, is part of the triple-marker tests for Down syndrome.

The nurse advises the woman who wants to have a nurse-midwife provide obstetric care that: a. She will have to give birth at home b. She must see an obstetrician as well as the midwife during pregnancy c. She will not be able to have epidural analgesia for labor pain d. She must be having a low-risk pregnancy

d Midwives usually see low-risk obstetric clients. Nurse-midwives must refer clients to physicians for complications. Most nurse-midwife births are managed in hospitals or birth centers; a few may be managed in the home. Nurse-midwives may practice with physicians or independently with an arrangement for physician backup. They must refer clients to physicians for complications, but patients are not required to see an obstetrician otherwise. Care in a midwifery model is noninterventional, and the woman and family usually are encouraged to be active participants in the care; this does not imply that medications for pain control are prohibited.

A pregnant woman at 33 weeks of gestation is brought to the birthing unit after a minor automobile accident. The client is experiencing no pain and no vaginal bleeding, her vital signs are stable, and the fetal heart rate (FHR) is 132 beats per minute with variability. What is the nurse's highest priority? a. Monitoring the woman for a ruptured spleen b. Obtaining a physician's order to discharge her home c. Monitoring her for 24 hours d. Using continuous electronic fetal monitoring (EFM) for a minimum of 4 hours

d Monitoring theexternal FHR and contractions is recommended after blunt trauma in a viable gestation for a minimum of 4 hours, regardless of injury severity. Fetal monitoring should be initiated as soon as the woman is stable. In this scenario, no clinical findings indicate the possibility of a ruptured spleen. If the maternal and fetal findings are normal, then EFM should continue for a minimum of 4 hours after a minor trauma or a minor automobile accident. Once the monitoring has been completed and the health care provider is reassured of fetal well-being, the client may be discharged home. Monitoring for 24 hours is unnecessary unless the ERM strip is abnormal or nonreassuring.

Over-the-counter (OTC) pregnancy tests usually rely on which technology to test for human chorionic gonadotropin (hCG)? a. Radioimmunoassay b. Radioreceptor assay c. Latex agglutination test d. Enzyme-linked immunosorbent assay (ELISA)

d OTC pregnancy tests use ELISA for its one-step, accurate results. Radioimmunoassays test for the subunit of hCG in serum or urine samples and must be performed in the laboratory. The radioreceptor assay is a serum test that measures the ability of a blood sample to inhibit the binding of hCG to receptors. The latex agglutination test in no way determines pregnancy. Rather it is done to detect specific antigens and antibodies.

A nurse is reviewing clinical manifestations between abruptio placentae and placenta previa. Which finding should the nurse identifying as being the most significant difference between the two? a. Cramping b. Bleeding c. Uterine activity d. Intense abdominal pain

d Pain is absent with placenta previa and may be agonizing with abruptio placentae. Bleeding, uterine activity, and cramping may be present in varying degrees for both placental conditions.

A nurse is monitoring a client's reflexes (DTRs) while receiving magnesium sulfate therapy for treatment of preeclampsia. Which assessment finding if observed by the nurse would indicate a cause for concern? a. DTRs response has been noted at 1+ since onset of therapy b. Client reports no pain upon examination of DTRs by nurse c. Bilateral DTRs noted at 2+ d. Positive clonus response elicited unilaterally

d Positive clonus response elicited unilaterally is a cause for concern as it suggests a hyperactive response. Typically, there is no pain associated with determination of DTRs so this finding would be considered to be normal, as would bilateral DTRs noted at 2+.Even though DTRs at 1+ indicate a sluggish or decreased response, this finding is unchanged since the initiation of therapy. The nurse would continue to monitor.

A 32-year-old primigravida is admitted with a diagnosis of ectopic pregnancy. Which information assists the nurse in developing the plan of care? a. Bed rest and analgesics are the recommended treatment b. She will be unable to conceive in the future c. A D&C will be performed to remove the products of conception. d. Hemorrhage is the primary concern

d Severe bleeding occurs if the fallopian tube ruptures. The recommended treatment is to remove the pregnancy before rupture to prevent hemorrhaging. If the tube must be removed, then the woman's fertility will decrease; however, she will not be infertile. AD&C is performed on the inside of the uterine cavity. The ectopic pregnancy is located within the tubes.

When caring for a pregnant woman with cardiac problems, the nurse must be alert for the signs and symptoms of cardiac decompensation. Which critical findings would the nurse find on assessment of the client experiencing this condition? a. Regular heart rate and hypertension b. Increased urinary output, tachycardia, and dry cough c. Shortness of breath, bradycardia, and hypertension d. Edema, crackles, and cyanosis of nails and lips

d Signs of cardiac decompensation include dyspnea; crackles; an irregular, weak, and rapid pulse; rapid respirations; a moist and frequent cough; generalized edema; increasing fatigue; and cyanosis of the lips and nailbeds.

A pregnant woman at 28 weeks of gestation has been diagnosed with gestational diabetes. The nurse caring for this client understands that: a. Oral hypoglycemic agents can be used if the woman is reluctant to give herself insulin b. Dietary modifications and insulin are both required for adequate treatment c. Glucose levels are monitored by testing urine four times a day and at bedtime d. Dietary management involves distributing nutrient requirements over three meals and two or three snacks

d Small frequent meals over a 24-hour period help decrease the risk for hypoglycemia and ketoacidosis. In some women gestational diabetes can be controlled with dietary modifications alone. Blood, not urine, glucose levels are monitored several times a day. Urine is tested for ketone content; results should be negative.Oral hypoglycemic agents can be harmful to the fetus and less effective than insulin in achieving tight glucose control.

A nurse is providing instruction for an obstetrical patient to perform a daily fetal movement count (DFMC). Which instructions could be included in the plan of care? Select all that apply. a. The fetal alarm signal is reached when there are no fetal movements noted for 5 hours b. The patient can monitor fetal activity once daily for a 60-minute period and note activity c. Monitor fetal activity two times a day either after meals or before bed for a period of 2 hours or until 10 fetal movements are noted d. Count all fetal movements in a 12-hour period daily until 10 fetal movements are noted

d The nonstress test is one of the most widely used techniques to determine fetal well-being and is accomplished by monitoring fetal heart rate in conjunction with fetal activity and movements. An ultrasound requires a full bladder. An amniocentesis is the test after which a pregnant woman should be driven home. A maternal serum alpha-fetoprotein test is used in conjunction with unconjugated estriol levels and human chorionic gonadotropin helps to detect Down syndrome.

The nurse suspects that her postpartum client is experiencing hemorrhagic shock. Which observation indicates or would confirm this diagnosis? a. Absence of cyanosis in the buccal mucosa b. Cool, dry skin c. Calm mental status d. Urinary output of at least 30 ml/hr

d The presence of adequate urinary output indicates adequate tissue perfusion (answer d). The assessment of the buccal mucosa for cyanosis can be subjective (option a). The presence of cool, pale, clammy skin is associated with hemorrhagic shock (option b). Hemorrhagic shock is associated with lethargy, not restlessness.

In comparing the abdominal and transvaginal methods of ultrasound examination, which information should the nurse provide to the client? a. Both require the woman to have a full bladder b. The abdominal examination is more useful in the first trimester c. Initially, the transvaginal examination can be painful d. The transvaginal examination allows pelvic anatomy to be evaluated in greater detail

d The transvaginal examination allows pelvic anatomy to be evaluated in greater detail than the abdominal method and also allows intrauterine pregnancies to be diagnosed earlier. The abdominal examination requires a full bladder; the transvaginal examinationrequires an empty one. The transvaginal examination is more useful in the first trimester; the abdominal examination works better after the first trimester. Neither the abdominal nor the transvaginal method of ultrasound examination should be painful, although the woman will feel pressure as the probe is moved during the transvaginal examination.

A woman arrives at the clinic seeking confirmation that she is pregnant. The following information is obtained: She is 24 years old with a body mass index (BMI) of 17.5. She admits to having used cocaine "several times" during the past year and occasionally drinks alcohol. Her blood pressure is 108/70 mm Hg. The family history is positive for diabetes mellitus and cancer. Her sister recently gave birth to an infant with a neural tube defect (NTD). Which characteristics places this client in a high-risk category? a. Blood pressure, age, BMI b. Drug and alcohol use, age, family history c. Family history, blood pressure (BP), BMI d. Family history, BMI, drug and alcohol abuse

d The woman's family history of an NTD, her low BMI, and her drug and alcohol use abuse are high risk factors of pregnancy. The woman's BP is normal, and her age does not put her at risk. Her BMI is low and may indicate poor nutritional status, which is a high risk.

An expectant couple asks the nurse about intercourse during pregnancy and whether it is safe for the baby. The nurse should tell the couple that: a. Intercourse should be avoided if any spotting from the vagina occurs afterward b. Intercourse is safe until the third trimester c. Safer-sex practices should be used once the membranes rupture d. Intercourse and orgasm are often contraindicated if a history or signs of preterm labor are present

d Uterine contractions that accompany orgasm can stimulate labor and would be problematic if the woman were at risk for or had a history of preterm labor. Some spotting can normally occur as a result of the increased fragility and vascularity of the cervix and vagina during pregnancy. Intercourse can continue as long as the pregnancy is progressing normally. Safer-sex practices are always recommended; rupture of the membranes may require abstaining from intercourse.

A woman at 39 weeks of gestation with a history of preeclampsia is admitted to the labor and birth unit. She suddenly experiences increased contraction frequency of every 1 to 2 minutes, dark red vaginal bleeding, and a tense, painful abdomen. Which clinical change does the nurse anticipate? a. Eclamptic seizure b. Rupture of the uterus c. Placenta previa d. Placental abruption

d Uterine tenderness in the presence of increasing tone may be the earliest sign of placental abruption. Women with preeclampsia are at increased risk for an abruption attributable to decreased placental perfusion. Eclamptic seizures are evidenced by the presence of generalized tonic-clonic convulsions. Uterine rupture exhibits hypotonic uterine activity, signs of hypovolemia, and, in many cases, the absence of pain. Placenta previa exhibits bright red, painless vaginal bleeding.

Which statement best describes chronic hypertension? a. Chronic hypertension is defined as hypertension that begins during pregnancy and lasts for the duration of the pregnancy b. Chronic hypertension is considered severe when the systolic BP is higher than 140 mm Hg or the diastolic BP is higher than 90 mm Hg c. Chronic hypertension is general hypertension plus proteinuria d. Chronic hypertension can occur independently of or simultaneously with preeclampsia

d Women with chronic hypertension may develop superimposed preeclampsia, which increases the morbidity for both the mother and the fetus. Chronic hypertension is present before pregnancy or diagnosed before the 20 weeks of gestation and persists longer than 6 weeks postpartum. Chronic hypertension becomes severe with a diastolic BP of 110 mm Hg or higher. Proteinuria is an excessive concentration of protein in the urine and is a complication of hypertension, not a defining characteristic.

In planning for the care of a 30-year-old woman with pregestational diabetes, the nurse recognizes that the most important factor affecting pregnancy outcome is the: a. Mother's age b. Number of years since diabetes was diagnosed c. The amount of insulin required prenatally d. Degree of glycemic control during pregnancy

d Women with excellent glucose control and no blood vessel disease should have good pregnancy outcomes. Although advanced maternal age may pose some health risks, the most important factor for the woman with pregestational diabetes remains the degree of glycemic control during pregnancy. The number of years since diagnosis and the amount of insulin required are not as relevant to outcomes as the degree of glycemic control.

After giving birth to a stillborn infant, the woman turns to the nurse and says, "I just finished painting the baby's room. Do you think that caused my baby to die?" What is the nurse's most appropriate response? a. "That's an old wives' tale; lots of women are around paint during pregnancy, and this doesn't happen to them." b. "That's not likely. Paint is associated with elevated pediatric lead levels." c. Silence. d. "I can understand your need to find an answer to what caused this. What else are you thinking about?"

d. "I can understand your need to find an answer to what caused this. What else are you thinking about?" The statement "I can understand your need to find an answer to what caused this. What else are you thinking about?" is very appropriate for the nurse. It demonstrates caring and compassion and allows the mother to vent her thoughts and feelings, which is therapeutic in the process of grieving. The nurse should resist the temptation to give advice or to use clichés in offering support to the bereaved. In addition, trying to give bereaved parents answers when no clear answers exist or trying to squelch their guilt feeling does not help the process of grieving. Silence would probably increase the mother's feelings of guilt. One of the most important goals of the nurse is to validate the experience and feelings of the parents by encouraging them to tell their stories and then listening with care. The nurse should encourage the mother to express her thoughts.

Which statement is the most appropriate for the nurse to make when caring for bereaved parents? a. "This happened for the best." b. "You have an angel in heaven." c. "I know how you feel." d. "What can I do for you?"

d. "What can I do for you?" Acknowledging the loss and being open to listening is the best action that the nurse can do. No bereaved parent would find the statement "This has happened for the best" to be comforting in any way, and it may sound judgmental. Nurses must resist the impulse to speak about the afterlife to people in pain. They should also resist the temptation to give advice or to use clichés. Unless the nurse has lost a child, he or she does not understand how the parents feel.

A macrosomic infant is born after a difficult forceps-assisted delivery. After stabilization, the infant is weighed, and the birth weight is 4550 g (9 lb, 6 oz). What is the nurse's first priority? a. Leave the infant in the room with the mother. b. Immediately take the infant to the nursery. c. Perform a gestational age assessment to determine whether the infant is large for gestational age. d. Frequently monitor blood glucose levels, and closely observe the infant for signs of hypoglycemia.

d. Frequently monitor blood glucose levels, and closely observe the infant for signs of hypoglycemia. Regardless of gestational age, this infant is macrosomic (defined as fetal weight more than 4000 g) and is at high risk for hypoglycemia, which affects many macrosomic infants. Blood glucose levels should be frequently monitored, and the infant should be closely observed for signs of hypoglycemia. Close observation can be achieved in the mother's room with nursing interventions. However, depending on the condition of the infant, observation may be more appropriate in the nursery.

A nurse caring for a family during a loss might notice that a family member is experiencing survivor guilt. Which family member is most likely to exhibit this guilt? a. Siblings b. Mother c. Father d. Grandparents

d. Grandparents. Survivor guilt is sometimes felt by grandparents because they feel that the death is out of order; they are still alive, while their grandchild has died. They may express anger that theyare alive and their grandchild is not. The siblings of the expired infant may also experience a profound loss. A young child will respond to the reactions of the parents and may act out. Older children have a more complete understanding of the loss. School-age children are likely to be frightened, whereas teenagers are at a loss on how to react. The mother of the infant is experiencing intense grief at this time. She may be dealing with questions such as, "Why me?" or "Why my baby?" and is unlikely to be experiencing survival guilt. Realizing that fathers can be experiencing deep pain beneath their calm and quiet appearance and may need help acknowledging these feelings is important. This need, however, is not the same as survivor guilt.

What bacterial infection is decreasing because of effective drug treatment? a. Escherichia coli infection b. Tuberculosis c. Candidiasis d. Group B streptococci (GBS) infection

d. Group B streptococci (GBS) infection. Penicillin has significantly decreased the incidence of GBS infection. E. coli may be increasing, perhaps because of the increasing use of ampicillin (resulting in a more virulent E. coli resistant to the drug). Tuberculosis is increasing in the United States and in Canada. Candidiasis is a fairly benign fungal infection.

A pregnant woman at 37 weeks of gestation has had ruptured membranes for 26 hours. A cesarean section is performed for failure to progress. The fetal heart rate (FHR) before birth is 180 beats per minute with limited variability. At birth the newborn has Apgar scores of 6 and 7 at 1 and 5 minutes and is noted to be pale and tachypneic. Based on the maternal history, what is the most likely cause of this newborn's distress? a. Hypoglycemia b. Phrenic nerve injury c. Respiratory distress syndrome d. Sepsis

d. Sepsis The prolonged rupture of membranes and the tachypnea (before and after birth) suggest sepsis. A differential diagnosis can be difficult because signs of sepsis are similar to noninfectious problems such as anemia and hypoglycemia. Phrenic nerve injury is usually the result of traction on the neck and arm during childbirth and is not applicable to this situation. The earliest signs of sepsis are characterized by lack of specificity (e.g., lethargy, poor feeding, irritability), not respiratory distress syndrome.

Parents are often asked if they would like to have an autopsy performed on their infant. Nurses who are assisting parents with this decision should be aware of which information? a. Autopsies are usually covered by insurance. b. Autopsies must be performed within a few hours after the infant's death. c. In the current litigious society, more autopsies are performed than in the past. d. Some religions prohibit autopsy.

d. Some religions prohibit autopsy. Some religions prohibit autopsies or limit the choice to the times when it may help prevent further loss. The cost of the autopsy must be considered; it is not covered by insurance and can be very expensive. There is no rush to perform an autopsy unless evidence of a contagious disease or maternal infection is present at the time of death. The rate of autopsies is declining, in part because of a fear by medical facilities that errors by the staff might be revealed, resulting in litigation.

A family is visiting two surviving triplets. The third triplet died 2 days ago. What action indicates that the family has begun to grieve for the dead infant? a. Refers to the two live infants as twins b. Asks about the dead triplet's current status c. Brings in play clothes for all three infants d. Refers to the dead infant in the past tense

d. refers to the dead infant in the past tense. Accepting that the infant is dead (in the past tense of the word) demonstrates an acceptance of the reality and that the family has begun to grieve. Parents of multiples are challenged with the task of parenting and grieving at the same time. Referring to the two live infants as twins does not acknowledge an acceptance of the existence of their third child. Bringing in play clothes for all three infants indicates that the parents are still in denial regarding the death of the third triplet. The death of the third infant has imposed a confusing and ambivalent induction into parenthood for this couple. If the two live infants are referred to as twins and/or if play clothes for all three infants are still considered, then the family is clearly still in denial regarding the death of one of the triplets.

taking-in phase of postpartum

dependent behaviors like limited readiness to learn, reduced attention span, and wanting to discuss the events of her labor and delivery this stage lasts from the first 24 hours until 2 days after delivery

taking-hold phase of postpartum

dependent-independent behaviors like being talkative, learning how to take care of the baby, and excited about becoming a mother one week after birth

Fasting and postprandial glucose levels should be....

fasting: <95 mg/dL 1hr: <140 mg/dL 2hr: <120 mg/dL

letting-go phase of postpartum

interdependent behaviors like emergence of family unit, sexual intimacy relationship continuing, and defining one's individual role as a mother final stage of postpartum psychosocial adjustment

Nitrous Oxide

laughing gas mild analgesia controlled by mom little to no effect on fetus

A nurse caring for a newborn should be aware that the sensory system least mature at the time of birth is: Hearing Taste Smell Vision

vision


Ensembles d'études connexes

Advanced Physiology of Exercise Exam 1 (Ch. 4&5)

View Set

PROTOCOLS: TRAUMA RESUSCITATION, PROTOCOLS: Trauma, PROTOCOLS: additional Trauma

View Set

History 1700 possible questions final exam 2

View Set

CH 12. CNS Depressants and Muscle Relaxants

View Set

Real Estate Law & Regs - Section 5

View Set